SlideShare a Scribd company logo
1 of 67
ĐẠI HOC THÁI NGUYÊN
TRƯ NG ĐẠI HOC KHOA HOC
Tải tài liệu tại sividoc.com
Viết đề tài giá sinh viên – ZALO:0973.287.149-TEAMLUANVAN.COM
PHÙNG TH± THU HÀ
M T SO DẠNG TOÁN VE DÃY SO SINH B I CÁC
HÀM SO SƠ CAP
LU N VĂN THẠC SĨ TOÁN HOC
Thái Nguyên - 2016
ĐẠI HOC THÁI NGUYÊN
TRƯ NG ĐẠI HOC KHOA HOC
Tải tài liệu tại sividoc.com
Viết đề tài giá sinh viên – ZALO:0973.287.149-TEAMLUANVAN.COM
PHÙNG TH± THU HÀ
M T SO DẠNG TOÁN VE DÃY SO SINH B I CÁC
HÀM SO SƠ CAP
LU N VĂN THẠC SĨ TOÁN HOC
Chuyên ngành: Phương pháp Toán sơ cap
Mã so: 60 46 01 13
NGƯ I HƯ NG DAN KHOA HOC
GS.TSKH. NGUYEN VĂN M U
Thái Nguyên - 2016
i
Viết đề tài giá sinh viên – ZALO:0973.287.149-TEAMLUANVAN.COM
Mnc lnc
M đau 1
1 M t so kien thfíc bo tr ve dãy so 3
1.1 Dãy so, định nghĩa và tính chat . . . . . . . . . . . . . . . . . . . . . 3
1.2 Giới hạn của dãy so . . . . . . . . . . . . . . . . . . . . . . . . . . . . 5
1.3 M®t vài dãy so đ c bi t . . . . . . . . . . . . . . . . . . . . . . . . . . 6
2 M t so phương pháp giải bài toán ve xác định dãy so 10
2.1 Dãy so sinh bởi hàm đa thác . . . . . . . . . . . . . . . . . . . . . . 10
2.2 Dãy so sinh bởi hàm phân thác hǎu t . . . . . . . . . . . . . . . . . 16
2.3 Dãy so sinh bởi hàm cháa căn thác . . . . . . . . . . . . . . . . . . . 22
2.4 Dãy so sinh bởi các hàm lượng giác và siêu vi t . . . . . . . . . . . . 24
3 M t so phương pháp xác định gi i hạn của dãy so 28
3.1 Sả dụng tính đơn đi u và bị ch n đe tính giới hạn của dãy so . . . . 28
3.2 Sả dụng nguyên lý kep đe tính giới hạn của dãy so . . . . . . . . . . 35
3.3 Sả dụng định lý Lagrange đe tính giới hạn của dãy so . . . . . . . . 37
3.4 Xác định giới hạn của dãy tőng . . . . . . . . . . . . . . . . . . . . . 42
4 Các dạng toán khác liên quan đen dãy so 46
4.1 M®t so dạng toán liên quan đen tính chat của dãy so . . . . . . . . 46
4.2 M®t so dạng toán khác . . . . . . . . . . . . . . . . . . . . . . . . . . 57
Ket lu n 62
Tài li u tham khảo 63
1
Viết đề tài giá sinh viên – ZALO:0973.287.149-TEAMLUANVAN.COM
M đau
Dãy so là m®t phan quan trong của chương trình Toán phő thông và trong các
ngành đại so và giải tích toán hoc. Dãy so có m®t vị trí đ c bi t quan trong trong
toán hoc, không chỉ như là m®t đoi tượng đe nghiên cáu mà còn đóng m®t vai
trò như m®t công cụ đac lực của các mô hình rời rạc của giải tích trong lý thuyet
phương trình, lý thuyet xap xỉ, lý thuyet bieu dien. . . Trong chương trình, sách giáo
khoa trung hoc phő thông, n®i dung đe c p đen dãy so rat ít. Vì v y hoc sinh g p
rat nhieu khó khăn trong vi c giải các bài toán liên quan đen dãy so khi tham gia
thi hoc sinh giỏi các cap.
Trong các kỳ thi hoc sinh giỏi Toán cap tỉnh, cap quoc gia, thi Olympic Toán
quoc te, thi Olympic sinh viên giǎa các trường đại hoc và cao đȁng, các bài toán
ve dãy so được đe c p nhieu và thường thu®c loại khó. Các bài toán ve ước lượng;
xác định dãy so và tính giá trị các tőng, tích; các bài toán ve cực trị, xác định
giới hạn dãy hay các tính chat của dãy so thường liên quan đen đ c trưng của dãy
tương áng.
Lu n văn M®t so dạng toán ve dãy so sinh bới các hàm so sơ cap nham nêu m®t
so phương pháp xác định dãy so, giới hạn của dãy so và các bài toán liên quan.
Lu n văn gom có mở đau, bon chương n®i dung, ket lu n và tài li u tham khảo.
Chương 1. M t so kien thfíc bo tr ve dãy so
Chương này trình bày các kien thác liên quan đen dãy so.
Chương 2. M t so phương pháp giải bài toán ve xác định dãy so
Chương này trình bày các bài toán liên quan đen xác định so hạng tőng quát
của dãy so sinh bởi các hàm sơ cap cơ bản đó là hàm đa thác, hàm phân thác hǎu
t , hàm lượng giác, hàm so mũ và hàm so logarit.
Chương 3. M t so phương pháp xác định gi i hạn của dãy so
Chương này trình bày m®t so phương pháp xác định giới hạn của dãy so như
2
Viết đề tài giá sinh viên – ZALO:0973.287.149-TEAMLUANVAN.COM
phương pháp sả dụng tính đơn đi u và bị ch n, phương pháp sả dụng nguyên lí
kep, phương pháp sả dụng định lí Lagrange và xác định giới hạn của dãy tőng.
Chương 4. Các dạng toán khác liên quan đen dãy so
Chương này trình bày m®t so bài toán liên quan đen tính chat của dãy so
nguyên, các dãy so cháa hàm phan nguyên, hàm phan lẻ.
Lu n văn được hoàn thành tại trường Đại hoc Khoa hoc - Đại hoc Thái Nguyên
với sự hướng dan của GS.TSKH. Nguyen Văn M u. Tác giả xin được bày tỏ lòng
biet ơn sâu sac đoi với sự quan tâm hướng dan của thay, tới các thay cô trong
Ban giám hi u, Phòng đào tạo và Khoa Toán - Tin của trường Đại hoc Khoa hoc.
Đong thời tác giả xin cảm ơn tới Sở Giáo dục và đào tạo Yên Bái, Ban giám hi u
và các thay cô trường THPT Chuyên Nguyen Tat Thành đã tạo đieu ki n cho tác
giả hoc t p và hoàn thành ke hoạch hoc t p.
Thái Nguyên, ngày 25 tháng 5 năm 2016.
Hoc viên
Phùng Thị Thu Hà
3
Viết đề tài giá sinh viên – ZALO:0973.287.149-TEAMLUANVAN.COM
Chương 1
M t so kien thfíc bo tr ve dãy so
Trong chương này, tôi trình bày khái ni m cơ bản ve dãy so gom m®t so định
nghĩa và các định lý cơ bản, m®t vài dãy so đ c bi t và m®t so bài toán áp dụng.
1.1 Dãy so, định nghĩa và tính chat
Định nghĩa 1.1. Dãy so (thực) là m®t hàm so xác định trên t p con của t p so
tự nhiên. Với M ⊂ N, thay cho ký hi u
u : M → R
n ›→ u(n)
ta thường dùng ký hi u (un) hay {un} với n ∈ M.
Dãy so được goi là vô hạn neu chúng có vô hạn phan tả. Dãy so được goi là hũu
hạn neu so phan tả của dãy là hǎu hạn. Phan tả ui được goi là phan tả thá i của
dãy.
1.1.1. Dãy so đơn đi u
Dãy (un) được goi là đơn đi u tăng neu un ≤ un+1, với moi n = 1, 2, . . .
Dãy (un) được goi là đơn đi u giảm neu un ≥ un+1, với moi n = 1, 2, . . .
Dãy (un) được goi là tăng thực sự neu un < un+1, với moi n = 1, 2, . . .
Dãy (un) được goi là giảm thực sự neu un > un+1, với moi n = 1, 2, . . .
Dãy đơn đi u tăng và dãy đơn đi u giảm được goi chung là dãy đơn đi u.
4
Viết đề tài giá sinh viên – ZALO:0973.287.149-TEAMLUANVAN.COM
Nh n xét 1.1. • Neu dãy (xn) tăng, dãy (yn) tăng thì dãy (xn + yn) tăng.
• Neu dãy (xn) giảm, dãy (yn) giảm thì dãy (xn + yn) giảm.
• Neu dãy (xn) tăng thì dãy (−xn) giảm, và neu dãy (xn) giảm thì dãy (−xn)
tăng.
• Neu hai dãy so dương (xn), (yn) cùng tăng (giảm) thì dãy (xnyn) tăng (giảm).
• M®t dãy so có the không tăng, cũng không giảm. Ví dụ dãy so (xn) với
xn = (−1)n, ∀n ∈ N.
1.1.2. Dãy so bị ch n
Dãy (un) được goi là bị ch n trên neu ton tại m®t so M sao cho
un ≤ M, ∀n ∈ N∗
.
Dãy (un) được goi là bị ch n dưới neu ton tại m®t so m sao cho
un ≥ m, ∀n ∈ N∗
.
Dãy (un) được goi là bị ch n neu nó vàa bị ch n trên và vàa bị ch n dưới nghĩa
là ton tại m®t so M và m®t so m sao cho
m ≤ un ≤ M, ∀n ∈ N∗
.
1.1.3. Dãy so Cauchy
Định nghĩa 1.2 (xem [5]). Dãy so (un) được goi là dãy Cauchy neu ∀ε > 0, ∃N0 ∈
N : ∀m, n > N0, |un − um| < ε.
Định lj 1.1 (Tiêu chuȁn Cauchy, xem [5]). Dãy so (un) có giới hạn hǎu hạn khi
và chỉ khi nó là dãy Cauchy.
1.1.4. Dãy so tuan hoàn
Định nghĩa 1.3 (xem [3]). Dãy so (un) được goi là m®t dãy so tuan hoàn (c®ng
tính) neu ton tại so nguyên dương l sao cho
un+l = un, ∀n ∈ N. (1.1)
5
Viết đề tài giá sinh viên – ZALO:0973.287.149-TEAMLUANVAN.COM
So nguyên dương l nhỏ nhat đe dãy (un) thỏa mãn (1.1) được goi là chu kỳ cơ
sở của dãy.
Dãy so (un) được goi là m®t dãy phản tuan hoàn (c®ng tính) neu ton tại so
nguyên dương l sao cho
un+l = −un, ∀n ∈ N. (1.2)
So nguyên dương l nhỏ nhat đe dãy (un) thỏa mãn (1.2) được goi là chu kỳ cơ
sở của dãy.
Nh n xét 1.2. a) Dãy tuan hoàn chu kỳ 1 khi và chỉ khi dãy đó là m®t dãy hang.
b) Dãy phản tuan hoàn chu kỳ l là dãy tuan hoàn chu kỳ 2l.
Tương tự, ta cũng có định nghĩa ve dãy tuan hoàn nhân tính.
Định nghĩa 1.4 (xem [3]). Dãy so (un) được goi là m®t dãy tuan hoàn nhân tính
neu ton tại so nguyên dương s (s > 1) sao cho
usn = un, ∀n ∈ N. (1.3)
So nguyên dương s nhỏ nhat đe dãy so (un) thỏa mãn (1.3) được goi là chu kỳ
cơ sở của dãy.
Dãy so (un) được goi là dãy phản tuan hoàn nhân tính neu ton tại so nguyên
dương s (s > 1) sao cho
usn = −un, ∀n ∈ N. (1.4)
So nguyên dương s (s > 1) nhỏ nhat đe dãy so (un) thỏa mãn (1.4) được goi là
chu kỳ cơ sở của dãy.
Nh n xét 1.3. Dãy phản tuan hoàn nhân tính chu kỳ s là m®t dãy tuan hoàn
nhân tính chu kỳ s2.
1.2 Gi i hạn của dãy so
Định nghĩa 1.5 (xem [5]). Ta nói dãy so (un) có giới hạn hǎu hạn a khi n dan tới
vô cùng neu với moi ε > 0, ton tại so tự nhiên N0 (phụ thu®c vào dãy so un và ε )
sao cho với moi n > N0 ta có |un − a| < ε.
lim
n→+∞
un = a ⇔ ∀ε > 0, ∃N0 ∈ N : ∀n > N0, |un − a| < ε.
6
Viết đề tài giá sinh viên – ZALO:0973.287.149-TEAMLUANVAN.COM
—
n
—
n
→+∞
n
Ta nói dãy so (un) dan đen vô cùng khi n dan đen vô cùng neu với moi so thực
dương M lớn tùy ý, ton tại so tự nhiên N0 (phụ thu®c vào dãy so un và M) sao cho
với moi n > N0 ta có |un| > M.
lim
n→+∞
un = ∞ ⇔ ∀M > 0, ∃N0 ∈ N : ∀n > N0, |un| > M.
Dãy so (un) có giới hạn hǎu hạn được goi là dãy h®i tụ. Dãy so không có giới
hạn ho c dan đen vô cùng khi n dan đen vô cùng goi là dãy phân kỳ.
Định lj 1.2 (xem [5]). Giả sả ton tại lim
n→+∞
un = a; lim
n→+∞
vn = b thì
a) lim
n→+∞
b) lim
n→+∞
(un + vn) = lim
n→+∞
(un vn) = lim
→+∞
un + lim
n→+∞
un lim
→+∞
vn = a + b.
vn = a − b.
c) lim
n→+∞
(un.vn) = lim
n→+∞
un. lim
n→+∞
vn = ab.
lim u
u n a
d) neu b /= 0 thì lim n
=
→+∞
= .
n→+∞ vn lim vn b
n→+∞
Định lj 1.3. Neu un ≤ vn, ∀n ≥ N0, N0 ∈ N và ton tại
n
lim un = a; lim
n→+∞
vn = b
thì a ≤ b.
Định lj 1.4 (Định lý Weierstrass, xem [5]). a) Neu dãy (un) đơn đi u tăng và bị
ch n trên bởi M thì ton tại giới hạn hǎu hạn lim
n→+∞
un = a và a ≤ M.
b) Neu dãy (un) đơn đi u giảm và bị ch n dưới bởi m thì ton tại giới hạn hǎu
hạn lim
n→+∞
un = a và a ≥ m.
Nói ngan gon hơn, m®t dãy so đơn đi u và bị ch n thì h®i tụ.
Định lj 1.5 (Nguyên lý kep, xem [5]). Neu vn ≤ un ≤ wn, ∀n ≥ N0, N0 ∈ N và
lim
n→+∞
vn = lim
n→+∞
wn = a thì lim
n→+∞
un = a.
1.3 M t vài dãy so đ c bi t
1.3.1. Cap so c ng
Định nghĩa 1.6 (xem [5]). Dãy so (un) được goi là m®t cap so c®ng khi và chỉ khi
ton tại d ∈ R sao cho
∀n ∈ N, un+1 = un + d.
u1 được goi là so hạng đau, d được goi là công sai của cap so c®ng.
7
Viết đề tài giá sinh viên – ZALO:0973.287.149-TEAMLUANVAN.COM
k
· · ·
∀ ∈
Tính chat 1.1. Dãy so (un) là cap so c®ng với công sai d thì
i) un = u1 + (n − 1)d với moi n = 1, 2, . . . ;
ii) uk =
uk−1 + uk+1
2
với moi k = 2, 3, . . . ;
iii) Cho cap so c®ng hǎu hạn u1, u2, . . . , un−1, un. Ta có
u1 + un = u2 + un−1 = u3 + un−2 = . . .
M®t cách tőng quát: u1 + un = uk + un+1−k với moi k = 2, 3, . . . , n − 1.
iv) Đ t Sn = u1 + u2 + · · · + un−1 + un. Ta có
S =
(u1 + un)n
=
[2u1 + (n − 1)d]n
.
n 2 2
1.3.2. Cap so nhân
Định nghĩa 1.7 (xem [5]). Dãy so (un) được goi là m®t cap so nhân khi và chỉ
khi ton tại q ∈ R sao cho
∀n ∈ N, un+1 = unq.
u1 được goi là so hạng đau, q được goi là công b®i của cap so nhân.
Tính chat 1.2. Dãy so (un) là cap so nhân với công b®i q thì
i) un = u1qn−1 với moi n = 1, 2, . . . ;
ii) u2 = uk−1.uk+1 với moi k = 2, 3, . . . ;
iii) Đ t Sn = u1 + u2 + · · · + un−1 + un. Khi q =
/
u1(qn − 1)
1 ta có
Sn = .
q − 1
Nh n xét 1.4. Neu |q| < 1 thì (un) được goi là cap so nhân lùi vô hạn. Tőng của
cap so nhân lùi vô hạn được tính theo công thác
S = u1
1.3.3. Cap so đieu hòa
+ u2 + u3
+ =
u1
.
1 − q
Định nghĩa 1.8 (xem [3]). Dãy so (un) (un 0 với moi n ∈ N) thỏa mãn đieu ki n
un
được goi là cap so đieu hòa.
=
2un−1un+1
, n N∗
un−1 + un+1
8
Viết đề tài giá sinh viên – ZALO:0973.287.149-TEAMLUANVAN.COM
n+1
2 1
−
Bài toán 1.1. Cháng minh rang dãy (un) (un 0 với moi n ∈ N) l p thành m®t
cap so đieu hòa khi và chỉ khi dãy đã cho thỏa mãn đieu ki n
u =
1
, ∀n ∈ N∗
.
Bài giai.
Ta có
un
−
un−1
1
u = ⇔ u =
unun−1
n+1 2 1 n+1 2un−1 − un
un
−
un−1
⇔ un(un−1 + un+1) = 2un−1un+1
⇔ un =
2un−1un+1
.
un−1 + un+1
V y dãy so (un) l p thành m®t cap so đieu hòa.
1.3.4. Dãy so Fibonacci
Định nghĩa 1.9 (xem [5]). Dãy so (un) xác định như sau
u1 = 1, u2 = 1
un = un−1 + un−2, ∀n = 3, 4, . . .
được goi là dãy so Fibonacci.
Dãy Fibonacci có rat nhieu tính chat thú vị và xuat hi n m®t cách tự nhiên
trong nhieu lĩnh vực khác nhau. Chúng ta có công thác sau đây đe tìm so hạng
tőng quát của dãy so Fibonacci:
Công thfíc Binet
1 +
√
5
n
1 −
√
5
n
un = √
5
.
1.3.5. Dãy so sinh b i hàm so
Dãy so dạng xn+1 = f(xn).
Đây là dạng dãy so thường g p nhat trong các bài toán ve giới hạn dãy so. Dãy
so này sě hoàn toàn xác định khi biet giá trị ban đau x0. Do v y sự h®i tụ của dãy
2
2
9
Viết đề tài giá sinh viên – ZALO:0973.287.149-TEAMLUANVAN.COM
m
so sě phụ thu®c vào tính chat của hàm so f (x) và x0. M®t đ c điem quan trong của
dãy so này là neu a là giới hạn của dãy so thì a phải là nghi m của phương trình
x = f(x). Chúng ta có m®t so ket quả cơ bản như sau:
Định nghĩa 1.10 (xem [5]). Hàm so f : D → D được goi là m®t hàm so co trên D
neu ton tại so thực q, 0 < q < 1 sao cho |f(x) − f(y)| ≤ q. |x − y| với moi x, y thu®c
D.
Định lj 1.6 (xem [5]). Neu f (x) là m®t hàm so co trên D thì dãy so (xn) xác định
bởi x0 = a ∈ D, xn+1 = f(xn) h®i tụ. Giới hạn của dãy so là nghi m duy nhat trên
D của phương trình x = f(x).
ChGng minh.
Với moi n > m thì áp dụng định nghĩa hàm so co, ta có
|xn − xm| = |f(xn−1) − f(xm−1)| ≤ q |xn−1 − xm−1| ≤ · · · ≤ q |xn−m − x0| (1.5)
Tà đây |xn − x0| ≤ |xn − xn−1| + · · · + |x1 − x0| ≤ qn−1 + · · · + 1 |x1 − x0|, suy ra
dãy (xn) bị ch n.
Xét ε > 0. Tà (1.5), do q < 1 và |xn−m − x0| bị ch n nên ta suy ra ton tại N sao
cho qN |xn−m − x0| < ε. Suy ra (xn) là dãy Cauchy và do đó nó h®i tụ.
1.3.6. Định lj trung bình Cesaro
Định lj 1.7 (Định lý trung bình Cesaro, xem [5]). Neu lim
n→+∞
xn = a thì
lim
n→+∞
x1 + x2 + · · · + xn
n
= a.
Định lý này có the phát bieu dưới dạng tương đương như sau:
"Neu lim
n→+∞
(xn+1 — xn
) = a thì lim
xn
n→+∞ n
= a."
10
Viết đề tài giá sinh viên – ZALO:0973.287.149-TEAMLUANVAN.COM
n
Σ
−
Chương 2
M t so phương pháp giải bài toán ve
xác định dãy so
Trong chương này ta xét các bài toán liên quan đen xác định so hạng tőng quát
của dãy so sinh bởi các hàm sơ cap cơ bản đó là hàm đa thác, hàm phân thác
hǎu t , hàm vô t , hàm lượng giác và các hàm siêu vi t. Các phương pháp chủ yeu
sả dụng trong chương này là phương pháp ước lượng, phương pháp lượng giác và
phương pháp sả dụng hàm l p.
2.1 Dãy so sinh b i hàm đa thfíc
Bài toán 2.1. Cho a, c > 0. Xét dãy (an) xác định bởi:
a1 = a
an+1 = ca2 + an, ∀n ∈ N∗
.
(2.1)
Cháng minh rang
Bài giai.
Tà giả thiet ta có the viet
an ≥
√
cn−1nnan+1, ∀n ∈ N∗
.
1
−
1
=
can−1
.
V y nên
an−1 an an
n−1
1 1
=
a1 an
k=1
cak
.
ak+1
11
Viết đề tài giá sinh viên – ZALO:0973.287.149-TEAMLUANVAN.COM
Σ
√
‚
.
,
n n
Suy ra
1 1
n−1
= + cak ≥ n
r
n
cn−1 a1
.
a1 an k=1 ak+1
2
Do đó an ≥
√
cn−1nnan+1, ∀n ∈ N∗
.
Bài toán 2.2. Tìm so hạng tőng quát của dãy (un) xác định bởi:
√
3
u1 =
2 (2.2)
Bài giai.
Ta có u
√
3 π
= = cos .
un = 4u3
n−1 — 3un−1, ∀n ≥ 2.
1 2 6
Tà công thác lượng giác cos 3α = 4 cos3 α − 3 cos α, ta có
u = 4 cos3
π
− 3 cos
π
= cos
3π
2
6
3 3π
6 6
3π 32π
u3 = 4 cos
3k−1π
6
− 3 cos
∗
= cos
6 6
Giả sả uk = cos
6
với k ∈ N . Khi đó
uk+1 = 4 cos3 3k−1π
6
− 3 cos
3k−1π
= cos
6
3kπ
.
6
3n−1π ∗
V y bang phương pháp quy nạp ta cháng minh được un = cos
Bài toán 2.3 (xem [1]). Cho dãy (xn) xác định bởi:
6
, ∀n ∈ N .
x1 = 2 +
√
3
xn+1 = x4 − 4x2 + 2, ∀n ∈ N∗
.
(2.3)
n n
Tìm so hạng tőng quát của dãy so (xn).
Bài giai.
Đ t xn = 2yn. Khi đó
2yn+1 = 16y4 − 16y2 + 2, ∀n = 1, 2, . . .
4 2
⇔ yn+1 = 8yn − 8yn + 1, ∀n = 1, 2, . . .
Ta có
y1 =
2 +
√
3
=
2
1 +
√
3
2 =
2
1 + cos
π
6
2
π
= cos .
12
n
a
√
s
12
Viết đề tài giá sinh viên – ZALO:0973.287.149-TEAMLUANVAN.COM
3
12 12 12
k+1
12 12 12
1
12
Do công thác lượng giác cos 4α = 8 cos4 α − 8 cos2 α + 1 nên
y = 8 cos4
π
2 12
2 π
— 8 cos
12
+ 1 = cos 4.
π
.
12
y = 8 cos4 4.
π
− 8 cos2 4.
π
+ 1 = cos 42.
π
.
Bang quy nạp ta sě cháng minh
yn = cos 4n−1.
π
. (2.4)
12
Th t v y, (2.4) đã đúng với n = 2, n = 3 như trên.
Giả sả (2.4) đúng với n = k, k ∈ N∗
tác là yk = cos 4k−1.
π
.
12
Khi đó y = 8 cos4 4k−1.
π
− 8 cos2 4k−1.
π
+ 1 = cos 4k.
π
.
Suy ra (2.4) đúng với n = k + 1.
V y theo nguyên lý quy nạp ta có yn = cos 4n−1.
π
, ∀n = 1, 2, . . .
Do đó xn = 2 cos 4n−1. , ∀n = 1, 2, . . .
π
Bài toán 2.4 (xem [1]). Cho dãy (xn) xác định bởi:
x1 =
2 (2.5)
xn+1 = 16x5 − 20x3 + 5xn, ∀n ∈ N∗
.
Tìm so hạng tőng quát của dãy so (xn).
Bài giai.
Ta có x 1 π
= = cos .
1 2 3
Tà công thác cos 5α = 16 cos5 α − 20 cos3 α + 5 cos α, ta có
x = 16 cos5
π
− 20 cos3
π
+ 5 cos
π
= cos
5π
.
2
3
5 5π
3
3 5π
3 3
5π 52π
Giả sả xn = cos
x3 = 16 cos
5n−1π
. Khi đó
3
3
− 20 cos + 5 cos
3
= cos .
3 3
xn+1 = 16 cos5 5n−1π
3
− 20 cos
5n−1π
+ 5 cos
3
5n−1π
5n−1π
= cos
3
5nπ
.
3
Theo nguyên lý quy nạp, suy ra xn = cos
3
, ∀n = 1, 2, . . .
n
n
12
3
13
Viết đề tài giá sinh viên – ZALO:0973.287.149-TEAMLUANVAN.COM
r
−
r
n
Bài toán 2.5 (Đe thi hoc sinh giỏi Quoc gia 2014). Cho hai dãy so dương (xn), (yn)
xác định bởi: x1 = 1, y1 =
√
3 và
xn+1yn+1 − xn = 0
(2.6)
Cháng minh rang hai dãy so trên h®i tụ và tìm giới hạn của chúng.
Bài giai.
Ta nh n thay
π
x1 = 1 = 2 sin
6
;
y =
√
3 = 2 cos
π
.
1
6
Ta sě cháng minh bang quy nạp rang với moi n nguyên dương thì
π π
xn = 2 sin
3.2n , yn = 2 cos
3.2n . (2.7)
Th t v y, với n = 1 m nh đe (2.7) đúng.
Giả sả đã có xn
π
= 2 sin
3.2n
, yn
π
= 2 cos
3.2n .
Theo công thác truy hoi ta có
xn+1 =
√
2 − yn
xn
= 2 2 cos
π
3.2n
π
2 sin
3.2n
= 4 sin2
π
3.2n+1
π
π
= 2 sin
3.2n+1 ;
yn+1 =
xn+1
= π
2 sin
3.2n+1
= 2 cos
3.2n+1 .
Đieu này cháng tỏ (2.7) đúng với n + 1.
Theo nguyên lý quy nạp ta có
π π
xn = 2 sin
3.2n , yn = 2 cos
3.2n , ∀n = 1, 2, . . .
Do đó lim xn
n→+∞
= lim
n→+∞
2 sin
π
3.2n
= 0 và lim yn
→+∞
= lim
n→+∞
2 cos
π
3.2n
= 2.
V y các dãy (xn), (yn) có giới hạn hǎu hạn và lim
n→+∞
xn = 0 và lim
n→+∞
yn = 2.
Nh n xét 2.1. Đây là bài toán de nhat của kỳ thi. Tuy nhiên, neu không nh n
xét được tính chat đ c bi t của x1, y1 thì rat khó khăn đe giải bài toán này bang
phương pháp lượng giác.
2
n+1
x + yn = 2, ∀n = 1, 2, . . .
14
Viết đề tài giá sinh viên – ZALO:0973.287.149-TEAMLUANVAN.COM
6 6
6
6
6 6
n
Tiep sau đây ta sě xét m®t so bài toán xác định dãy so bang phương pháp sả
dụng hàm l p.
Đe tìm so hạng tőng quát của dãy so (un) bang phương pháp hàm l p ta thường
tìm các hàm so f(x) và h(x) sao cho
f(un+1) = h(f(un)) (2.8)
Sả dụng (2.8) liên tiep ta thu được
f(un+1) = h(f(un)) = h(h(f(un−1))) := h2(f(un−1)) = · · · = hn(f(u1)). (2.9)
Tà (2.9) ta tìm được un. Hàm so f được goi là hàm phụ, còn hàm so h được goi là
hàm l p.
Bài toán 2.6. Tìm so hạng tőng quát của dãy (xn) cho như sau
x1 = 3, xn+1 = 7xn − 1, ∀n = 1, 2, . . .
Bài giai.
Ta có
V y
xn+1
1
—
6
= 7xn
1
— 1 −
6
= 7xn —
7
= 7 x —
1
, ∀n = 1, 2, . . .
x −
1
= 7 xn−1 —
1
= 72 xn−2 —
1
Do đó x
= · · · = 7n−1 x1
=
17
.7n−1.
6
=
17
.7n−1 +
1
, ∀n = 1, 2, . . .
—
1
Nh n xét 2.2. Trong lời giải trên, đieu quan trong là phải biet xét hi u x
Ta có
n+1
1
—
6
.
xn+1 − k = 7xn − 1 − k = 7 xn − k + 6k − 1.
Ta can chon k sao cho 6k −1 = 0 ⇔ k =
1
. V y ta sě xét x n+1
1
—
6
. Có the thay rang
dãy so đã cho có dạng x
phương trình f(x) = x.
n+1 = f(xn
1
), trong đó f(x) = 7x − 1, và so
6
là nghi m của
6
6
6
n
n
15
Viết đề tài giá sinh viên – ZALO:0973.287.149-TEAMLUANVAN.COM
n
2
2
3
n n
Bài toán 2.7 (xem [1]). Cho dãy (un) như sau
u1 = α ∈ R, un+1 = −5u2 − 4un
6
—
5
, ∀n = 1, 2, . . .
Tìm so hạng tőng quát của dãy so đã cho.
Bài giai.
Đ t un
1
= −
5
xn . Khi đó
1 1 2 4 6
—
5
xn+1 = −
5
xn +
5
xn −
5
⇔xn+1 = xn − 4xn + 6
⇔xn+1 − 2 = (xn − 2) .
V y xn − 2 =
2
xn−1 − 2 = xn−2 − 2
22
= · · · = x1 − 2
2n−1
.
So hạng tőng quát của dãy đã cho là un = −
h
2 + (5α + 2)2
i
, ∀n ∈ N∗
.
1 n−1
Bài toán 2.8 (xem [1]). Cho dãy (un) như sau
u1 = α ∈ R, un+1 = 25u3 − 15u2 + 3un, ∀n ∈ N∗
.
Tìm so hạng tőng quát của dãy so đã cho.
Bài giai.
Đ t un =
xn
. Khi đó ta được dãy (x
5
) thỏa mãn x1 = 5α và
1
x =
1
x3 −
3
x2 +
3
x
5
n+1
5 n 5 n 5 n
3 2
⇔xn+1 = xn − 3xn + 3xn
⇔xn+1 − 1 = (xn − 1) .
V y với moi so nguyên dương n, ta có
xn − 1 =
3
xn−1 − 1 = xn−2 − 1
32
= · · · = x1 − 1
3n−1
= 5α − 1
3n−1
.
So hạng tőng quát của dãy đã cho là
3n−1
un =
5α − 1 + 1
5
, ∀n ∈ N∗
.
5
n
16
Viết đề tài giá sinh viên – ZALO:0973.287.149-TEAMLUANVAN.COM
2 2n
n
Bài toán 2.9. Xét dãy (an) thỏa mãn các đieu ki n sau
0 < an < 1
(2.10)
Cháng minh rang
an+1 (1 − an ) ≥
1
, ∀n ∈ N∗
.
1 1
— < a ≤
1
, ∀n ∈ N∗
.
Bài giai.
2 2n n 2
Bang phương pháp quy nạp ta cháng minh
1
−
1
< a .
Th t v y, ta có a =
1
−
1
(đúng). Giả sả
1
−
1
< a . Suy ra
1 2 2 2 2k k
1 1 k + 1
Tà giả thiet
1 − ak <
2
+
2k
=
2k
.
1
suy ra
ak+1(1 − ak) ≥
4
,
1 2k 1 1
ak+1 >
4(1 − a
(đieu phải cháng minh).
)
>
4(k + 1)
=
2
−
2(k + 1)
Đe cháng minh bat đȁng thác an
1
≤
2
, ta sả dụng h thác
1
an(1 − an) ≤
4
≤ an+1(1 − an).
Suy ra an ≥ an+1 và dãy (an
1
) bị ch n trên bởi a với a(1 − a) =
4
.
Do đó a =
1
2
và an
1
≤
2
.
2.2 Dãy so sinh b i hàm phân thfíc hfiu t
Bài toán 2.10 (Đe thi hoc sinh giỏi Quoc gia 2001, bảng B). Cho dãy (xn) xác
định bởi:
2
x1 =
3
xn (2.11)
xn+1 =
2(2n + 1)x
+ 1
, ∀n = 1, 2, . . .
Hãy tính tőng của 2001 so hạng đau tiên của dãy so (xn).
n
4
k
17
Viết đề tài giá sinh viên – ZALO:0973.287.149-TEAMLUANVAN.COM
1 2 2001
1 3
1
3
4002
5 4001 4003
xn+1 =
1 + (1 −
√
2)x
, ∀n ∈ N
1 = tan = tan
4
+
8 8 2 π
1 − tan
8
2
√
Bài giai.
De thay xn > 0, ∀n = 1, 2, . . .
Tà (2.11), ta có
xn+1 =
1
1
2(2n + 1) +
xn
1
⇔
xn+1
1
= 2(2n + 1) + .
xn
Đ t
2
= u
xn
n. Khi đó u1 = 3 và un+1 = 4(2n + 1) + un , ∀n = 1, 2, . . .
Suy ra un+1 − 4(n + 1)2 − 1 = un − 4n2 − 1 = · · · = u1 − (4.12 − 1) = 0
Do đó un = (2n − 1)(2n + 1), ∀n = 1, 2, . . .
V y x 2 2 1 1
= = = − , ∀n = 1, 2, . . .
n
un
nên
(2n − 1)(2n + 1) 2n − 1 2n + 1
x + x + · · · + x =
1
−
1
+
1
−
1
+ · · · +
1
−
1
⇔x1 + x2 + · · · + x2001 = 1 −
4003
=
4003
.
Bài toán 2.11 (xem [1]). Cho dãy (xn) xác định bởi:
x1 =
√
3
xn +
√
2 − 1 ∗
(2.12)
Tìm so hạng tőng quát của dãy (xn).
Bài giai.
Tà công thác tan (x + y) =
tan x + tan y
1 − tan x tan y
, ta có
π
π π π
2 tan
8
π π tan
π
= −1 +
√
2
⇔ tan + 2 tan
8 8
— 1 = 0 ⇔
π
8
tan
8
= −1 − 2
Vì tan
π
> 0 nên tan
π
=
√
2 − 1.
8 8 π
Ta có xn+1 =
xn + tan
8
π , ∀n = 1, 2, . . .
1 − xn. tan
8
n
.
=
18
Viết đề tài giá sinh viên – ZALO:0973.287.149-TEAMLUANVAN.COM
−
∀
−
−
−
π π
+
3 8
1 − tan 3
+ (n − 1)
8
. tan
8
3 8 8 3 8
n
3 8
n
n
Do đó
x =
√
3 = tan
π
.
1
3
π π
tan + tan
3 8 π π
1 − tan
3
tan
8
tan
π
+
π
+ tan
π
3 8 8 π π
x3 = π π π = tan + 2. .
3 8
Bang phương pháp quy nạp, ta sě cháng minh x = tan
π
+(n−1)
π
, ∀n = 1, 2, . . .
Trường hợp n = 1 đã kiem tra ở trên.
Giả sả x = tan
π
+ (n − 1)
π
. Khi đó
n 3 8
n 3 8
tan
π
+ (n 1)
π
+ tan
π
3 8 8
xn+1 = π π π
= tan
hπ
+ (n − 1)
π
+
π
i
= tan
hπ
+ n.
π
i
.
Theo nguyên lý quy nạp, ta được x = tan
hπ
+ (n − 1)
π
i
, ∀n = 1, 2, . . .
Bài toán 2.12 (xem [1]). Cho dãy (xn) như sau
x1 = 5, xn+1
=
5xn + 4
, n = 1, 2, . . .
xn + 2
Cháng minh rang với moi n ∈ N∗
thì xn /= 4. Tìm so hạng tőng quát của dãy (xn).
Bài giai.
Ta có x1 = 5 /= 4. Giả sả xn /= 4, ta cháng minh xn+1 /= 4.
Neu xn+1
= 4 thì
5xn + 4
xn + 2
= 4 ⇔ 5xn + 4 = 4xn + 8 ⇔ xn = 4, mâu thuan với giả
thiet quy nạp. V y xn =
/ 4, ∀n ∈ N∗
.
Ta có
x − 4 =
5xn−1 + 4
− 4 =
xn−1 − 4
;
xn−1 + 2 xn−1 + 2
x + 1 =
5xn−1 + 4
+ 1 =
6(xn−1 + 1)
.
xn−1 + 2 xn−1 + 2
Suy ra
xn + 1
= 6
xn−1 + 1
= 62.
xn−2 + 1
= · · · = 6n−1.
x1 + 1
= 6n.
xn − 4 xn−1 − 4
n n
xn 2 4
4.6n + 1 x1 − 4
∗
Do đó xn + 1 = 6 xn − 4.6 ⇔ xn =
6n 1
, ∀n ∈ N .
4.6n + 1 ∗
V y so hạng tőng quát của dãy (xn) là xn =
6n − 1
, ∀n ∈ N .
x2 = = tan .
1 − tan +
3 8
tan
8
19
Viết đề tài giá sinh viên – ZALO:0973.287.149-TEAMLUANVAN.COM
n
∀ ∈ N
n−1
i
(5 − k) x −
2k − 4
= 2
−
Nh n xét 2.3. Đe sả dụng phương pháp hàm l p ta xét
x − k =
5xn + 4
− k =
5xn − 4 − kxn − 2k
n+1
xn + 2 xn + 2
Ta chon k sao cho
=
(5 − k)xn + 4 − 2k
=
xn + 2
n
xn + 2
5 − k .
2k − 4
= k ⇔ 2k − 4 = 5k − k2 ⇔ k2 − 3k − 4 = 0 ⇔
"
k = −1
5 − k
V y nên trong lời giải trên ta đã xét xn − 4 và xn + 1.
k = 4
Bài toán 2.13 (xem [1]). Tìm so hạng tőng quát của dãy so (xn) cho như sau
Bài giai.
x1 = α ∈ R, xn+1
=
8xn
, n ∗.
4 + x2
Neu α = −2 thì xn = −2, ∀n = 1, 2, . . .
Xét α /= −2. Ta có
2 − xn = 2 −
8xn−1
=
4 + x2
n−1
2x2 − 8xn−1 + 8
4 + x2
n−1
2(2 − xn−1)2
n−1
(1);
2 + xn = 2 + 8xn−1
=
2x2
n−1 + 8xn−1 + 8 2(2 + xn−1)2
(2)
4 + x2
n−1
4 + x2
n−1
4 + x2
n−1
Xét hàm so f(x) =
2 − x
. Tà (1) và (2) ta có
2 + x
f(x ) =
2 − xn
=
n 2 + xn
2 − xn−1
2
2 + xn−1
=
h
f(xn−1)
i
h i2n−1
=
h
f(xn−2)
i22
= · · · =
h
f(x1)
i2n−1
=
h
f(α)
2n−1
.
2 − xn
2 + xn
= β ⇔ 2 − xn = 2β + βxn ⇔ xn =
2 − 2β
.
1 + β
V y, neu α = −2 thì xn = −2, ∀n = 1, 2, . . .
2
h
1 − 2 − α 2n−1
2 + α
neu α −2 thì xn =
2 α 2n−1 , ∀n = 1, 2, . . .
1 +
2 + α
. Ta có
f(α)
Đ t β =
4 + x
=
2
i
20
Viết đề tài giá sinh viên – ZALO:0973.287.149-TEAMLUANVAN.COM
n
−
2
f(xn) =
x
=
+ 2
= f(xn−1)
n
n n n n
Bài toán 2.14 (xem [1]). Tìm so hạng tőng quát của dãy so (xn) cho như sau
x3 + 12xn
Bài giai.
Ta có
x1 = α > 0, xn+1 = n
3x2 + 4
, ∀n ∈ N∗
.
x3
−1 + 12xn−1 x3
−1 − 6x2
−1 + 12xn−1 − 8 (xn−1 − 2)3
xn − 2 = n
3x2 + 4
n−1
— 2 = n
3x2 + 4
n−1
=
3x2 + 4
n−1
. (2.13)
xn + 2 = x3
−1 + 12xn−1 + 2 = x3
n−1
+ 6x2
n−1 + 12xn−1 + 8 (xn−1 + 2)3
. (2.14)
3x2 + 4
n−1
3x2 + 4
n−1
3x2 + 4
n−1
Xét hàm so f(x) =
x − 2
. Tà (2.13) và (2.14) ta có
x + 2
xn − 2 xn−1 − 2 3 h i3
Tà (2.15) ta có
=
h
f(xn−2)
i32
= · · · = h
f(x1)
i3n−1
=
h
f(α)
i3n−1
. (2.15)
xn − 2
xn + 2
.
=
h
f(α)
i3n−1
⇔ xn − 2 =
h
f(α)
i3n−1
.xn + 2.
h
f(α)
i3n−1
2 + 2
h
f(α)
i3n−1
2 + 2 α − 2 3n−1
α + 2
⇔ xn =
1 −
h
f(α)
3n−1 ⇔ xn =
1 −
α − 2 3n−1 , ∀n = 1, 2, . . .
α + 2
α − 2 3n−1
V y so hạng tőng quát của dãy đã cho là xn =
2 + 2
α + 2 , ∀n = 1, 2, . . .
α − 2 3n−1
α + 2
Bài toán 2.15 (xem [1]). Tìm so hạng tőng quát của dãy so (un) cho như sau:
4un(4u2 + 1)
u1 = α ∈ R, un+1 =
n
16u4 + 24u2 + 1 , ∀n ∈ N∗
.
n n
Bài giai.
De thay, với moi so nguyên dương n thì luôn ton tại un.
Neu α =
1
2
thì un
1
= −
2
, ∀n = 1, 2, . . .
Xét α /= −
1
. Ta có
32u3 + 8un
2un+1 + 1 =
16u4 + 32u3 + 24u2 + 8un + 1
+ 1 =
16u4 + 24u2 + 1 16u4 + 24u2 + 1
n n n n
+ 2
n−1
x
n
n
=
i
1 −
21
Viết đề tài giá sinh viên – ZALO:0973.287.149-TEAMLUANVAN.COM
−
2
n
n
−
1
4
2α− 1
n n
∀ ∈ N∗
1 n+1
n
n
n n
n n
n n
−
n
16u4 + 24u2 + 1 16u4 + 24u2 + 1
f(un) =
2u + 1
= − = f(un−1)
n
2un + 1
n n n n
√ √
√ √
4n−1
.
(2un + 1)4
=
16u4 + 24u2 + 1
. (2.16)
M t khác
n n
32u3 + 8un −16u4 + 32u3 − 24u2 + 8un − 1
2un+1 − 1 = − 1 =
n n n n
−(2un + 1)4
=
16u4 + 24u2 + 1
. (2.17)
n n
Xét hàm so f(x) =
2x − 1
. Tà (2.16)và (2.17) ta có
2x + 1
2un − 1 2un−1 − 1 4 h i4
Tà (2.18) ta có
= −
hf(un−2)
i42
= · · · = −
h
f(u1)
i4n−1
= −
hf(α)
i4n−1
. (2.18)
2u − 1
h i4n−1
1 −
h
f(α)
i4n−1
V y, neu α =
1
2
thì un
1
= −
2
, ∀n = 1, 2, . . .
2α − 1 4n−1
neu α /= −
1
thì u = 1 −
2 + 2
2α + 1 , ∀n = 1, 2, . . .
2α + 1
Bài toán 2.16 (xem [1]). Tìm so hạng tőng quát của dãy so (xn) cho như sau
x4 + 12x2 + 4
x = α > 0, x = , n .
4x3 + 8xn
Bài giai.
De thay xn > 0, ∀n ∈ N∗
. Ta có
xn+1 +
x4 + 12x2 + 4
2 = + 2
4x3 + 8xn
x4 + 4
√
2x3 + 12x2 + 8
√
2xn + 4 (xn +
√
2)4
= n n n = . (2.19)
M t khác
4x3 + 8xn 4x3 + 8xn
xn+1 −
x4 + 12x2 + 4
2 = 2
4x3 + 8xn
2 + 2
f(α)
= −
+ 1
n−1
2u
n
⇔ un = h
f(α)
i
22
Viết đề tài giá sinh viên – ZALO:0973.287.149-TEAMLUANVAN.COM
n
= n n n
=
n
−
−
−
r q
1 − u
6
r q
−
−
r
−
x4 − 4
√
2x3 + 12x2 − 8
√
2xn + 4
4x3 + 8xn
(xn −
√
2)4
4x3 + 8xn
. (2.20)
x
√
2
Xét hàm so f(x) =
x +
√
2
, ∀x > 0. Tà (2.19) và (2.20) ta suy ra
xn −
√
2 xn−1 −
√
2 4 h i4
f(xn) =
xn
h
+
√
2
=
i42
xn−1
h
+
√
2
i4n−1
= f(xn−1)
h i4n−1
= f(xn−2) = · · · = f(x1)
h i4n−1
= f(α) . (2.21)
Đ t βn = f(α) . Tà (2.21) ta có
xn −
√
2 √ √
√
2 +
√
2βn
x +
√
2
= βn ⇔ xn − 2 = βnxn + 2βn ⇔ xn = .
1 − βn
So hạng tőng quát của dãy đã cho là
√
2 +
√
2
α
√
2 4n−1
α +
√
2
xn =
α
√
2 4n−1
, ∀n ∈ N∗
.
1 −
α +
√
2
2.3 Dãy so sinh b i hàm chfía căn thfíc
Bài toán 2.17. Tìm so hạng tőng quát của dãy (un) xác định bởi:
1
u1 =
Bài giai.
un =
2
2 − 2 2
n−1
2
, ∀n ≥ 2.
(2.22)
Ta có u 1 π
= = sin .
1 2 6
Tà công thác lượng giác cos2 α = 1 − sin2 α ta có
r
2 − 2
q
1 − sin2
π
r
2 1 − cos
π
Giả sả u
u2 =
2
=
π
= sin .
= sin .
n
6 π
23
Viết đề tài giá sinh viên – ZALO:0973.287.149-TEAMLUANVAN.COM
2 2 .6
n 2n−1.6
2 2 1 sin2
π
2n−1.6
2 1 cos
π
2n−1
.6 π
Khi đó un+1 =
2
=
2
= sin
2n.6
.
V y, theo nguyên lý quy nạp ta cháng minh được un
π
= sin
2n−1.6
.
24
Viết đề tài giá sinh viên – ZALO:0973.287.149-TEAMLUANVAN.COM
n
n+1
3 23 24
k+1 k
2k+1 2k+2
Bài toán 2.18. Tìm so hạng tőng quát của dãy (un) xác định bởi:
Bài giai.
Ta có
u =
√
2 = 2 cos
π
;
1
22
u1 =
√
2
un+1 =
√
2 + un, ∀n ≥ 1.
(2.23)
u =
√
2 +
√
2 =
r
2 1 +
√
2
=
r
2 1 + cos
π
= 2 cos
π
;
2
u =
√
2 + u
2 4 23
=
r
2 1 + cos
π
= 2 cos
π
.
Giả sả uk
π
= 2 cos
2k+1
với k ≥ 1.
Khi đó u =
√
2 + u =
r
2 1 + cos
π
= 2 cos
π
.
V y, theo nguyên lý quy nạp ta cháng minh được un
Bài toán 2.19. Cho dãy (xn) xác định bởi:
π
= 2 cos
2n+1 , ∀n ≥ 1.
1
x1 =
4 xn ∗ (2.24)
xn+1 =
1 + 2xn
Σ
n
+ 2
√
x2 + 2xn
, ∀n ∈ N .
Bài giai.
Tà giả thiet ta có xn > 0, ∀n ∈ N∗
.
Đ t
1
xn
= un . Tà công thác (2.24), ta có u1 = 4 và
un+1 = un + 2 + 2
√
1 + 2un, ∀n ∈ N∗
. (2.25)
Nhân hai ve của (2.25) với 8, roi c®ng hai ve với 4 ta được
4 + 8un+1 = 4 + 8un + 16 + 16
√
1 + 2un, ∀n ∈ N∗
. (2.26)
Đ t vn = 2
√
1 + 2un, vn > 0. Tà (2.26) suy ra v2 = (vn + 4)2, ∀n ∈ N∗
.
Do vn > 0, ∀n ∈ N∗
nên vn+1 = vn + 4, ∀n ∈ N∗
.
V y (vn) là cap so c®ng với công sai d = 4, so hạng đau v1 = 6.
k=1
2
Đ t yn = xk. Tìm so hạng tőng quát của dãy (yn).
25
Viết đề tài giá sinh viên – ZALO:0973.287.149-TEAMLUANVAN.COM
n k
2 n + 1
n n
2 n + 1
k=1
Do đó vn = 6 + (n − 1)4 = 2 + 4n ⇒ un = 2(n2 + n), ∀n ∈ N∗
.
Suy ra
x =
1
=
1 1
−
1
, ∀n ∈ N∗
.
Do đó
n 2n(n + 1) 2 n n + 1
n
y =
Σ
x =
1
1 −
1
, ∀n ∈ N∗
.
V y so hạng tőng quát của dãy (y ) là y =
1
1 −
1
, ∀n ∈ N∗
.
2.4 Dãy so sinh b i các hàm lư ng giác và siêu vi t
2.4.1. Dãy so sinh b i các hàm lư ng giác
Bài toán 2.20. Cho dãy so (xn) xác định như sau:
x0 = a
xn+1 = xn + sin xn, ∀n ∈ N.
(2.27)
Cháng minh rang với moi so thực a dãy (xn) có giới hạn hǎu hạn khi n → +∞.
Bài giai.
Trường hợp 1. Với a = kπ, k ∈ Z. Tà công thác xác định dãy ta có xn = a, ∀n ∈ N.
Do đó dãy đã cho có giới hạn hǎu hạn khi n → +∞ và lim xn = a
Trường hợp 2. Với a /= kπ, k ∈ Z. Xét hàm so
f(x) = x + sin x, x ∈ R.
Khi đó dãy (xn
) được viet lại dưới dạng x0 = a
xn+1 = f(xn), ∀n ∈ N.
Ta có fJ(x) = 1 + cos x ≥ 0, ∀x ∈ R, suy ra f(x) đong bien trên R và do đó dãy
(xn) đơn đi u. Ta xét các khả năng sau:
i) Neu a ∈ (2kπ; (2k + 1)π), k ∈ Z thì sin a > 0 nên dãy (xn) đơn đi u tăng.
Bang quy nạp ta cháng minh xn ∈ (2kπ; (2k + 1)π), n ∈ N.
Th t v y, với n = 0 hien nhiên x0 ∈ (2kπ; (2k + 1)π).
Giả sả đã có xn ∈ (2kπ; (2k + 1)π), n ∈ N. Do f(x) đong bien trên R nên
2kπ = f(2kπ) < f(xn) = xn+1 < f((2k + 1)π) = (2k + 1)π
26
Viết đề tài giá sinh viên – ZALO:0973.287.149-TEAMLUANVAN.COM
nghĩa là có xn+1 ∈ (2kπ; (2k + 1)π).
Theo nguyên lý quy nạp ta có xn ∈ (2kπ; (2k + 1)π), n ∈ N.
Do dãy (xn) đơn đi u tăng và bị ch n trong khoảng (2kπ; (2k + 1)π) nên dãy (xn)
có giới hạn hǎu hạn khi n → +∞.
ii) Neu a ∈ ((2k − 1)π; 2kπ), k ∈ Z thì sin a < 0 nên dãy (xn) đơn đi u giảm. Tương
tự trường hợp i), bang quy nạp ta cháng minh được xn ∈ ((2k − 1)π; 2kπ), n ∈ N.
Do dãy (xn) đơn đi u giảm và bị ch n trong khoảng ((2k − 1)π; 2kπ) nên dãy (xn)
có giới hạn hǎu hạn khi n → +∞.
2.4.2. Dãy so sinh b i hàm siêu vi t
Bài toán 2.21. Tìm so hạng tőng quát của dãy (xn) xác định bởi:
Bài giai.
Đ t yn = xn − 3n, ta được
x0 = 8
xn+1 = 2xn + 3n, ∀n ∈ N.
(2.28)
y0 = 7
yn+1 + 3.3n = 2(yn + 3n) + 3n ⇔
y0 = 7
yn+1 = 2yn.
Tà đó ta có (yn) là cap so nhân với so hạng đau y0 = 7, công b®i q = 2. Theo công
thác tőng quát của cap so nhân ta được yn = 7.2n, suy ra công thác so hạng tőng
quát của dãy (xn) là xn = 7.2n + 3n.
Bài toán 2.22. Tìm so hạng tőng quát của dãy (xn) xác định bởi:
Bài giai.
Đ t xn = yn.7n, ta được
x0 = 101
xn+1 = 7xn + 7n+1, ∀n ∈ N.
(2.29)
x0 = y0.70
yn+1.7n+1
= 7.yn.7n
+ 7n+1 ⇔
y0 = 101
yn+1 = yn + 1.
27
Viết đề tài giá sinh viên – ZALO:0973.287.149-TEAMLUANVAN.COM
n−1
n
n
n+1
n 1
Tà đó ta có (yn) là cap so c®ng với so hạng đau y0 = 101, công sai d = 1. Theo công
thác tőng quát của cap so c®ng ta được yn = 101 + n, suy ra công thác so hạng
tőng quát của dãy (xn) là xn = (101 + n)7n.
Bài toán 2.23 (Đe thi hoc sinh giỏi Quoc gia 2010). Cho dãy (an) xác định bởi:
a1 = 5 (2.30)
an =
q
n
an−1 + 2n−1 + 2.3n−1, ∀n ≥ 2.
a) Tìm so hạng tőng quát của dãy (an).
b) Cháng minh rang (an) là dãy so giảm.
Bài giai.
a) Tà công thác (2.30) ta có an = an−1 + 2n−1 + 2.3n−1
n n−1
Thay lan lượt n bởi n − 1, n − 2, . . . , 2 ta được
an−1
= an−2
+ 2n−2
+ 2.3n−2
n−1 n−2
. . .
a2 = a1 + 21 + 2.31.
2 1
C®ng các đȁng thác trên theo ve và giản ước các so hạng bang nhau ở hai ve ta
được
an = a1 +
Σ
(2k−1 + 2.3k−1) = 5 + 2n − 2 + 3n − 3 = 2n + 3n.
Suy ra an =
√
n
2n + 3n, ∀n ≥ 2.
b) Ta có
an+1 =
√
n
2n + 3n(2n + 3n)
>
√
n
3n(2n + 3n)
= 3.2n + 3n+1 > 2n+1 + 3n+1 = an+1, ∀n = 2, 3, . . .
V y an > an+1, ∀n ≥ 2 nên (an) là dãy so giảm.
Bài toán 2.24. Cháng minh rang neu dãy so (un) là m®t cap so nhân với các so
hạng dương thì dãy so (vn) với vn = loga un, ∀n ∈ N, 0 < a /= 1 sě l p thành m®t
cap so c®ng.
k=2
28
Viết đề tài giá sinh viên – ZALO:0973.287.149-TEAMLUANVAN.COM
Bài giai.
Giả sả (un) là m®t cap so nhân với công b®i q.
Xét dãy so (vn) với vn = loga un, ∀n ∈ N, 0 < a =
/ 1.
Ta có
Khi đó
v0 = loga u0, v1 = loga u1, v2 = loga u2, . . . , vn = loga un.
v1 − v0 = v2 − v1 = v3 − v2 = · · · = vn − vn−1 = loga q.
V y, dãy so (vn) l p thành m®t cap so c®ng với công sai bang loga q.
29
Viết đề tài giá sinh viên – ZALO:0973.287.149-TEAMLUANVAN.COM
∀
n+1
Chương 3
M t so phương pháp xác định gi i
hạn của dãy so
3.1 Sfi dnng tính đơn đi u và bị ch n đe tính gi i hạn
của dãy so
Đe cháng minh dãy so xn+1 = f(xn) h®i tụ bang phương pháp sả dụng tính đơn
đi u và bị ch n, ta có the thực hi n như sau:
Bước 1: Dự đoán xem dãy so đã cho tăng hay giảm (tính giá trị m®t vài so hạng
đau). Giải phương trình L = f(L) đe dự đoán giới hạn L của dãy (xn).
Neu dự đoán dãy tăng thì cháng minh xn ≤ L, ∀n = 1, 2, . . . đe suy ra dãy bị
ch n trên.
Neu dự đoán dãy giảm thì cháng minh xn ≥ L, ∀n = 1, 2, . . . đe suy ra dãy bị
ch n dưới.
Bước 2: Sả dụng ket quả dãy bị ch n ở bước 1 đe cháng minh dãy đơn đi u.
Bước 3: Sả dụng định lý Weierstrass đe cháng minh dãy đã cho h®i tụ.
Bài toán 3.1. Cho dãy (xn) xác định bởi:
x1 = a (a > 1)
x2 + 7 (3.1)
x = n , n = 1, 2, . . .
2(xn + 3)
Cháng minh rang dãy (xn) có giới hạn. Tìm giới hạn đó.
30
Viết đề tài giá sinh viên – ZALO:0973.287.149-TEAMLUANVAN.COM
— −
n+1
— n
−
n+1
n n
Bài giai.
Theo giả thiet x1 = a > 1.
Giả sả xn > 1 với n ∈ N∗
.
Ta có
x2 + 7 (xn − 1)2
x 1 = n 1 =
2(xn + 3) 2(xn + 3)
> 0.
Suy ra xn+1 > 1. Theo nguyên lý quy nạp ta được xn > 1, ∀n ∈ N∗
. Suy ra dãy
(xn) bị ch n dưới bởi 1.
Xét hi u
x2 + 7
x x = x =
2(xn + 3)
(1 − xn)(7 + xn)
2(xn + 3)
< 0.
Suy ra xn+1 < xn, ∀n ∈ N∗
. Do đó dãy so (xn) là dãy giảm.
Vì dãy so (xn) là dãy giảm và bị ch n dưới bởi 1 nên có giới hạn hǎu hạn là b ≥ 1.
Tà công thác (3.1), chuyen qua giới hạn ta được
b2 + 7 2
b =
2(b + 3)
⇔ b + 6b − 7 = 0 ⇔
b = 1
b = −7.
Vì b ≥ 1 nên được b = 1 hay lim xn = 1.
Bài toán 3.2 (Đe thi hoc sinh giỏi Quoc gia 2012). Cho dãy (xn) xác định bởi:
x1 = 3
n + 2 (3.2)
xn =
3n
xn−1 + 2 , ∀n ≥ 2.
Cháng minh rang dãy (xn) có giới hạn. Tìm giới hạn đó.
Bài giai.
Bang quy nạp, ta sě cháng minh với moi n = 3, 4, . . . , ta có
10
Do x2 =
3
n + 2
xn−1 ≥
n − 1
. (3.3)
nên (3.3) đúng khi n = 3.
Giả sả xk−1
k + 2
≥
k − 1
, ∀k ≥ 3.
"
31
Viết đề tài giá sinh viên – ZALO:0973.287.149-TEAMLUANVAN.COM
2 2
k − 1
k
k 3k k−1
k
3k k − 1 k
2ak
2
Với k ≥ 3, xét
x ≥
k + 3
⇔
k + 2
x + 2 ≥
k + 3
⇔
k + 2 k + 2
+ 2 ≥
k + 3
(luôn đúng).
⇔ (k + 2)
k + 2
+ 2 ≥ 3(k + 3)
⇔ (k + 2)3k ≥ 3(k + 3)(k − 1)
⇔ 3k + 6k ≥ 3k + 6k − 9.
V y xk
k + 3
≥
k
. Theo nguyên lý quy nạp, suy ra xn−1
n + 2
≥
n − 1
, ∀n = 3, 4, . . .
De thay rang xn ≥ 0 với moi n nên dãy (xn) bị ch n dưới bởi 0.
Xét xn — xn−1 =
n + 2
x
3n n−1 + 2 − x n−1 =
2[n + 2 − (n − 1)xn−1]
3n
≤ 0, ∀n ≥ 3
(do (3.3).
V y dãy (xn) giảm và bị ch n dưới bởi 0 nên có giới hạn hǎu hạn là a.
Tà công thác (3.2) chuyen qua giới hạn ta được
V y lim
n→+∞
xn = 1.
1
a =
3
(a + 2) ⇔ a = 1
Bài toán 3.3 (Đe thi hoc sinh giỏi Quoc gia 2013). Cho dãy (an) xác định bởi:
a1 = 3
an + 2 (3.4)
an+1 = 3 −
2an
, ∀n ≥ 1.
Cháng minh rang dãy (an) có giới hạn. Tìm giới hạn đó.
Bài giai.
Bang quy nạp, ta sě cháng minh
1 < an < 2, ∀n ≥ 1. (3.5)
3
Do a2 =
2
nên(3.5) đúng khi n = 1, n = 2.
Giả sả (3.5) đúng với n = k > 1 tác là 1 < ak < 2, ta can cháng minh
1 < 3 −
ak + 2
< 2 ⇔
ak + 2
< 2ak
< a + 2.
k
32
Viết đề tài giá sinh viên – ZALO:0973.287.149-TEAMLUANVAN.COM
2
2
2
−
−
∀ ∈
2x
− — −
Xét các hàm so f(x) = 2x − (x + 2) và g(x) = 2x −
x + 2
.
Ta có fJ(x) = 2x ln 2 − 1 > 0, ∀x ∈ (1; 2) và gJ(x) = 2x ln 2 −
1
> 0, ∀x ∈ (1; 2).
Suy ra f và g đeu đong bien trên khoảng (1; 2). Do đó tà 1 < ak < 2 ta có
f(ak) < f(2) ⇒ 2ak
− (ak + 2) < 0
g(ak) > g(1) ⇒ 2ak
−
ak + 2
> 0.
V y (3.5) đúng với moi n ∈ N∗
.
Tiep tục ta sě cháng minh dãy (an) là dãy so tăng.
Xét hàm so h(x) = 3
x + 2
2x
trên khoảng (1; 2).
Ta có hJ(x) =
(x + 2)2x ln 2 2x
22x
=
ln 4 + x ln 2 − 1
> 0, x (1; 2) nên hàm h(x)
2x
đong bien trên khoảng (1; 2).
Ket hợp với a 3
= > a , bang phương pháp quy nạp suy ra dãy (a ) là dãy tăng.
2
2
1 n
Dãy (an) đã cho tăng và bị ch n trên bởi 2 nên có giới hạn hǎu hạn. Giả sả giới
hạn đó là L ∈ (1; 2], ta có
L = 3 −
L + 2
2L
.
Ta sě cháng minh phương trình này có nghi m duy nhat trên (1; 2].
Th t v y, xét hàm so m(x) = 3 −
x + 2
− x, x ∈ (1; 2].
mJ(x) =
ln 4 + x ln 2 − 1
1 =
2x
ln 4 + x ln 2 1 2x
2x .
Ta xét tiep hàm so n(x) = ln 4 + x ln 2 − 1 − 2x, x ∈ (1; 2].
nJ(x) = ln 2(1 − 2x) < 0, ∀x ∈ (1; 2].
nên n(x) là hàm so nghịch bien trên (1; 2].
Suy ra n(x) < n(1) = ln 4 + ln 2 − 3 = ln 8 − 3 < 0 hay ln 4 + x ln 2 − 1 − 2x < 0 với
x ∈ (1; 2].
Do đó, hàm so m(x) nghịch bien trên nảa khoảng (1; 2] và phương trình m(x) = 0
có không quá m®t nghi m trên (1; 2].
M t khác m(2) = 0 nên x = 2 là nghi m duy nhat của phương trình m(x) = 0. Tà
đó suy ra L = 2.
V y lim
n→+∞
an = 2.
33
Viết đề tài giá sinh viên – ZALO:0973.287.149-TEAMLUANVAN.COM
n
n 1 n+1
2
n
4 n
Bài toán 3.4. Cho dãy so (un) xác định bởi:
u1 = u2 = 1
un+1 =
√
un +
√
un−1, ∀n ≥ 2.
(3.6)
Tính lim
n→+∞
Bài giai.
un.
Ta thay u1 = u2, u3 = 1 + 1 = 2 > u2, u4 =
√
u3 +
√
u2 =
√
2 + 1 > u3.
Dự đoán dãy so (un) là dãy so dương và tăng.
Ta sě cháng minh dự đoán bang phương pháp quy nạp, tác là un+1 > un, ∀n ≥ 2.
Rõ ràng un > 0, ∀n ≥ 1. Khi n = 2 ta có u3 = 2 > u1 = 2.
Giả sả uk+1 > uk, ∀k ≥ 2. Ta có
uk+2 =
√
uk+1 +
√
uk >
√
uk +
√
uk−1 = uk+1, ∀k ≥ 2
nên dãy (un) là dãy so dương tăng, suy ra un ≥ u1 = 1, ∀n ≥ 1.
Hơn nǎa, ta thay ∀n ≥ 3, un =
√
un−1 +
√
un−2 <
√
un +
√
un = 2
√
un
hay u2 < 4un ⇒ un < 4 (do un > 0), suy ra dãy (un) bị ch n trên bởi 4.
Dãy so (un) tăng và bị ch n trên nên có giới hạn hǎu hạn. Giả sả lim
n→+∞
un = α.
Khi đó α ≥ 1.
Tà h thác truy hoi suy ra lim
n→+∞
un+1 = lim
n→+∞
√
un + lim
n→+∞
√
un−1 hay α =
√
α +
√
α
suy ra α2 = 4α. Do α ≥ 1 nên α = 4.
V y lim
n→+∞
un = 4.
Bài toán 3.5 (Đe thi hoc sinh giỏi Quoc gia 2015). Cho a là m®t so thực không
âm và dãy (un) xác định bởi:
u1 = 3 (3.7)
un+1 =
1
2
un +
n2
4n2 + a
√
u2 + 3, ∀n ≥ 1.
a) Với a = 0, cháng minh rang dãy (un) có giới hạn. Tìm giới hạn đó.
b) Với moi a ∈ [0; 1], cháng minh rang dãy (un) có giới hạn.
Bài giai.
a) Với a = 0, ta có dãy (u ) xác định bởi u = 3, u =
1
u +
1√
u2 + 3, ∀n ≥ 1.
Xét hàm so f(x) =
1
x +
1√
x2 + 3 với x > 0. Ta có
2 4
fJ(x) =
1
+
1
√
x
> 0, ∀x > 0.
2 4 x2 + 3
n
34
Viết đề tài giá sinh viên – ZALO:0973.287.149-TEAMLUANVAN.COM
√
⇔
n
n
n
2 4
un + u2 + 3 ≤ un+1 ≤ un +
xn+1 =
2
xn +
4
xn + 3 yn+1 =
2
yn +
4n2 + 1
1 1 n
n
Suy ra hàm so f(x) đong bien trên khoảng (0; +∞).
Tà công thác thác truy hoi ta suy ra với moi n thì un > 0, un+1 = f(un) và
3 3
u2 =
2
+
2
< u1, bang phương pháp quy nạp suy ra dãy (un) là dãy giảm và bị
ch n dưới bởi 0 nên nó có giới hạn hǎu hạn.
Đ t lim
n→+∞
un = x, x ≥ 0. Tà công thác (3.7) chuyen qua giới hạn ta được
x =
1
x +
1√
x2 + 3 ⇔
√
x2 + 3 = 2x
V y lim
n→+∞
un = 1.
x ≥ 0
x2 + 3 = 4x2
2 2
⇔ x = 1.
b) Với a ∈ [0; 1], ta có
n
≤
n
≤
1
. Do đó
4n2 + 1
1 n2
4n2 + a 4
√ 1 1√
Xét hai dãy so (xn) và (yn) xác định bởi
x1 = 3
và
y1 = 3
1√
2
2 √
Theo ket quả câu a) ta có lim
n→+∞
xn = 1. Ta sě cháng minh lim
n→+∞
yn = 1 và tà đó
suy ra được ket quả lim
n→+∞
un = 1 và đó là đieu can cháng minh.
Ta có
1 n2 (yn − 1)(yn + 1) 2n2 1
yn+1 − 1 =
2
(yn − 1) +
4n2 + 1 √
y2 + 3 + 2
+
4n2 + 1
−
2
1 n2 (yn − 1)(yn + 1) 1
=
2
(yn − 1) +
4n2 + 1
√
y2 + 3 + 2
−
2(4n2 + 1)
1 n2 (yn − 1)(yn + 1)
<
2
(yn − 1) +
4n2 + 1
√
y2 + 3 + 2
1 n2 (yn + 1)
De thay
= (yn − 1) 2
+
4n2 + 1
√
y2 + 3 + 2
.
1 n2 (yn + 1) 1 n2 1 1 3
0 <
2
+
4n2 + 1
√
y2 + 3 + 2
<
2
+
4n2 + 1
<
2
+
4
=
4
, ∀n = 1, 2, . . .
Neu yn > 1, ∀n thì ta có |yn+1 — 1|
3
<
4
. |yn — 1| < . . . <
3 n−1
|y1 — 1|.
Tà đó suy ra lim
n→+∞
yn = 1.
4
n
2
4n
2 n
n
n
+ 1 2 4
u2 + 3.
y2 + 3.
.
35
Viết đề tài giá sinh viên – ZALO:0973.287.149-TEAMLUANVAN.COM
→+∞
≥
uk
n n
k
uk
Neu ton tại n0 ∈ N∗
đe yn0 ≤ 1 thì ta có
n
lim yn < 1, ∀n > n0.
2
Neu ton tại N > n0 sao cho yN+1 > yN
, khi đó do f(x) =
x
4x2 + 1
là m®t hàm so
đong bien trên khoảng (0; +∞) nên ta có
(N + 1)2 N2
Suy ra
4(N + 1)2 + 1
>
4N2 + 1
.
1 (N + 1)2 q
2 1 N2 q
2
2
yN+1 +
4(N + 1)2 + 1
hay yN+2 > yN+1.
yN+1 + 3 >
2
yN +
4N2 + 1
yN + 3
Bang phương pháp quy nạp ta cháng minh được dãy (yn) tăng tà so hạng thá N
trở đi. Do đó dãy (yn) h®i tụ. Đ t lim
n→+∞
yn = y ta tìm được y = 1.
Neu dãy (yn) là dãy giảm ke tà so hạng thá n0 trở đi, ket hợp với dãy (yn) bị
ch n dưới bởi 0 ta suy ra dãy (yn) h®i tụ. Đ t lim
n→+∞
yn = y ta được y = 1 nên
trường hợp này loại.
V y lim
n→+∞
yn = 1 và bài toán được cháng minh.
Bài toán 3.6. Cho a > 0, dãy so (un) xác định bởi:
u1 > 0
1 a
(3.8)
Tính lim
n→+∞
un.
un+1 =
2 un + n
, ∀n ≥ 1.
Nh n xét 3.1. Với ví dụ này, vi c tìm được công thác của so hạng tőng quát của
dãy so là khá khó khăn nên ta sě cháng minh rang dãy (un) là dãy so giảm và bị
ch n dưới bởi
√
a.
Bài giai.
Th t v y, theo bat đȁng thác AM - GM ta có u2
=
1
u
2
1
+
a √
a.
u1
Giả sả uk ≥
√
a, ∀k ≥ 2, ta cháng minh uk+1 ≥
√
a.
Theo bat đȁng thác AM - GM và giả thiet quy nạp ta có
uk+1
=
1
u
2 k +
a
≥
r
u .
a =
√
a.
Do đó un ≥
√
a, ∀n ≥ 2, nên dãy (un) bị ch n dưới bởi
√
a.
M t khác, ta có
un+1
=
1
+
a
mà u ≥
√
a, ∀n ≥ 2, suy ra
1
≤
1
.
un 2 2u2 n
2u2 2a
u
36
Viết đề tài giá sinh viên – ZALO:0973.287.149-TEAMLUANVAN.COM
n
4
4
0 < un =
u
.
u
· · ·
u
. · · ·
4
.u1 =
4
, ∀n ∈ N
n
Do đó
un+1 1 a
= + 1 a
≤ + = 1 ⇒ u ≤ u , ∀n ≥ 1 nên dãy (u ) là dãy giảm.
un 2 2u2 2 2a n+1 n n
V y dãy so (un) có giới hạn hǎu hạn.
Giả sả lim
n→+∞
un = α, khi đó α > 0. Tà h thác truy hoi suy ra
lim un+1 = lim 1
u +
a
⇒
1
α +
a
⇒ α =
√
a (do α > 0).
n→+∞ n→+∞ 2 un 2 α
V y lim
n→+∞ un =
√
a.
3.2 Sfi dnng nguyên lj kep đe tính gi i hạn của dãy
so
Bài toán 3.7. Cho dãy so (un) xác định bởi:
1
u1 =
4
Tìm lim
n→+∞
Bài giai.
un.
un+1 = u2 + un
2
, ∀n ≥ 1.
Bang quy nạp ta de dàng cháng minh được un > 0, ∀n ∈ N∗
.
1
Ta cũng cháng minh un ≤ , ∀n ∈ N∗
.
Với n = 1 thì u1
1
= , m nh đe đúng.
4
Giả sả uk
1
≤
4
, ∀k ≥ 1, ta cháng minh uk+1
1
≤
4
.
Th y v y, u 1 2 1 3
≤ ⇒ u ≤ u và u 3 1 3
≤ . = .
Do đó
k 4 k 4 k 4 k 4 4 16
1 1 3 3 1
uk+1 ≤
4
uk +
2
uk =
4
uk ≤
16
<
4
.
1
Theo nguyên lý quy nạp, ta có 0 < un ≤ , ∀n ∈ N∗
.
Suy ra
un+1
= u +
1
≤
1
+
1
=
3
, ∀n ∈ N∗
.
un
Do đó, ta có
n
2
un
4 2 4
un−1 u2 3 3 3 1 3 n−1
∗
Mà lim 1 3 n−1
= 0, nên theo nguyên lý kep thì lim un = 0.
n→+∞ 4 4 n→+∞
4
4
4
1
n−2
n−1
n
.u1 ≤ .
37
Viết đề tài giá sinh viên – ZALO:0973.287.149-TEAMLUANVAN.COM
2
n
n
√
u + 1
n
0 < un =
u
.
u
· · ·
u .u1 ≤ . . =
n n
u2 + 1
a2 + 1
Nh n xét 3.2. Bài toán trên khó tìm được so hạng tőng quát. Có the giải bài
toán trên bang cách chỉ ra dãy so trên là dãy so giảm và bị ch n dưới bởi 0.
Bài toán 3.8. Cho dãy so (un) xác định bởi:
1
u1 =
2 un
Tìm lim
n→+∞
un.
un+1 =
n + 1
, ∀n ≥ 1.
Nh n xét 3.3. Vi c xác định công thác tőng quát của dãy so là khá khó khăn.
Do đó đe tìm giới hạn của dãy so ta đánh giá t so
un+1
un
tà đó dùng định lý giới
hạn kep đe tìm giới hạn.
Bài giai.
Bang quy nạp ta cháng minh được un > 0, ∀n ≥ 1.
Tà h thác truy hoi, ta có
un+1 1 1
= ≤ , ∀n ≥ 1.
Ta có
un
un
un−1
n + 1 2
u2 1 1 1 1 1 n
Mà lim
n→+∞
1 n
= 0 nên theo nguyên lý giới hạn kep thì lim
n→+∞
un = 0.
Bài toán 3.9. Cho dãy so (un) xác định bởi:
u1 = a, (−1 < a < 0)
un + 1 (3.9)
un+1 = √
u2 + 1
− 1, ∀n ≥ 1.
1
a) Cháng minh rang 0 < un+1 + 1 ≤ √
a2 + 1
(un + 1), ∀n ≥ 1.
b) Tính lim
n→+∞
un.
Bài giai.
Bang cháng minh quy nạp ta có −1 < un < 0, ∀n ∈ N∗
.
Tà đó suy ra 0 < un + 1 < 1 và
√
u2 + 1 > 1.
un + 1
Suy ra un+1 =
2
− 1 < (un + 1) − 1 = un, ∀n ≥ 1, nên dãy (un) là dãy giảm.
n
Do đó −1 < un ≤ un−1 ≤ · · · ≤ u1 = a < 0, ∀n ≥ 1.
⇒ u2 ≥ a2 ⇒
√
u2 + 1 ≥
√
a2 + 1 ⇒ √
1
1
≤ √ .
2
2
2
2
2
1
n−2
n−1
· · · , ∀n ≥ 1.
.
38
Viết đề tài giá sinh viên – ZALO:0973.287.149-TEAMLUANVAN.COM
n
∈
| |
n
xn+1 = f(xn), ∀n ∈ N∗
.
0 < < 1
a2 + 1
lim
n→+∞
(a + 1)
a2 + 1
1 = 1
Suy ra
un + 1 1
0 < un+1 + 1 = √
u2 + 1
≤ √
a2 + 1
(un + 1), ∀n ≥ 1.
⇒ 0 < un + 1
1
≤ √
a2 + 1
(un−1 + 1) ≤ √
1
a2 + 1
2
(un−2 + 1)
≤ · · · ≤ √
1
n−1
∀ ≥
hay −1 < un ≤ √
1
a2 + 1
n−1
.(a + 1) − 1, ∀n ≥ 1.
a2 + 1
(u1 + 1), n 1.
1
Vì √ nên
h
√
1 n−1
−
i
− .
Do đó, theo nguyên lý giới hạn kep ta có lim
n→+∞
un = −1.
3.3 Sfi dnng định lj Lagrange đe tính gi i hạn của
dãy so
Định nghĩa 3.1 (xem [5]). Hàm so y = f(x) xác định trên khoảng (a; b) được goi
là liên tục tại x0 (a; b) neu lim
x→x0
f(x) = f(x0).
Nh n xét 3.4. Hàm so sơ cap liên tục trên t p xác định của nó.
Định lj 3.1. Neu hàm so y = f (x) liên tục trên đoạn [a; b] và f (a).f (b) < 0 thì
phương trình f(x) = 0 có nghi m trên khoảng (a; b). Neu thêm giả thiet f(x) đơn
đi u trên đoạn [a; b] thì phương trình f (x) = 0 có đúng m®t nghi m trên khoảng
(a; b).
Định lj 3.2 (Định lý Lagrange, xem [5]). Neu hàm so y = f(x) liên tục trên đoạn
[a; b] và có đạo hàm trên khoảng (a; b) thì ton tại m®t so c ∈ (a; b) sao cho
f(b) − f(a) = fJ(c)(b − a).
Định lj 3.3. Neu q là so mà q < 1 thì lim
→+∞
qn = 0.
Khi g p dãy so (xn) xác định bởi:
x1 = a
với f(x) là hàm so xác định trên D và |fJ(x)| < 1, ∀x ∈ D, phương trình f(x) = x có
nghi m duy nhat trên t p con C nào đó của D, ta có the sả dụng định lý Lagrange.
39
Viết đề tài giá sinh viên – ZALO:0973.287.149-TEAMLUANVAN.COM
n
n
≥
x
3
3
=
1
x
3
+
125 3
x2
125
. 2 = 5.
n
3 x2
3 3x3 3
5
x2
Khi đó xn+1 n + xn xn.xn
Bài toán 3.10. Cho dãy (xn) xác định bởi:
x1 = a (a > 0) (3.10)
xn+1
=
1
2x
3
n +
125
, ∀n ∈ N∗
.
Cháng minh rang dãy (xn) có giới hạn. Tìm giới hạn đó.
Bài giai.
Tà giả thiet suy ra xn > 0, ∀n ∈ N∗
.r
Xét hàm so f(x) =
1
2x +
125
trên khoảng (0; +∞).
Khi đó xn+1 = f(xn), ∀n = 1, 2, . . . và hàm so f(x) xác định, liên tục trên khoảng
(0; +∞).
Ta có fJ(x) =
2
−
250
, suy ra |fJ(x)| <
2
< 1, ∀x ≥ 5.
Theo định lý Lagrange ta có
|xn — 5| = |f(xn−1) − f(5)| = |fJ(xn )|.|xn−1
2
— 5| ≤
3
.|xn−1 — 5|, ∀n > 1.
Do đó |xn — | ≤
2 n
| — 5|, ∀n > 1.
Do lim
n→+∞
2 n
= 0 nên lim
n→+∞
xn = 5.
Bài toán 3.11. Cho dãy (xn) xác định bởi:
x1 = a (a > 0)
xn+1 = ln(1 + 2e−xn
) (n = 1, 2, . . .)
Cháng minh rang dãy (xn) có giới hạn. Tìm giới hạn đó.
Bài giai.
Tà giả thiet suy ra xn > 0, ∀n ∈ N∗
.
Xét hàm so f(x) = ln(1 + 2e−x) trên khoảng (0; +∞).
(3.11)
Khi đó xn+1 = f(xn), ∀n = 1, 2, . . . và hàm so f(x) xác định, liên tục trên khoảng
(0; +∞). −x
Ta có fJ(x) =
−2e
1 + 2e−x =
−2
ex + 2
suy ra |fJ(xn
2
)| =
ex + 2
2
<
3
< 1, ∀x > 0.
Đ t g(x) = f(x) − x. Khi đó gJ(x) = fJ(x) − 1 < 0, ∀x > 0 suy ra g(x) nghịch bien
trên khoảng (0; +∞) mà g(ln 2) = ln(1 + 2e− ln 2) − ln 2 = 0 nên x = ln 2 là nghi m
duy nhat của phương trình f(x) = x trên khoảng (0; +∞).
x1
40
Viết đề tài giá sinh viên – ZALO:0973.287.149-TEAMLUANVAN.COM
4 ∗
2 2
= .fJ(c). . |xn − 2| <
3 ln 3
. |xn − 2|
Ta cháng minh L = ln 2 là giới hạn của dãy (xn).
Theo định lý Lagrange ton tại c nam giǎa xn và ln 2 sao cho
|xn+1 − ln 2| = |f(xn) − f(ln 2)| = |fJ(c)|.|xn − ln 2|
2
<
3
.|xn − ln 2|
<
3
.|xn−1 − ln 2|
2 n
Do lim 2 n
= 0 nên lim
< · · · <
3
|x1 − ln 2|.
xn = 0.
n→+∞ 3 n→+∞
Bài toán 3.12. Cho dãy (xn) xác định bởi:
x1 = a (a > 0)
(3.12)
4
x = log (x3
1
, ∀n ∈ N∗
.
Cháng minh rang dãy (xn) có giới hạn. Tìm giới hạn đó.
Bài giai.
Tà giả thiet suy ra xn >
3
, ∀n ∈ N .
3 1 4
Xét hàm so f(x) = log3(x + 1)3 +
3
trên khoảng (0; +∞).
Khi đó xn+1 = f(xn), ∀n = 1, 2, . . . và hàm so f(x) xác định, liên tục trên khoảng
(0; +∞).
2
Ta có fJ(x) =
x
.
(1 + x3) ln 3
3 x3 x3
r
3 x6 3 2
Với moi x > 0 ta có x
√
3
4
+ 1 =
2
+
2
+ 1 ≥ 3
4
= √
3
4
x .
Do đó |fJ(x)| ≤
3 ln 2
< 1, ∀x > 0.
Đ t g(x) = f(x) − x. Khi đó gJ(x) = fJ(x) − 1 < 0, ∀x > 0 suy ra g(x) nghịch bien
trên khoảng (0; +∞) mà g(2) = 0 nên x = 2 là nghi m duy nhat của phương trình
f(x) = x trên khoảng (0; +∞).
Ta cháng minh L = 2 là giới hạn của dãy (xn).
Theo định lý Lagrange ton tại c nam giǎa xn và 2 sao cho
|xn+1 − 2| = |f(xn) − f(2)|
. .
√
3
4
√
3
4
<
3 ln3
2
. |xn−1 − 2| < · · · <
√
3
4
3 ln 3
n
. |x1 − 2| .
n + 1)3 +
3
3
n+1
41
Viết đề tài giá sinh viên – ZALO:0973.287.149-TEAMLUANVAN.COM
2
2
2 2
2
4
. .
. .
−
fJ(x) = 1 + cos x − sin x = 1 −
√
2 sin x −
π
< 0 với x ∈
hπ
; π
i
π
0
4
3π
2
Do lim
n→+∞
√
3
4 n
3 ln 3
= 0 nên lim
n→+∞
xn = 2.
Bài toán 3.13. Cho dãy (xn) xác định bởi:
x1 =
2 (3.13)
xn+1 = xn + sin xn + cos xn, ∀n ∈ N∗
.
Cháng minh rang dãy (xn) có giới hạn. Tìm giới hạn đó.
Bài giai.
Ta cháng minh xn ∈
h
; π
i
, ∀n ∈ N∗
.
π
Xét hàm so f(x) = x + sin x + cos x trên
hπ
; π
i
.
Ta có f(x) xác định, liên tục trên khoảng
π
; π .
4 2
nên hàm so f(x) nghịch bien trên khoảng
π
; π .
2
Với x ≥ ta có x = f(x ) ≤ f = + 1 < π.
π π π
Với xn ≤ π ta có xn+1 = f(xn
π
) ≥ f(π) = π − 1 ≥
2
.
V y với moi xn ∈
hπ
; π
i
thì xn+1 ∈
hπ
; π
i
.
Trên
hπ
; π
i
, phương trình f(x) = x + sin x + cos x = 0 có nghi m x =
3π
,
|fJ(x)| = |1 + cos x − sin x| = |1 −
√
2 sin x −
π
| ≤
√
2 − 1.
Theo định lý Lagrange ton tại c nam giǎa xn và x0 sao cho
|xn+1 − x0| = |f(xn) − f(x0)| = fJ(c) .|f(xn) − f(x0)|
= fJ(c) .|xn − x0| ≤ (
√
2 − 1).|xn − x0|.
Suy ra |xn − x0| ≤ (
√
2 − 1)n−1|x1 − x0|, ∀n > 1.
Do lim
n→+∞
(
√
2 1)n−1 = 0 nên lim xn = .
n→+∞ 4
Bài toán 3.14. Xét phương trình (trong đó n là so nguyên dương)
1 1 1 1 1
x − 1
+
4x − 1
+ · · · +
k2x − 1
+ · · · +
n2x − 1
=
2
. (3.14)
a) Cháng minh rang với moi so nguyên dương n, phương trình nêu trên có nghi m
duy nhat lớn hơn 1, kí hi u nghi m đó là xn.
b) Cháng minh rang dãy so (xn) có giới hạn bang 4 khi n → +∞.
2
2
n
n+1
2
n
42
Viết đề tài giá sinh viên – ZALO:0973.287.149-TEAMLUANVAN.COM
∞
x
− ∀ ∈
n
−
2 2 2 − 1 2 + 1 2n − 1 2n + 1
2 2 2n + 1 2(2n + 1)
n
Bài giai.
Viet lại phương trình của bài toán dưới dạng:
1 1 1 1 1
−
2
+
x − 1
+
4x − 1
+ · · · +
k2x − 1
+ · · · +
n2x − 1
= 0.
Với moi n = 1, 2, 3, . . . ta xét hàm so:
1 1 1 1 1
fn(x) = −
2
+
x − 1
+
4x − 1
+ · · · +
k2x − 1
+ · · · +
n2x − 1
.
a) De thay, với moi n ∈ N∗
, hàm so fn(x) liên tục và nghịch bien trên khoảng
(1; +∞). Hơn nǎa ta có
lim
x→1+
fn(x) = + , lim
→+∞
1
fn(x) = −
2
< 0.
Tà đó suy ra với moi n = 1, 2, . . . phương trình fn(x) = 0 có nghi m duy nhat
xn ∈ (1; +∞).
b) Với moi n ∈ N∗
, ta có:
1 1 1 1 1
fn(4) = −
2
+
22 − 1
+
42 − 1
+ · · · +
(2k)2 − 1
+ · · · +
(2n)2 − 1
1 1 1 1
= −
2
+
(2 − 1)(2 + 1)
+
(4 − 1)(4 + 1)
+ · · · +
(2n − 1)(2n + 1)
= −
1
+
1
h 1
−
1
+ · · · +
1
−
1
i
= −
1
+
1
1 −
1
=
−1
< 0 = f (x).
Tà đó, do fn(x) = 0 nghịch bien trên (1; +∞) nên suy ra xn < 4, ∀n ∈ N∗
.
M t khác, do với moi n = 1, 2, . . . , hàm fn(x) có đạo hàm trên đoạn [xn; 4] nên theo
định lý Lagrange, suy ra với moi n ∈ N∗
ton tại tn ∈ (xn; 4) sao cho
2
fn(4) − fn(xn)
= f
′
(t) = −
1
−
4
− · · · −
n
< −
1 (3.15)
4 − xn (t − 1)2 (4t − 1)2 (n2t − 1)2 9
(do 1 < t < 4). Mà fn(xn) = 0 nên tà (3.15) ta có
−1
< −
1
⇔ 4 − x 9
< ⇔ x 9
> 4 − (3.16)
2(2n + 1)(4 − xn) 9 n 2(2n + 1) n 2(2n + 1)
Tà (3.16) có 4
9
2(2n + 1)
< xn < 4, n N∗
mà lim
→+∞
4
9
= 4 =
2(2n + 1)
lim
n→+∞
4, nên sả dụng nguyên lý kep ta được lim
n→+∞
xn = 4.
n
43
Viết đề tài giá sinh viên – ZALO:0973.287.149-TEAMLUANVAN.COM
Σ
n n
x
=
n
+
1
n
un+1 = u2 − 5un + 9, ∀n ∈ N∗
.
=
— 2 — 3
−
u
=
u — 3
−
u — 3
, ∀n ∈ N∗
.
n→+∞
n
u1 − 3 un+1 − 3 u1 − 3 7
un.
3.4 Xác định gi i hạn của dãy tong
Bài toán 3.15. Cho dãy so (un) xác định bởi:
u1 = 10
(3.17)
Đ t vn =
n
k=1
1
uk − 2
, ∀n ∈ N∗ . Tìm lim
n→+∞
vn.
Bài giai.
Tà (3.17), ta có
un+1 − 3 = (un − 2)(un − 3)
1
⇔
un+1
=
— 3 un
1 1
— 3
−
un − 2
⇔
1
=
1
−
1
, ∀n ∈ N∗
.
Do đó
un − 2 un − 3 un+1 − 3
Σ 1 Σ 1 1 1 1
M t khác un+1 − un = (un − 3)2 > 0, ∀n ∈ N∗
nên dãy (un) là dãy tăng. Neu dãy
(un) bị ch n trên thì dãy (un) có giới hạn là a. Tà (3.17), chuyen qua giới hạn ta
có a = a2 − 5a + 9 ⇔ a2 − 6a + 9 = 0 ⇔ a = 3. Đieu này vô lý vì u1 = 10 và dãy (un)
là dãy tăng. Do đó lim
n→+∞
un = +∞.
V y,
lim v = lim
1
−
1
=
1
=
1
.
Bài toán 3.16. Cho dãy so (xn) xác định bởi:
x1 = 3
Σ
n 1
√
xn(xn + 3)(x2 + 3xn + 2) + 1, ∀n ∈ N∗
.
(3.18)
Bài giai.
Tà giả thiet suy ra xn > 0, ∀n ∈ N∗
.
i=1 xi + 2
n+1
1
— 3
k+1
k
u
k=1
k
u
k=1
n
vn =
Đ t un = . Tìm lim
n→+∞
44
Viết đề tài giá sinh viên – ZALO:0973.287.149-TEAMLUANVAN.COM
√
x = x (x + 3)(x + 3
x
+2) + 1
n+1 n n
n
n
n
−
n n
n
∀ ∈
N
∗
n
+
1
n
n n
n
n
n
n n
n→+∞
n
n→+∞ x1 + 1 xn+1 + 1 4
u1 = 2017
Σ
1
Ta có
2
n
=
q
(x2 + 3xn + 1)2 = x2 + 3xn + 1.
Xét xn+1 −xn = x2 +3xn +1−xn = (xn + 1)2 > 0, ∀n ∈ N∗
nên dãy (xn) là dãy tăng.
Neu dãy (xn) bị ch n trên thì dãy (xn) có giới hạn hǎu hạn là a. Tà xn+1 = x2 +3xn+1
suy ra a = a2 + 3a + 1 suy ra a = 1 (vô lí). Do đó lim
n→+∞
xn = +∞.
Tà xn+1 = x2 + 3xn + 1 suy ra xn+1 + 1 = x2 + 3xn + 2 = (xn + 1)(xn + 2).
n
1
⇒
xn+1
=
+ 1 (xn
1
+ 1)(xn
n
=
+ 2) xn
1 1
+ 1
−
xn + 2
⇒
xn
1
=
+ 2 xn
1
+ 1
−
x
1
n+1 + 1
Σ 1 Σ 1 1 1 1
Suy ra
⇒ un = i=1
=
xi + 2
i=1 xi + 1
−
xi + 1 =
x + 1
−
x .
n+1 + 1
lim u = lim
1
−
1
=
1
.
Bài toán 3.17. Cho dãy so (un) xác định bởi:
u4 + 20162
u = , n .
u3 − un + 4032
(3.19)
n 1
Đ t vn = 3
, ∀n ∈ N∗
. Tìm lim
n vn.
k=1 uk + 2016
Bài giai.
Tà (3.19), ta có
u4 + 20162
→+∞
(un − 2016)(u3 + 2016)
un+1 − 2016 = n − 2016 = n .
u3 − un + 4032 un(u2 − 1) + 2016
Tà đó bang quy nạp ta cháng minh được un > 2016, ∀n ∈ N∗
.
Ta có
Suy ra
un+1 − 2016 =
(un − 2016)(u3 + 2016)
(u3 + 2016) − (un − 2016)
1 1 1
un+1
= −
— 2016 un − 2016 u3 + 2016
.
45
Viết đề tài giá sinh viên – ZALO:0973.287.149-TEAMLUANVAN.COM
n
n n
n
n n
−
2016
1
hay
Do đó
1
u3 + 2016
=
u
1
— 2016
−
u n+1
1
.
— 2016
Σ 1 Σ 1 1
vn =
k=1 u3 + 2016
=
1
k=1 uk − 2016
−
u
1
k+1 — 2016
1
M t khác
=
u − 2016
−
u
n+1
— 2016
= 1 −
u
n+1
.
— 2016
un+1 − un =
u4 − 4032un + 20162
u3 − un + 4032
(un − 2016)2
u3 − un + 4032
> 0, ∀n ∈ N∗
nên dãy (un) là dãy tăng. Neu dãy (un) bị ch n trên thì dãy (un) có giới hạn là a.
a4 + 20162
Tà (3.19), chuyen qua giới hạn ta có a =
a3 − a + 4032
⇔ a = 2016. Đieu này vô
lý vì u1 = 2017 và dãy (un) là dãy tăng. Do đó lim
n→+∞ un = +∞.
V y, lim vn
n→+∞
= lim
n→+∞
1
1
= 1.
un+1 − 2016
Bài toán 3.18. Cho dãy so (un) xác định bởi:
u1 = 1
u2016 (3.20)
un+1 = n
2016
+ un, ∀n ∈ N∗
.
Đ t vn = 2015
1
+
2015
2
+
2015
3 2015
+ · · · + n . Tìm lim
vn.
u2 u3 u4 un+1 n→+∞
Bài giai.
Tà giả thiet suy ra un > 0, ∀n ∈ N∗
.
Ta có
u2016
un+1 − un = n > 0, ∀n ∈ N∗
nên dãy (un) là dãy tăng. Neu dãy (un) bị ch n trên thì dãy (un) có giới hạn là a.
a2016
Tà (3.20), chuyen qua giới hạn ta có a = + a ⇔ a = 0. Đieu này vô lý vì
2016
u1 = 1 và dãy (un) là dãy tăng. Do đó lim
n un = +∞.
Tà un+1 — un
u2016
= n
2016
> 0, ta có
1
→+∞
1 u2016
n
un
−
u
n+1
=
2016un
.
un+1
u
u
u
u
k
n
=
46
Viết đề tài giá sinh viên – ZALO:0973.287.149-TEAMLUANVAN.COM
n
−
u1 un+1 un+1
3
Suy ra
Khi đó
2016
1
un
1
un+1
2015
= n .
un+1
vn =
Σ
k=1
2015
k
uk+1
= 2016
1
u1
1 1
+
u2 u2
1
u
+ · · · +
1
un+1
= 2016
1
−
1
= 2016 1 −
1
.
V y, lim vn
n→+∞
= lim
n→+∞
2016 1
1
= 2016.
un+1
u
u
−
− −
47
Viết đề tài giá sinh viên – ZALO:0973.287.149-TEAMLUANVAN.COM
n
Chương 4
Các dạng toán khác liên quan đen
dãy so
4.1 M t so dạng toán liên quan đen tính chat của
dãy so
Trong phan này, ta xét m®t so dạng toán cháng minh tính chat của dãy so như
dãy so nguyên, dãy so tuan hoàn.
Bài toán 4.1. Cho dãy so (un) xác định bởi:
Cháng minh rang:
u0 = 1, u1 = 2
un+2 = 4un+1 − un, ∀n ∈ N.
(4.1)
a) un − 1 là m®t so chính phương với moi n lẻ.
b) un − 1
6 là m®t so chính phương với moi n chȁn.
Nh n xét 4.1. Đây là dạng toán ve phương trình sai phân tuyen tính cap hai,
song ở đây ta sě xét phương pháp cháng minh không sả dụng đen tính chat của
sai phân. Đe cháng minh dãy so (bn) thỏa mãn bn là so chính phương với moi so
nguyên dương n ta thường sả dụng m®t so hướng sau:
• Hướng 1: Ta sě chỉ ra sự ton tại dãy so nguyên (cn) thỏa mãn bn = c2 , ∀n ∈ N∗
.
Dãy so (cn) thường dự đoán bang cách tính m®t so giá trị đau c1, c2, . . . và
tìm ra quy lu t của dãy (cn).
48
Viết đề tài giá sinh viên – ZALO:0973.287.149-TEAMLUANVAN.COM
n
n
⇔
n
• Hướng 2: Ta cháng minh bnbn+2 là m®t so chính phương với moi so nguyên
dương n, sau đó cháng minh bang quy nạp.
• Hướng 3: Dựa vào công thác truy hoi ta tính được bn = c2 .
Bài giai.
a) Cách 1.
Ta dự đoán dãy so (cn) sao cho u2n+1 − 1 = c2 , ta có
u1 = 2, u3 = 26, u5 = 362, u7 = 5042
suy ra
c0 = 1, c1 = 5, c2 = 19, c3 = 71.
Khi đó ta thả thiet l p quan h truy hoi của dãy (cn) theo dãy tuyen tính cap
hai.
Giả sả cn+2 = acn+1 + bcn và tà c0 = 1, c1 = 5, c2 = 19, c3 = 71, ta được
5a + b = 19
19a + 5b = 71
a = 4
b = −1.
Do đó ta dự đoán dãy so (cn ) là
c0 = 1, c1 = 5
cn+2 = 4cn+1 − cn, n = 0, 1, . . .
Ta sě cháng minh bang quy nạp
u2n+1 − 1 = c2 , n = 0, 1, . . . (4.2)
Th t v y (4.2) đúng với n = 0.
Giả sả (4.2) đúng đen n, ta sě cháng minh (4.2) đúng đen n + 1. Ta có
u2n+3 − 1 = 4u2n+2 − u2n+1 − 1 = 4(4u2n+1 − u2n) − u2n+1 − 1
= 16u2n+1 − 4u2n − u2n+1 − 1 = 15u2n+1 − (u2n+1 + u2n−1) − 1
= 14u2n+1 − u2n−1 − 1 = 14(c2 + 1) − c2 − 1 − 1
hay
n n−1
u2n+3 − 1 = 12c2 − c2 − 12. (4.3)
n n−1
49
Viết đề tài giá sinh viên – ZALO:0973.287.149-TEAMLUANVAN.COM
n
2
n+1
n+1
Theo h thác cơ bản của dãy tuyen tính cap hai ta được
cn+1cn−1 − c2
= −6
⇒ (4cn − cn−1)cn−1 − cn = −6
2 2
Ta có
⇒ cn + cn−1 − 4cncn−1 − 6 = 0.
c2 = (4cn − cn−1)2 = 16c2 − 8cncn−1 + c2 = 16c2 − 2 c2 + c2 — 6 + c2
hay
n+1 n n−1 n n n−1 n−1
c2 = 14c2 − c2 − 12. (4.4)
n+1 n n−1
Tà (4.3) và (4.4) suy ra u2n+3 − 1 = c2 .
n+1
Do đó ta cháng minh được (4.2) đúng đen n + 1 suy ra (4.2) đúng.
V y un − 1 là m®t so chính phương với moi n lẻ.
Cách 2.
Ta có un+2un − u2 = 3, ∀n ≥ 0. Tà h thác này ta được
(un+2 −1)(un −1) = un+2un −un+2 −un +1 = u2 +3−4un+1 +1 = (un+1 − 2)2 . (4.5)
Tà h thác (4.5), bang phương pháp quy nạp suy ra un − 1 là so chính phương với
moi so nguyên dương n lẻ.
b) Ta cháng minh theo hướng 2 như sau:
un+2 − 1
.
6
un − 1
6
=
un+2un − un+2 − un + 1
=
36
2
n+1 — 4un+1 + 4
36
un+1 − 2 2
Tà đȁng thác này, bang phương pháp quy nạp suy ra
un − 1
6
là so chính phương.
Bài toán 4.2 (TST Vi t Nam 2012). Cho dãy so (un) được xác định như sau:
u1 = 1, u2 = 2011
un+2 = 4022un+1 − un, n = 1, 2, . . .
(4.6)
Cháng minh rang
u2012 + 1
2012
là m®t so chính phương.
Bài giai.
Ta sě giải bài toán tőng quát sau: Cho p là m®t so nguyên dương lẻ và dãy so (un)
được xác định như sau:
u1 = 1, u2 = p
un+2 = 2pun+1 − un, n = 1, 2, . . .
6
u
= .
50
Viết đề tài giá sinh viên – ZALO:0973.287.149-TEAMLUANVAN.COM
n+1 2
2
n+1
2
n+1
n+1 2
p + 1 p + 1 p + 1
n+1 n+1 n n
p + 1 p + 1
2
2 2
Cháng minh rang
u2n + 1
p + 1
là so chính phương với moi so nguyên dương n.
Cách 1. Ta sě cháng minh theo hướng 1. Ta tính m®t vài giá trị đau tiên
u2 + 1
= 1,
u4 + 1
= (2p − 1)2,
u6 + 1
= (4p2 − 2p + 1)2, . . .
Ta dự đoán được
u2n + 1
= x2 , trong đó (x ) là dãy so được xác định như sau:
p + 1 n n
x1 = 1, x2 = 2p − 1, · · · , xn+2 = 2pxn+1 − xn, n = 1, 2, . . .
Ta sě cháng minh ket quả trên bang phương pháp quy nạp. Ta có
xn+2xn − x2 = (−1)n−1(x3x1 − x2)2 = 2p − 2
2
⇒ xn+2xn = xn+1 + 2p − 2
⇒ (2pxn+1 − xn)xn = xn+1 + 2p − 2
2 2
Suy ra
⇒ xn+1 + xn + 2p − 2 = 2pxnxn+1.
2
n+2 = (2pxn+1 − xn)2
=4p2x2
— 4pxn+1xn + xn
=4p2x2 − 2 x2 + x2 + 2p − 2 + x2
= 4p2 − 2 x2 − xn − 4p + 4.
Do đó
4p2 − 2
u2n+2 + 1
−
u2n + 1
− 4p + 4
4p2 − 2 u2n+2 − u2n + 1
p + 1
=
u2n+4 + 1
.
p + 1
Suy ra
u2n+4 + 1
= x2 .
Cách 2.
p + 1 n+2
Ta sě cháng minh theo hướng 2. Trước het ta có h thác cơ bản sau
un+2un − u2 = (−1)n−1 u3u1 − u2 = 2p2 − 1 − p2 = p2 − 1
⇒ un+2un = un+1 + p − 1.
x
=
51
Viết đề tài giá sinh viên – ZALO:0973.287.149-TEAMLUANVAN.COM
u
u = 7u − u , n = 2,3, . ..n+1
n n−1
n
n+1
n+1
p + 1
Ta có
un+2 + 1 un + 1
=
un+2un + un+2 + un + 1
(p + 1)2
2
= n+1
u2
+ p2 − 1 + 2pun+1 + 1
(p + 1)2
+ p
2
Tà đȁng thác này, bang phương pháp quy nạp ta được
u2n + 1
p + 1
là so chính phương
với moi so nguyên dương n.
Bài toán 4.3 (China South East Mathematical 2011). Cho dãy so (un) được xác
định như sau
u1 = u2 = 1
Cháng minh rang với moi so nguyên dương n ta có un + un+1 + 2 là m®t so chính
phương.
Bài giai.
Tính m®t vài giá trị đau tiên ta được:
u1 + u2 + 2 = 22, u2 + u3 + 2 = 32, u3 + u4 + 2 = 72, u4 + u5 + 2 = 182.
Tà đó ta dự đoán un + un+1 + 2 = x2 , trong đó dãy so (xn) được xác định như sau:
x1 = 2, x2 = 3, xn+1 = 3xn − xn−1, n = 2, 3, . . . Ta sě cháng minh dự đoán này bang
phương pháp quy nạp. Ta có
xn+1xn−1 − x2 = 5 ⇒ (3xn − xn−1)xn−1 − x2 = 5
n n
2 2
⇒ 3xnxn−1 = xn−1 + xn + 5
= xn−1 + xn + xn + xn+1 + 9 = xn+1 + 2xn + xn−1 + 9.
Theo công thác truy hoi của dãy (xn) ta được:
x2 = (3xn − xn−1)2 = 9x2 + x2 — 6xnxn−1
n+1 n n−1
Do đó x2
= 9(un + un+1 + 2) + un−1 + un + 2 − 2(un+1 + 2un + un−1 + 9)
= 7un+1 − un + 7un − un−1 + 2 = un+2 + un+1 + 2.
= un+1 + un+2 + 2 hay bài toán được cháng minh.
p + 1
p + 1
= .
52
Viết đề tài giá sinh viên – ZALO:0973.287.149-TEAMLUANVAN.COM
u = 3u− u , n = 1, 2, .. .n+2
n+1 n
2
√
∗
8
u + 1, n ∈ N .
n
⇔
Bài toán 4.4 (Balkan MO 2002). Cho dãy so (un) xác định như sau:
u1 = 20, u2 = 30
Tìm tat cả các so nguyên dương n sao cho 1 + 5unun+1 là m®t so chính phương.
Bài giai.
De thay dãy (un) là dãy so tăng, suy ra với n ≥ 4 ta có
un + un+1 ≥ u4 + u5 > u3 + u4 = 250. (4.7)
+) n ∈ {1, 2} không thỏa mãn.
+ n = 3 thì 1 + 5u3u4 = 2512 suy ra n = 3 thỏa mãn.
+) n ≥ 4, theo tính chat cơ bản của dãy tuyen tính cap hai ta có:
un+2un = u2 + (−1)n−1(u3u1 − u2) = u2
+ 500
n+1
2
2 n+1
⇒ (3un+1 − un)un = un+1 + 500
2 2
⇒ 3un+1un = un+1 + un + 500
⇒ 5un+1un + 1 = (un+1 + un) + 501.
Giả sả 1 + 5unun+1 là so chính phương, 1 + 5unun+1 = a2, a ∈ N∗
. Khi đó ta có
(un+1 + un)2 + 501 = a2 ⇔ (a − un+1 − un)(a + un+1 + un) = 501 = 1.501 = 3.167.
Ta xét các trường hợp sau:
Trường hợp 1:
a + un + un+1 = 501
a − un − un+1 = 1
Trường hợp 2:
a = 251
un + un+1 = 250
(mâu thuan với (4.7)).
⇔
a + un + un+1 = 167 a = 85
(mâu thuan với (4.7)).
a − un − un+1 = 3 un + un+1 = 82
Do đó với n ≥ 4 thì 1 + 5unun+1 không phải là so chính phương.
V y n = 3 là so nguyên dương thỏa mãn yêu cau bài toán.
Bài toán 4.5. Cho dãy so (un) xác định bởi:
u1 = 1
un+1 = 3un + 2
53
Viết đề tài giá sinh viên – ZALO:0973.287.149-TEAMLUANVAN.COM
2
u n
2
n n+1 n
Cháng minh moi so hạng của dãy so đeu là so nguyên.
Bài giai.
Tà giả thiet ta có
un+1 − 3un =
√
8u2 + 1 ⇔ u2 — 6un+1un + u = 1.
Thay n bang n + 1 ta được h :
2
n+1 — 6un+1un + u2 = 1
u2 − 6unun−1 + u2
= 1.
n
Trà tàng ve của hai phương trình ta có
n−1
2
n+1 — un−1 — 6un+1un + 6unun−1 = 0
⇔ (un+1 − un−1)(un+1 + un−1 − 6un) = 0.
Suy ra un+1 = un−1 ho c un+1 = 6un − un−1.
Trường hợp un+1 = un−1 không the xảy ra.
Trường hợp un+1 = 6un − un−1, xác định dãy (un) như sau:
u1 = 1, u2 = 6, un+1 = 6un − un−1, ∀n ≥ 2.
Do u1, u2 ∈ Z nên tà công thác xác định dãy ta có un ∈ Z, ∀n ∈ N∗
.
V y moi so hạng của dãy so đeu là so nguyên.
Bài toán 4.6. Cho dãy so (un) thỏa mãn un+2
=
unun+1
, n = 1, 2, . . .
2un − un+1
Tìm đieu ki n can và đủ đoi với u1, u2 đe dãy so có vô so so hạng nguyên.
Bài giai.
Trước het ta cháng minh uk 0 với ∀k = 1, 2, . . .
Giả sả ∃k ∈ N∗
đe uk = 0 thì uk+2 =
ukuk+1
= 0.
2uk − uk+1
Tương tự, suy ra uk+3 = 0, suy ra 2uk+2 − uk+3 = 0, do đó uk+4 không ton tại.
V y uk /= 0, ∀k ∈ N∗
.
Đ t vn
1
= , theo cách thiet l p dãy so, ta có v
un
n+2
1
=
un+2
, suy ra
vn+2 =
2un − un+1
unun+1
= 2vn+1 — vn, n = 1, 2, . . .
⇔ vn+2 + vn = 2vn+1, ∀n ∈ N∗
.
u
54
Viết đề tài giá sinh viên – ZALO:0973.287.149-TEAMLUANVAN.COM
| |
n
n
n
n
n
n+1
n+1
Khi đó dãy (vn) l p thành cap so c®ng với công sai d.
Vì vn
1
= , nên neu un
un
∈ Z thì vn ∈ [−1; 1]. Do đó neu ton tại dãy con vô hạn của
(un) là các so nguyên khác 0 thì trong dãy (vn) có m®t dãy con mà moi phan tả
của nó đeu thu®c [−1; 1].
Đieu này chỉ xảy ra khi d = 0, vì neu d /= 0 thì ta thay
lim
n→+∞
vn = lim
→+∞
|v1 + (n − 1) d| = +∞
nên neu ∃k0 đe ∀k ≥ k0 thì |vk| > 1, đieu này không xảy ra.
V y d = 0, suy ra v1 = v2 = · · · = vn = . . . hay u1 = u2 = · · · = un = . . .
Do đó đieu ki n can và đủ đe dãy (un) có vô so so hạng nguyên là
u1 = u2 = a ∈ Z.
Bài toán 4.7. Cho dãy so (un) được xác định như sau:
u0 = 1, u1 = 13
un+2 = 14un+1 − un, n ≥ 0.
(4.8)
Cháng minh rang với moi so tự nhiên n, ton tại các so tự nhiên k, l sao cho
un = k2 + (k + 1)2, u2 = (l + 1)3 − l3.
Bài giai.
Ta có un = k2 + (k + 1)2 = 2k2 + 2k + 1 ⇔ 2un − 1 = (2k + 1)2 và u2 = (l + 1)3 − l3 =
3l2 + 3l + 1 ⇔ 12u2 − 3 = (6l + 3)2.
Như v y bài toán quy ve cháng minh 2un − 1, 12u2 − 3 là các so chính phương.
Trước het ta có h thác cơ bản sau un+2un − u2 = 12, ∀n ≥ 0.
Th t v y, ta có
un+2 + un
un+1
=
un+1 + un−1
un
⇒un(un+2 + un) = un+1(un+1 + un−1)
2 2 2
Xét
⇒un+2un − un+1 = un+1un−1 − un = · · · = u0u2 − u1 = 12.
(2un+2 − 1)(2un − 1) = 4un+2un − 2(un+2 + un) + 1
= 4(u2
+ 12) − 28un+1 + 1 = (2un+1 + 7)2.
55
Viết đề tài giá sinh viên – ZALO:0973.287.149-TEAMLUANVAN.COM
2 2 2
n
n
n+2 n n
n
n
Lại có
(12u2
— 3)(12u − 3) = 144(un+2un) − 36(u + u2 ) + 9
= 144(un+2un)2 − 36(un+2 + un)2 + 72un+2un + 9
= 144(un+2un)2 − 36(14un+1)2 + 72un+2un + 9
= 144(un+2un)2 − 36.142(un+2un − 12)2 + 72un+2un + 9
= 144(un+2un)2 − 36.194un+2un + 2912 = (12un+2un − 291)2.
V y ta cháng minh được 2un − 1, 12u2 − 3 là các so chính phương.
Sau đây ta xét m®t so bài toán ve dãy so tuan hoàn c®ng tính và tuan hoàn
nhân tính.
Bài toán 4.8. Xác định dãy (xn) sao cho xn+3 = xn + 1, n = 0, 1, 2, . . .
Bài giai.
Đ t xn
n
= + yn
3
. Khi đó ta có
yn+3 +
n + 3
=
3
n
3
+ yn + 1,
hay yn+3 = yn, n = 0, 1, 2, . . .
V y nên
Do đó
y0 = y3 = y6 = . . .
y1 = y4 = y7 = . . . ⇔ yn =
y2 = y5 = y8 = . . .
a tùy ý với n = 3k, k ∈ N
b tùy ý với n = 3k + 1, k ∈ N
c tùy ý với n = 3k + 2, k ∈ N.
xn =
a +
3
tùy ý với n = 3k, k ∈ N
b +
3
tùy ý với n = 3k + 1, k ∈ N
n
c +
3
tùy ý với n = 3k + 2, k ∈ N.
Bài toán 4.9. Xác định dãy (xn) sao cho xn+3 = 2xn, n = 0, 1, 2, . . .
Bài giai.
Đ t xn = 23 yn. Khi đó ta có
n+3 n
2 3 yn+3 = 2(23 yn),
n+2
56
Viết đề tài giá sinh viên – ZALO:0973.287.149-TEAMLUANVAN.COM
hay yn+3 = yn, n = 0, 1, 2, . . .
V y nên
y0 = y3 = y6 = . . .
y1 = y4 = y7 = . . . ⇔ yn =
y2 = y5 = y8 = . . .
n
a tùy ý với n = 3k, k ∈ N
b tùy ý với n = 3k + 1, k ∈ N
c tùy ý với n = 3k + 2, k ∈ N.
Do đó xn = 2 3 yn, trong đó
yn =
a tùy ý với n = 3k, k ∈ N
b tùy ý với n = 3k + 1, k ∈ N
c tùy ý với n = 3k + 2, k ∈ N.
Bài toán 4.10. Xác định dãy (un) thỏa mãn đieu ki n
u2n+1 = 3un, ∀n ∈ N. (4.9)
Bài giai.
Đ t n + 1 = m, m = 1, 2, . . . Khi đó ta có the viet (4.9) dưới dạng
u2m−1 = 3um−1, ∀m ∈ N∗
hay
với
v2m = 3vm, ∀m ∈ N∗
(4.10)
vm = um−1, ∀m ∈ N∗
.
Tà (4.10) ta có v0 = 0. Đ t vm = mlog2 3
ym, m ∈ N∗
. Khi đó (4.10) có dạng
y2m = ym, m ∈ N∗
.
V y (ym) là dãy tuan hoàn nhân tính chu kỳ 2. Khi đó, ta có
yn
Tà đó suy ra
với
yn
=
=
tùy ý với n lẻ
y2k+1 với n có dạng 2m(2k + 1), m ∈ N∗
, k ∈ N.
um = vm+1 = mlog2 3
ym+1,
tùy ý với n lẻ
y2k+1 với n có dạng 2m(2k + 1), m ∈ N∗
, k ∈ N.
57
Viết đề tài giá sinh viên – ZALO:0973.287.149-TEAMLUANVAN.COM
Bài toán 4.11. Xác định dãy (un) thỏa mãn đieu ki n
u2n+1 = −3un + 4, ∀n ∈ N. (4.11)
Bài giai.
Đ t n + 1 = m, m = 1, 2, . . . Khi đó ta có the viet (4.11) dưới dạng
u2m−1 = −3um−1 + 4, ∀m ∈ N∗
hay
với
v2m = −3vm + 4, ∀m ∈ N∗
(4.12)
vm = um−1, ∀m ∈ N∗
.
Đ t vm = 1 + xm, m ∈ N∗
. Khi đó (4.12) có dạng
x2m = −3xm, m ∈ N∗
. (4.13)
Đ t xm = mlog2 3
ym, m ∈ N∗
. Khi đó (4.13) có dạng
y2m = −ym, m ∈ N∗
.
V y (ym) là dãy phản tuan hoàn nhân tính chu kỳ 2. Khi đó, ta có
yn =
Tà đó suy ra
tùy ý với n lẻ ,
−y2k+1 với n có dạng 22m+1(2k + 1), m, k ∈ N,
y2k+1 với n có dạng 22m(2k + 1), m ∈ N∗
, k ∈ N.
um = vm+1 = 1 + (m + 1)log2 3
ym+1,
với
yn =
tùy ý với n lẻ ,
−y2k+1 với n có dạng 22m+1(2k + 1), m, k ∈ N,
y2k+1 với n có dạng 22m(2k + 1), m ∈ N∗
, k ∈ N.
58
Viết đề tài giá sinh viên – ZALO:0973.287.149-TEAMLUANVAN.COM
1
2
2
3
— −
un+1 = un(u2
n−1 — 2) −
2
4.2 M t so dạng toán khác
Bài toán 4.12. Cho dãy (un) xác định bởi:
u0 = 2
5
u1 =
2
5
(4.14)
Cháng minh rang [un] = 2
22 − (−1)n
3 , ∀n ∈ N∗
.
( đây, ta kí hi u [un] là so nguyên lớn nhat không vượt quá un).
Bài giai.
Ta có
1
u2 = 2 +
2
.
u3 = u2(u2 − 1) − 2 +
1
2 = 2 +
1
h
2 +
1
— 2
i
= 23
1
+
23 .
u4 = 23 +
h
2 + − 2
i
= 25
1 1 1
+
25 .
Ta cháng minh bang quy nạp un = 2an
+ 2
Th t v y, tà (4.14), ta có
−an
với an =
2n − (−1n)
3
, ∀n ≥ 1.
un+1 = 2an
+ 2−an
h
2an−1
+ 2−an−1
− 2
i
− 2 +
1
= 2an+2an−1
+ 2−an−2an−1
+ 22an−1−an
+ 2an−2an−1
− 2 − 2−1.
De thay
do đó un+1 = 2an+1
+ 2−an+1
.
an + 2an−1 = an+1
2an−1 − an = (−1)n
Vì v y un = 2
2n−(−1)n
3
1
2
2n−(−1)n , ∀n ≥ 1.
Mà 2
2n−(−1)n
3 ∈
+
và 0 <
1
2n−(−1)n
3
< 1 nên [un] = 2
22 ( 1)n
3 , ∀n ≥ 1.
Bài toán 4.13 (Đe thi hoc sinh giỏi Quoc gia 2016). a) Cho dãy (an) xác định
bởi an = ln(2n2 + 1) − ln(n2 + n + 1), với n = 1, 2, . . . Cháng minh rang chỉ có hǎu
hạn so n sao cho {an
1
} <
2
.
2
23
2
+
Z
2
, ∀n ∈ N∗
.
2
2
59
Viết đề tài giá sinh viên – ZALO:0973.287.149-TEAMLUANVAN.COM
{ }
n
— −
n
b) Cho dãy (bn) xác định bởi bn = ln(2n2 + 1) + ln(n2 + n + 1), với n = 1, 2, . . .
Cháng minh rang ton tại vô hạn so n sao cho {bn
1
} <
2016
.
Trong đó {x} là ký hi u phan lẻ của so thực x : {x} = x − [x].
Bài giai.
2n2 + 1
a) De thay 1 ≤
n2 + n + 1
< 2 với moi n = 1, 2, . . . Tà đó suy ra 0 ≤ an < ln 2 < 1
và [an] = 0.
Với ket quả này, ta có {an} = an và
lim
n→+∞
an = lim
→+∞
an = lim
n→+∞
2n2 + 1
ln
n2 + n + 1
= ln 2.
Do đó, ton tại n0 ∈ N∗
đe {an
1
} = ln 2 −
1992
với moi n ≥ n0.
Bây giờ, neu có vô hạn so n đe {an
1
} <
2
, ta chon n1 > n0 là m®t trong các so đó.
Khi đó, theo các lý lu n ở trên, ta có
1 1
hay
2
> {an1 } > ln 2 −
1992
,
1 1
1992
> ln 2 −
2
.
Mâu thuan này cháng tỏ chỉ có hǎu hạn so n sao cho {an
1
} <
2
.
b) De thay dãy (bn) tăng và lim
n→+∞
bn = +∞. Ngoài ra, ta cũng có
(2n2 + 1)(n2 + n + 1)
lim
n→+∞
(bn bn 1) = lim
→+∞
ln
(2n2 − 4n + 3)(n2
— n + 1)
= 0.
Trở lại bài toán, giả sả ton tại hǎu hạn n đe {bn
1
} <
2016
. Khi đó, ta thay ton
tại n0 ∈ N∗
đe {bn
1
} ≥
2016 với moi n ≥ n0
. Do lim
n→+∞
(bn — bn−1) = 0 nên ton tại
n1 ∈ N∗
đủ lớn đe
1
bn − bn−1 <
2016
với moi n ≥ n1. Vì dãy (bn) tăng và dan tới vô hạn nên ton tại vô so các so
n > max{n0, n1} đe [bn] − [bn−1] = 1. Xét các so n như the, tà bat đȁng thác ở trên,
ta suy ra
hay
1
[bn] − [bn−1] + {bn} − {bn−1} <
2016
,
2015
{bn−1} > {bn} +
2016
.
60
Viết đề tài giá sinh viên – ZALO:0973.287.149-TEAMLUANVAN.COM
hu
hu
k+1 k — u = 2u −
−
u
k
u0 = 1, u1 = 3
n+1
Do {bn
2015
} ≥
2016 nên {bn−1} > 1. Mâu thuan này cháng tỏ ton tại vô hạn so n sao
cho {bn
1
} <
2016
.
Bài toán 4.14 (TST Vi t Nam 2011). Cho dãy so (un) được xác định như sau:
un+2 = 1 +
2
n+1
un
i
, n ≥ 0.
(4.15)
Cháng minh rang un+2un − u2
= 2n với moi so tự nhiên n.
( đây, ta kí hi u [un] là so nguyên lớn nhat không vượt quá un).
Bài giai.
Ta thả dự đoán (un) là dãy tuyen tính dạng un+2 = pun+1 + qun + r với moi n ≥ 0.
Theo công thác truy hoi ta tính được u2 = 10, u3 = 34, u4 = 116.
Tà un+2 = pun+1 + qun + r ta có h
Do đó
3p + q + r = 10
10p + 3q + r = 34
34p + 10q + r = 116
p = 4
⇔ q = −2
r = 0.
un+2 = 4un+1 − 2un, ∀n ≥ 0. (1)
Ta sě cháng minh dãy (un) thỏa mãn công thác truy hoi (1) bang hai cách.
Cách 1.
Ta cháng minh bang quy nạp công thác truy hoi (1).
Tà đȁng thác un+2 = 1 +
2
n+1
un
i
, bang quy nạp ta suy ra un+1 > 2un, ∀n ≥ 0 nên
un > 2un−1 > · · · > 2nu0 = 2n, ∀n ≥ 1. (2)
De thay (1) đúng với n = 0.
Giả sả (1) đúng đen n = k ≥ 0 tác là uk+2 = 4uk+1 − 2un
uk+2 + 2uk
⇒
uk+1
2
=
uk+1 + 2uk−1
uk
2 k
⇒ uk+2uk − uk+1 = 2(uk+1uk−1 − uk) = · · · = 2 .
uk+2uk 2u2
Ta có uk+2uk − u2
= 2(uk+1uk−1 − u2) ⇒
uk+22uk
uk+1
k
k+1 k 1
k+1
4u2
⇒
uk+1
— 2uk+1 = 4uk−1 −
u k+1
61
Viết đề tài giá sinh viên – ZALO:0973.287.149-TEAMLUANVAN.COM
k
k+2
⇒ −
u
+ 4u − k
k
−
1
k
— k+2
k+2
k+1
hu hu
2 2 n
n+1
x x
hx hx
n+1
i
uk+2(4uk+1 − uk+2)
uk+1
4u2
— 2uk+1 = 4uk−1 −
u
u2
4uk+2 2uk+1 =
k+1
4u2
uk+1
u2
= k+2
uk+1
4(uk+1uk−1 − u2
)
uk+1
Ket hợp với (2) ta được:
u2
= k+2
+
uk+1
4.2k−1
uk+1
u2
= k+2
+
uk+1
2k+1
.
uk+1
2
k+2
uk+1
u2
< 4u 2u < + 1
uk+1
⇒ 4uk+2 − 2uk+1 =
2
k+2
uk+1
+ 1
i
=
2
k+2
uk+1
+ 1 = uk+3
Do đó (1) đúng với n = k + 1. V y đȁng thác (1) đúng với moi n ≥ 0.
Tà (1) suy ra được un+2 = 4un+1 − 2un
un+2 + 2un
⇒
un+1
=
un+1 + 2un−1
un
⇒ un+2un — un+1 = 2(un+1un−1 — un) = · · · = 2
hay un+2un − u2 = 2n với moi so tự nhiên n.
Cách 2.
Ta xây dựng dãy (xn) thỏa mãn x0 = 1, x1 = 3 và xn+2 = 4xn+1 − 2xn, ∀n ≥ 0. Tà
cách xây dựng dãy (xn) ta được:
xn+2 + 2xn
=
xn+1 + 2xn−1
⇒ x x + 2x2 = x2 + 2x x
xn+1 xn
2
n+2 n
2
n n+1
n
n+1
2
n−1
n
⇒ xn+2xn − xn+1 = 2(xn+1xn−1 − xn) = · · · = 2 (x2x0 − x1) = 2
x2
2n
n+1
⇒ xn+2 = + .
xn xn
Bang quy nạp de thay dãy (xn) là m®t dãy tăng và do đó
xn = 4xn−1 − 2xn−2 > 2xn−1 > 22xn−2 > · · · > 2nx1 = 2n.
Suy ra
2
n+1
xn
2
2n
< xn+2 = n+1
+
xn xn
2
< n+1
+ 1
xn
⇒ xn+2 =
2
n+1
xn
+ 1
i
= 1 +
2
n+1
xn
i
, ∀n ≥ 0.
hx2 i
V y dãy (xn) là m®t dãy thỏa mãn x0 = 1, x1 = 3 và xn+2 = 1 +
Do đó ta được un = xn, ∀n ≥ 0.
n+1
xn
, ∀n ≥ 0.
V y un+2un − u2 = 2n với moi so tự nhiên n.
k+1
u
x
⇒
+
62
Viết đề tài giá sinh viên – ZALO:0973.287.149-TEAMLUANVAN.COM
n
k
n−2
2
n
2
Bài toán 4.15. Cho dãy so (un) được xác định như sau:
u0 = 0, u1 = 1
(4.16)
un+2 − 3un+1 + un
2
= (−1)n+1, ∀n ≥ 0.
Cháng minh rang với moi so tự nhiên n, un là so chính phương.
Bài giai.
Ta có u2 = 1; u3 = 4; u4 = 9; u5 = 25.
Do đó u0 = F 2; u1 = F2; u2 = F2; u3 = F2; u4 = F2; u5 = F2, ở đó (Fn) là dãy
0 1 2
Fibonacci.
3 4 5
Tà đó ta có định hướng cháng minh un = F2 bang quy nạp theo n.
Giả sả uk = F 2 với moi k ≤ n. Như v y
un = F2; un−1 = F2 ; un−2 = F2 .
n n−1 n−2
Tà giả thiet ta có un+1 − 3un + un−1 = 2.(−1)n và un − 3un−1 + un−2 = 2.(−1)n−1.
C®ng hai đȁng thác trên ta được un+1 − 2un − 2un−1 + un−2 = 0, n ≥ 2.
Tà đó suy ra
un+1 = 2F2 + 2F2 − F
n n−1 n−2
= (Fn + Fn−1)2 + (Fn − Fn−1)2 − F2
2
n+1
+ F2
n−2 — Fn−2 n+1.
V y un = F2, ∀n ≥ 0 (đieu phải cháng minh).
2
= F = F
63
Viết đề tài giá sinh viên – ZALO:0973.287.149-TEAMLUANVAN.COM
Ket lu n
Lu n văn ”M®t so dạng toán ve dãy so sinh bởi các hàm so sơ cap” trình
bày nhǎng van đe sau:
1. Lu n văn đã trình bày chi tiet m®t so tính chat cơ bản của dãy so và
các dạng toán liên quan.
2. Trình bày các dạng toán ve xác định dãy so sinh bởi các hàm hǎu t
(đa thác, phân thác hǎu t ), hàm vô t , hàm lượng giác và các hàm siêu
vi t.
3. Trình bày m®t so phương pháp tính giới hạn của dãy so.
4. Trình bày các đe toán thi hoc sinh giỏi trong nước, Olympic khu vực
và quoc te liên quan đen dãy so.
Viết đề tài giá sinh viên – ZALO:0973.287.149-TEAMLUANVAN.COM
Tài li u tham khảo
[A] Tieng Vi t
[1] Nguyen Tài Chung (2013), Boi dưỡng hoc sinh giói chuyên khảo dãy so,
NXB ĐHQG Hà N®i.
[2] Phan Huy Khải (2009), Chuyên đe so hoc và dãy so, NXB Giáo dục.
[3] Nguyen Văn M u (2006), M®t so bài toán chon loc ve dãy so, NXB Giáo
dục.
[4] Nguyen Văn M u, Lê Ngoc Lăng, Phạm The Long, Nguyen Minh Tuan
(2006), Các đe thi Olympic Sinh viên toàn quoc, NXB Giáo dục.
[5] Nguyen Văn M u (Chủ biên), Tran Nam Dũng, Nguyen Minh Tuan
(2007), Chuyên đe chon loc dãy so và áp dựng, NXB Giáo dục.
[6] Nguyen Văn M u, Nguyen Thủy Thanh (2003), Giới hạn dãy so và hàm
so, NXB Giáo dục.
[7] Lê Đình Thịnh (Chủ biên), Đ ng Đình Châu, Lê Đình Định, Phan Văn
Hạp (2001), Phương trình sai phân và m®t so úng dựng, NXB Giáo dục.
[B] Tieng Anh
[8] Radulescu.T-L.T, Radulescu.V.D, Andreescu.T (2009), Problems in
Real Analysis: Advanced Calculus on the Real Axis, Springer Sci-
ences+Business Media.
[9] Paulo Ney de Sausa, Jorge- Nume Silva (1998), Berkeley Problems in
Mathematics, Springer.

More Related Content

Similar to M T So Dạng Toán Ve Dãy So Sinh B I Các Hàm So Sơ Cap.docx

Ứng Dụng Hình Học Giải Tích Vào Giải Phương Trình, Bất Phương Trình Và Hệ Phư...
Ứng Dụng Hình Học Giải Tích Vào Giải Phương Trình, Bất Phương Trình Và Hệ Phư...Ứng Dụng Hình Học Giải Tích Vào Giải Phương Trình, Bất Phương Trình Và Hệ Phư...
Ứng Dụng Hình Học Giải Tích Vào Giải Phương Trình, Bất Phương Trình Và Hệ Phư...Dịch vụ viết đề tài trọn gói 0934.573.149
 
giải tích tuần 1.pptx
giải tích tuần 1.pptxgiải tích tuần 1.pptx
giải tích tuần 1.pptxTinTng26
 

Similar to M T So Dạng Toán Ve Dãy So Sinh B I Các Hàm So Sơ Cap.docx (20)

V N Dụng Chuői Đieu Hòa Vào Giải M T So Bài Toán Dành Cho Hoc Sinh Giỏi.docx
V N Dụng Chuői Đieu Hòa Vào Giải M T So Bài Toán Dành Cho Hoc Sinh Giỏi.docxV N Dụng Chuői Đieu Hòa Vào Giải M T So Bài Toán Dành Cho Hoc Sinh Giỏi.docx
V N Dụng Chuői Đieu Hòa Vào Giải M T So Bài Toán Dành Cho Hoc Sinh Giỏi.docx
 
Đang thức, bat đang thức tích phân trong l p đa thức và phân thức hữu ty và m...
Đang thức, bat đang thức tích phân trong l p đa thức và phân thức hữu ty và m...Đang thức, bat đang thức tích phân trong l p đa thức và phân thức hữu ty và m...
Đang thức, bat đang thức tích phân trong l p đa thức và phân thức hữu ty và m...
 
Bat Phương Trình Hàm Sinh B I Các Đại Lư Ng Trung Bình B C Tùy Ý Và Các Dạng ...
Bat Phương Trình Hàm Sinh B I Các Đại Lư Ng Trung Bình B C Tùy Ý Và Các Dạng ...Bat Phương Trình Hàm Sinh B I Các Đại Lư Ng Trung Bình B C Tùy Ý Và Các Dạng ...
Bat Phương Trình Hàm Sinh B I Các Đại Lư Ng Trung Bình B C Tùy Ý Và Các Dạng ...
 
M T So L P Phương Trình Diophantine.docx
M T So L P Phương Trình Diophantine.docxM T So L P Phương Trình Diophantine.docx
M T So L P Phương Trình Diophantine.docx
 
Hàm Đơn Đi U, Tựa Đơn Đi U Và M T So Ứng Dụng Của Phép Đơn Đi U Hóa Hàm So.docx
Hàm Đơn Đi U, Tựa Đơn Đi U Và M T So Ứng Dụng Của Phép Đơn Đi U Hóa Hàm So.docxHàm Đơn Đi U, Tựa Đơn Đi U Và M T So Ứng Dụng Của Phép Đơn Đi U Hóa Hàm So.docx
Hàm Đơn Đi U, Tựa Đơn Đi U Và M T So Ứng Dụng Của Phép Đơn Đi U Hóa Hàm So.docx
 
Luận văn: Một số bài toán về dãy số, HAY, 9đ
Luận văn: Một số bài toán về dãy số, HAY, 9đLuận văn: Một số bài toán về dãy số, HAY, 9đ
Luận văn: Một số bài toán về dãy số, HAY, 9đ
 
Một số phương pháp tìm cực trị của các hàm phân thức Sinh bởi số tự nhiên.docx
Một số phương pháp tìm cực trị của các hàm phân thức Sinh bởi số tự nhiên.docxMột số phương pháp tìm cực trị của các hàm phân thức Sinh bởi số tự nhiên.docx
Một số phương pháp tìm cực trị của các hàm phân thức Sinh bởi số tự nhiên.docx
 
Các so to h p Và m t so ứng dụng trong thong kê.docx
Các so to h p Và m t so ứng dụng trong thong kê.docxCác so to h p Và m t so ứng dụng trong thong kê.docx
Các so to h p Và m t so ứng dụng trong thong kê.docx
 
Ve H Phương Trình Phi Tuyen Và Ứng Dụng.docx
Ve H Phương Trình Phi Tuyen Và Ứng Dụng.docxVe H Phương Trình Phi Tuyen Và Ứng Dụng.docx
Ve H Phương Trình Phi Tuyen Và Ứng Dụng.docx
 
Luận văn thạc sĩ: Quy hoạch toàn phương, HAY, 9đ
Luận văn thạc sĩ: Quy hoạch toàn phương, HAY, 9đLuận văn thạc sĩ: Quy hoạch toàn phương, HAY, 9đ
Luận văn thạc sĩ: Quy hoạch toàn phương, HAY, 9đ
 
Luận văn thạc sĩ - Một số lớp phương trình hàm trong số học.doc
Luận văn thạc sĩ - Một số lớp phương trình hàm trong số học.docLuận văn thạc sĩ - Một số lớp phương trình hàm trong số học.doc
Luận văn thạc sĩ - Một số lớp phương trình hàm trong số học.doc
 
Phương Pháp Lư Ng Giác Giải Phương Trình Đa Thức Và M T So Dạng Toán.docx
Phương Pháp Lư Ng Giác Giải Phương Trình Đa Thức Và M T So Dạng Toán.docxPhương Pháp Lư Ng Giác Giải Phương Trình Đa Thức Và M T So Dạng Toán.docx
Phương Pháp Lư Ng Giác Giải Phương Trình Đa Thức Và M T So Dạng Toán.docx
 
Đa Thức Trong Các Bài Toán Thi Học Sinh Giỏi.docx
Đa Thức Trong Các Bài Toán Thi Học Sinh Giỏi.docxĐa Thức Trong Các Bài Toán Thi Học Sinh Giỏi.docx
Đa Thức Trong Các Bài Toán Thi Học Sinh Giỏi.docx
 
Bất đẳng thức Trong lớp các hàm lượng giác và lượng giác ngược.docx
Bất đẳng thức Trong lớp các hàm lượng giác và lượng giác ngược.docxBất đẳng thức Trong lớp các hàm lượng giác và lượng giác ngược.docx
Bất đẳng thức Trong lớp các hàm lượng giác và lượng giác ngược.docx
 
Định lý zsigmondy và Tính chất số học của đa thức.docx
Định lý zsigmondy và Tính chất số học của đa thức.docxĐịnh lý zsigmondy và Tính chất số học của đa thức.docx
Định lý zsigmondy và Tính chất số học của đa thức.docx
 
Luận văn thạc sĩ - Đa thức trong các bài toán thi học sinh giỏi.doc
Luận văn thạc sĩ - Đa thức trong các bài toán thi học sinh giỏi.docLuận văn thạc sĩ - Đa thức trong các bài toán thi học sinh giỏi.doc
Luận văn thạc sĩ - Đa thức trong các bài toán thi học sinh giỏi.doc
 
M T So Ứng Dụng Của Công Thức N I Suy Lagrange Và Hermite.docx
M T So Ứng Dụng Của Công Thức N I Suy Lagrange Và Hermite.docxM T So Ứng Dụng Của Công Thức N I Suy Lagrange Và Hermite.docx
M T So Ứng Dụng Của Công Thức N I Suy Lagrange Và Hermite.docx
 
Ứng Dụng Hình Học Giải Tích Vào Giải Phương Trình, Bất Phương Trình Và Hệ Phư...
Ứng Dụng Hình Học Giải Tích Vào Giải Phương Trình, Bất Phương Trình Và Hệ Phư...Ứng Dụng Hình Học Giải Tích Vào Giải Phương Trình, Bất Phương Trình Và Hệ Phư...
Ứng Dụng Hình Học Giải Tích Vào Giải Phương Trình, Bất Phương Trình Và Hệ Phư...
 
giải tích tuần 1.pptx
giải tích tuần 1.pptxgiải tích tuần 1.pptx
giải tích tuần 1.pptx
 
Dãy Diatomic Của Stern.docx
Dãy Diatomic Của Stern.docxDãy Diatomic Của Stern.docx
Dãy Diatomic Của Stern.docx
 

More from DV Viết Luận văn luanvanmaster.com ZALO 0973287149

More from DV Viết Luận văn luanvanmaster.com ZALO 0973287149 (20)

Ảnh Hưởng Của Marketing Quan Hệ Đến Lòng Trung Thành Của Khách Hàng.Tình Huốn...
Ảnh Hưởng Của Marketing Quan Hệ Đến Lòng Trung Thành Của Khách Hàng.Tình Huốn...Ảnh Hưởng Của Marketing Quan Hệ Đến Lòng Trung Thành Của Khách Hàng.Tình Huốn...
Ảnh Hưởng Của Marketing Quan Hệ Đến Lòng Trung Thành Của Khách Hàng.Tình Huốn...
 
Phát triển nguồn nhân lực tại Uỷ ban nhân dân huyện Trà Bồng, tỉnh Quảng Ngãi...
Phát triển nguồn nhân lực tại Uỷ ban nhân dân huyện Trà Bồng, tỉnh Quảng Ngãi...Phát triển nguồn nhân lực tại Uỷ ban nhân dân huyện Trà Bồng, tỉnh Quảng Ngãi...
Phát triển nguồn nhân lực tại Uỷ ban nhân dân huyện Trà Bồng, tỉnh Quảng Ngãi...
 
Báo cáo tốt Nghiệp tài chính hợp nhất tại tổng công ty Indochina gol...
Báo cáo tốt Nghiệp  tài chính hợp nhất tại tổng công ty Indochina gol...Báo cáo tốt Nghiệp  tài chính hợp nhất tại tổng công ty Indochina gol...
Báo cáo tốt Nghiệp tài chính hợp nhất tại tổng công ty Indochina gol...
 
Tạo động lực thúc đẩy nhân viên làm việc tại ngân hàng TMCP Ngoại Thương Việt...
Tạo động lực thúc đẩy nhân viên làm việc tại ngân hàng TMCP Ngoại Thương Việt...Tạo động lực thúc đẩy nhân viên làm việc tại ngân hàng TMCP Ngoại Thương Việt...
Tạo động lực thúc đẩy nhân viên làm việc tại ngân hàng TMCP Ngoại Thương Việt...
 
Phát triển công nghiệp trên địa bàn Thành phố Tam Kỳ, Tỉnh Quảng Na...
Phát triển công nghiệp trên địa bàn Thành phố Tam Kỳ, Tỉnh Quảng Na...Phát triển công nghiệp trên địa bàn Thành phố Tam Kỳ, Tỉnh Quảng Na...
Phát triển công nghiệp trên địa bàn Thành phố Tam Kỳ, Tỉnh Quảng Na...
 
Giải pháp phát triển cho vay xuất nhập khẩu tại ngân hàng NN&PTNN ch...
Giải pháp phát triển cho vay xuất nhập khẩu tại ngân hàng NN&PTNN ch...Giải pháp phát triển cho vay xuất nhập khẩu tại ngân hàng NN&PTNN ch...
Giải pháp phát triển cho vay xuất nhập khẩu tại ngân hàng NN&PTNN ch...
 
Hoàn thiện công tác lập báo cáo tài chính hợp nhất tại tổng công ...
Hoàn thiện công tác lập báo cáo tài chính hợp nhất tại tổng công ...Hoàn thiện công tác lập báo cáo tài chính hợp nhất tại tổng công ...
Hoàn thiện công tác lập báo cáo tài chính hợp nhất tại tổng công ...
 
Luận Văn Thạc Sĩ Quản trị thành tích nhân viên tại Cục Hải quan TP Đà Nẵng.doc
Luận Văn Thạc Sĩ  Quản trị thành tích nhân viên tại Cục Hải quan TP Đà Nẵng.docLuận Văn Thạc Sĩ  Quản trị thành tích nhân viên tại Cục Hải quan TP Đà Nẵng.doc
Luận Văn Thạc Sĩ Quản trị thành tích nhân viên tại Cục Hải quan TP Đà Nẵng.doc
 
Hoàn thiện công tác quản lý thuế thu nhập cá nhân tại cục thuế Tỉ...
Hoàn thiện công tác quản lý thuế thu nhập cá nhân tại cục thuế Tỉ...Hoàn thiện công tác quản lý thuế thu nhập cá nhân tại cục thuế Tỉ...
Hoàn thiện công tác quản lý thuế thu nhập cá nhân tại cục thuế Tỉ...
 
Đề Tài Phát triển bền vững nông nghiệp Huyện Ba Tơ, Tỉnh Quảng Ngãi....
Đề Tài Phát triển bền vững nông nghiệp Huyện Ba Tơ, Tỉnh Quảng Ngãi....Đề Tài Phát triển bền vững nông nghiệp Huyện Ba Tơ, Tỉnh Quảng Ngãi....
Đề Tài Phát triển bền vững nông nghiệp Huyện Ba Tơ, Tỉnh Quảng Ngãi....
 
Hoàn thiện công tác bảo trợ xã hội trên địa bàn huyện Phong Điền, tỉnh Thừa T...
Hoàn thiện công tác bảo trợ xã hội trên địa bàn huyện Phong Điền, tỉnh Thừa T...Hoàn thiện công tác bảo trợ xã hội trên địa bàn huyện Phong Điền, tỉnh Thừa T...
Hoàn thiện công tác bảo trợ xã hội trên địa bàn huyện Phong Điền, tỉnh Thừa T...
 
Đề Tài Luận VănPhát triển sản phẩm du lịch tại thành phố Đà Nẵng.doc
Đề Tài Luận VănPhát triển sản phẩm du lịch tại thành phố Đà Nẵng.docĐề Tài Luận VănPhát triển sản phẩm du lịch tại thành phố Đà Nẵng.doc
Đề Tài Luận VănPhát triển sản phẩm du lịch tại thành phố Đà Nẵng.doc
 
Đào tạo nghề cho lao động thuộc diện thu hồi đất trên địa bàn Thàn...
Đào tạo nghề cho lao động thuộc diện thu hồi đất trên địa bàn Thàn...Đào tạo nghề cho lao động thuộc diện thu hồi đất trên địa bàn Thàn...
Đào tạo nghề cho lao động thuộc diện thu hồi đất trên địa bàn Thàn...
 
Tóm Tắt Luận Văn Thạc Sĩ Quản Trị Kinh Doanh Xây dựng chính sách Marketing tạ...
Tóm Tắt Luận Văn Thạc Sĩ Quản Trị Kinh Doanh Xây dựng chính sách Marketing tạ...Tóm Tắt Luận Văn Thạc Sĩ Quản Trị Kinh Doanh Xây dựng chính sách Marketing tạ...
Tóm Tắt Luận Văn Thạc Sĩ Quản Trị Kinh Doanh Xây dựng chính sách Marketing tạ...
 
Đề Tài Nghiên cứu rủi ro cảm nhận đối với mua hàng thời trang trực tuyến.docx
Đề Tài Nghiên cứu rủi ro cảm nhận đối với mua hàng thời trang trực tuyến.docxĐề Tài Nghiên cứu rủi ro cảm nhận đối với mua hàng thời trang trực tuyến.docx
Đề Tài Nghiên cứu rủi ro cảm nhận đối với mua hàng thời trang trực tuyến.docx
 
Giải pháp nâng cao động lực thúc đẩy người lao động tại công ty khai...
Giải pháp nâng cao động lực thúc đẩy người lao động tại công ty khai...Giải pháp nâng cao động lực thúc đẩy người lao động tại công ty khai...
Giải pháp nâng cao động lực thúc đẩy người lao động tại công ty khai...
 
Giải pháp phát triển dịch vụ ngân hàng điện tử tại ngân hàng đầu ...
Giải pháp phát triển dịch vụ ngân hàng điện tử tại ngân hàng đầu ...Giải pháp phát triển dịch vụ ngân hàng điện tử tại ngân hàng đầu ...
Giải pháp phát triển dịch vụ ngân hàng điện tử tại ngân hàng đầu ...
 
Giải pháp phát triển dịch vụ ngân hàng điện tử tại ngân hàng đầu ...
Giải pháp phát triển dịch vụ ngân hàng điện tử tại ngân hàng đầu ...Giải pháp phát triển dịch vụ ngân hàng điện tử tại ngân hàng đầu ...
Giải pháp phát triển dịch vụ ngân hàng điện tử tại ngân hàng đầu ...
 
Quản trị quan hệ khách hàng tại Chi nhánh Viettel Đà Nẵng – Tập đoàn Viễn thô...
Quản trị quan hệ khách hàng tại Chi nhánh Viettel Đà Nẵng – Tập đoàn Viễn thô...Quản trị quan hệ khách hàng tại Chi nhánh Viettel Đà Nẵng – Tập đoàn Viễn thô...
Quản trị quan hệ khách hàng tại Chi nhánh Viettel Đà Nẵng – Tập đoàn Viễn thô...
 
Đề Tài Đánh giá thành tích đội ngũ giảng viên trường Đại Học Phạm ...
Đề Tài Đánh giá thành tích đội ngũ giảng viên trường Đại Học Phạm ...Đề Tài Đánh giá thành tích đội ngũ giảng viên trường Đại Học Phạm ...
Đề Tài Đánh giá thành tích đội ngũ giảng viên trường Đại Học Phạm ...
 

Recently uploaded

30 ĐỀ PHÁT TRIỂN THEO CẤU TRÚC ĐỀ MINH HỌA BGD NGÀY 22-3-2024 KỲ THI TỐT NGHI...
30 ĐỀ PHÁT TRIỂN THEO CẤU TRÚC ĐỀ MINH HỌA BGD NGÀY 22-3-2024 KỲ THI TỐT NGHI...30 ĐỀ PHÁT TRIỂN THEO CẤU TRÚC ĐỀ MINH HỌA BGD NGÀY 22-3-2024 KỲ THI TỐT NGHI...
30 ĐỀ PHÁT TRIỂN THEO CẤU TRÚC ĐỀ MINH HỌA BGD NGÀY 22-3-2024 KỲ THI TỐT NGHI...Nguyen Thanh Tu Collection
 
30 ĐỀ PHÁT TRIỂN THEO CẤU TRÚC ĐỀ MINH HỌA BGD NGÀY 22-3-2024 KỲ THI TỐT NGHI...
30 ĐỀ PHÁT TRIỂN THEO CẤU TRÚC ĐỀ MINH HỌA BGD NGÀY 22-3-2024 KỲ THI TỐT NGHI...30 ĐỀ PHÁT TRIỂN THEO CẤU TRÚC ĐỀ MINH HỌA BGD NGÀY 22-3-2024 KỲ THI TỐT NGHI...
30 ĐỀ PHÁT TRIỂN THEO CẤU TRÚC ĐỀ MINH HỌA BGD NGÀY 22-3-2024 KỲ THI TỐT NGHI...Nguyen Thanh Tu Collection
 
Chuong trinh dao tao Su pham Khoa hoc tu nhien, ma nganh - 7140247.pdf
Chuong trinh dao tao Su pham Khoa hoc tu nhien, ma nganh - 7140247.pdfChuong trinh dao tao Su pham Khoa hoc tu nhien, ma nganh - 7140247.pdf
Chuong trinh dao tao Su pham Khoa hoc tu nhien, ma nganh - 7140247.pdfhoangtuansinh1
 
powerpoint lịch sử đảng cộng sản việt nam.pptx
powerpoint lịch sử đảng cộng sản việt nam.pptxpowerpoint lịch sử đảng cộng sản việt nam.pptx
powerpoint lịch sử đảng cộng sản việt nam.pptxAnAn97022
 
Chàm - Bệnh án (da liễu - bvdlct ctump) .pptx
Chàm - Bệnh án (da liễu - bvdlct ctump) .pptxChàm - Bệnh án (da liễu - bvdlct ctump) .pptx
Chàm - Bệnh án (da liễu - bvdlct ctump) .pptxendkay31
 
BỘ ĐỀ PHÁT TRIỂN THEO CẤU TRÚC ĐỀ MINH HỌA BGD NGÀY 22-3-2024 KỲ THI TỐT NGHI...
BỘ ĐỀ PHÁT TRIỂN THEO CẤU TRÚC ĐỀ MINH HỌA BGD NGÀY 22-3-2024 KỲ THI TỐT NGHI...BỘ ĐỀ PHÁT TRIỂN THEO CẤU TRÚC ĐỀ MINH HỌA BGD NGÀY 22-3-2024 KỲ THI TỐT NGHI...
BỘ ĐỀ PHÁT TRIỂN THEO CẤU TRÚC ĐỀ MINH HỌA BGD NGÀY 22-3-2024 KỲ THI TỐT NGHI...Nguyen Thanh Tu Collection
 
Đề cương môn giải phẫu......................
Đề cương môn giải phẫu......................Đề cương môn giải phẫu......................
Đề cương môn giải phẫu......................TrnHoa46
 
30 ĐỀ PHÁT TRIỂN THEO CẤU TRÚC ĐỀ MINH HỌA BGD NGÀY 22-3-2024 KỲ THI TỐT NGHI...
30 ĐỀ PHÁT TRIỂN THEO CẤU TRÚC ĐỀ MINH HỌA BGD NGÀY 22-3-2024 KỲ THI TỐT NGHI...30 ĐỀ PHÁT TRIỂN THEO CẤU TRÚC ĐỀ MINH HỌA BGD NGÀY 22-3-2024 KỲ THI TỐT NGHI...
30 ĐỀ PHÁT TRIỂN THEO CẤU TRÚC ĐỀ MINH HỌA BGD NGÀY 22-3-2024 KỲ THI TỐT NGHI...Nguyen Thanh Tu Collection
 
sách sinh học đại cương - Textbook.pdf
sách sinh học đại cương   -   Textbook.pdfsách sinh học đại cương   -   Textbook.pdf
sách sinh học đại cương - Textbook.pdfTrnHoa46
 
Kiểm tra chạy trạm lí thuyết giữa kì giải phẫu sinh lí
Kiểm tra chạy trạm lí thuyết giữa kì giải phẫu sinh líKiểm tra chạy trạm lí thuyết giữa kì giải phẫu sinh lí
Kiểm tra chạy trạm lí thuyết giữa kì giải phẫu sinh líDr K-OGN
 
chuong-7-van-de-gia-dinh-trong-thoi-ky-qua-do-len-cnxh.pdf
chuong-7-van-de-gia-dinh-trong-thoi-ky-qua-do-len-cnxh.pdfchuong-7-van-de-gia-dinh-trong-thoi-ky-qua-do-len-cnxh.pdf
chuong-7-van-de-gia-dinh-trong-thoi-ky-qua-do-len-cnxh.pdfVyTng986513
 
NQA Lợi ích Từ ISO và ESG Tăng Trưởng và Bền Vững ver01.pdf
NQA Lợi ích Từ ISO và ESG Tăng Trưởng và Bền Vững ver01.pdfNQA Lợi ích Từ ISO và ESG Tăng Trưởng và Bền Vững ver01.pdf
NQA Lợi ích Từ ISO và ESG Tăng Trưởng và Bền Vững ver01.pdfNguyễn Đăng Quang
 
QUẢN LÝ HOẠT ĐỘNG GIÁO DỤC KỸ NĂNG SỐNG CHO HỌC SINH CÁC TRƯỜNG TRUNG HỌC CƠ ...
QUẢN LÝ HOẠT ĐỘNG GIÁO DỤC KỸ NĂNG SỐNG CHO HỌC SINH CÁC TRƯỜNG TRUNG HỌC CƠ ...QUẢN LÝ HOẠT ĐỘNG GIÁO DỤC KỸ NĂNG SỐNG CHO HỌC SINH CÁC TRƯỜNG TRUNG HỌC CƠ ...
QUẢN LÝ HOẠT ĐỘNG GIÁO DỤC KỸ NĂNG SỐNG CHO HỌC SINH CÁC TRƯỜNG TRUNG HỌC CƠ ...ThunTrn734461
 
Sơ đồ tư duy môn sinh học bậc THPT.pdf
Sơ đồ tư duy môn sinh học bậc THPT.pdfSơ đồ tư duy môn sinh học bậc THPT.pdf
Sơ đồ tư duy môn sinh học bậc THPT.pdftohoanggiabao81
 
Sáng kiến Dạy học theo định hướng STEM một số chủ đề phần “vật sống”, Khoa họ...
Sáng kiến Dạy học theo định hướng STEM một số chủ đề phần “vật sống”, Khoa họ...Sáng kiến Dạy học theo định hướng STEM một số chủ đề phần “vật sống”, Khoa họ...
Sáng kiến Dạy học theo định hướng STEM một số chủ đề phần “vật sống”, Khoa họ...Nguyen Thanh Tu Collection
 
ôn tập lịch sử hhhhhhhhhhhhhhhhhhhhhhhhhh
ôn tập lịch sử hhhhhhhhhhhhhhhhhhhhhhhhhhôn tập lịch sử hhhhhhhhhhhhhhhhhhhhhhhhhh
ôn tập lịch sử hhhhhhhhhhhhhhhhhhhhhhhhhhvanhathvc
 
Trích dẫn trắc nghiệm tư tưởng HCM5.docx
Trích dẫn trắc nghiệm tư tưởng HCM5.docxTrích dẫn trắc nghiệm tư tưởng HCM5.docx
Trích dẫn trắc nghiệm tư tưởng HCM5.docxnhungdt08102004
 
30 ĐỀ PHÁT TRIỂN THEO CẤU TRÚC ĐỀ MINH HỌA BGD NGÀY 22-3-2024 KỲ THI TỐT NGHI...
30 ĐỀ PHÁT TRIỂN THEO CẤU TRÚC ĐỀ MINH HỌA BGD NGÀY 22-3-2024 KỲ THI TỐT NGHI...30 ĐỀ PHÁT TRIỂN THEO CẤU TRÚC ĐỀ MINH HỌA BGD NGÀY 22-3-2024 KỲ THI TỐT NGHI...
30 ĐỀ PHÁT TRIỂN THEO CẤU TRÚC ĐỀ MINH HỌA BGD NGÀY 22-3-2024 KỲ THI TỐT NGHI...Nguyen Thanh Tu Collection
 
GIÁO TRÌNH KHỐI NGUỒN CÁC LOẠI - ĐIỆN LẠNH BÁCH KHOA HÀ NỘI
GIÁO TRÌNH  KHỐI NGUỒN CÁC LOẠI - ĐIỆN LẠNH BÁCH KHOA HÀ NỘIGIÁO TRÌNH  KHỐI NGUỒN CÁC LOẠI - ĐIỆN LẠNH BÁCH KHOA HÀ NỘI
GIÁO TRÌNH KHỐI NGUỒN CÁC LOẠI - ĐIỆN LẠNH BÁCH KHOA HÀ NỘIĐiện Lạnh Bách Khoa Hà Nội
 
Sáng kiến “Sử dụng ứng dụng Quizizz nhằm nâng cao chất lượng ôn thi tốt nghiệ...
Sáng kiến “Sử dụng ứng dụng Quizizz nhằm nâng cao chất lượng ôn thi tốt nghiệ...Sáng kiến “Sử dụng ứng dụng Quizizz nhằm nâng cao chất lượng ôn thi tốt nghiệ...
Sáng kiến “Sử dụng ứng dụng Quizizz nhằm nâng cao chất lượng ôn thi tốt nghiệ...Nguyen Thanh Tu Collection
 

Recently uploaded (20)

30 ĐỀ PHÁT TRIỂN THEO CẤU TRÚC ĐỀ MINH HỌA BGD NGÀY 22-3-2024 KỲ THI TỐT NGHI...
30 ĐỀ PHÁT TRIỂN THEO CẤU TRÚC ĐỀ MINH HỌA BGD NGÀY 22-3-2024 KỲ THI TỐT NGHI...30 ĐỀ PHÁT TRIỂN THEO CẤU TRÚC ĐỀ MINH HỌA BGD NGÀY 22-3-2024 KỲ THI TỐT NGHI...
30 ĐỀ PHÁT TRIỂN THEO CẤU TRÚC ĐỀ MINH HỌA BGD NGÀY 22-3-2024 KỲ THI TỐT NGHI...
 
30 ĐỀ PHÁT TRIỂN THEO CẤU TRÚC ĐỀ MINH HỌA BGD NGÀY 22-3-2024 KỲ THI TỐT NGHI...
30 ĐỀ PHÁT TRIỂN THEO CẤU TRÚC ĐỀ MINH HỌA BGD NGÀY 22-3-2024 KỲ THI TỐT NGHI...30 ĐỀ PHÁT TRIỂN THEO CẤU TRÚC ĐỀ MINH HỌA BGD NGÀY 22-3-2024 KỲ THI TỐT NGHI...
30 ĐỀ PHÁT TRIỂN THEO CẤU TRÚC ĐỀ MINH HỌA BGD NGÀY 22-3-2024 KỲ THI TỐT NGHI...
 
Chuong trinh dao tao Su pham Khoa hoc tu nhien, ma nganh - 7140247.pdf
Chuong trinh dao tao Su pham Khoa hoc tu nhien, ma nganh - 7140247.pdfChuong trinh dao tao Su pham Khoa hoc tu nhien, ma nganh - 7140247.pdf
Chuong trinh dao tao Su pham Khoa hoc tu nhien, ma nganh - 7140247.pdf
 
powerpoint lịch sử đảng cộng sản việt nam.pptx
powerpoint lịch sử đảng cộng sản việt nam.pptxpowerpoint lịch sử đảng cộng sản việt nam.pptx
powerpoint lịch sử đảng cộng sản việt nam.pptx
 
Chàm - Bệnh án (da liễu - bvdlct ctump) .pptx
Chàm - Bệnh án (da liễu - bvdlct ctump) .pptxChàm - Bệnh án (da liễu - bvdlct ctump) .pptx
Chàm - Bệnh án (da liễu - bvdlct ctump) .pptx
 
BỘ ĐỀ PHÁT TRIỂN THEO CẤU TRÚC ĐỀ MINH HỌA BGD NGÀY 22-3-2024 KỲ THI TỐT NGHI...
BỘ ĐỀ PHÁT TRIỂN THEO CẤU TRÚC ĐỀ MINH HỌA BGD NGÀY 22-3-2024 KỲ THI TỐT NGHI...BỘ ĐỀ PHÁT TRIỂN THEO CẤU TRÚC ĐỀ MINH HỌA BGD NGÀY 22-3-2024 KỲ THI TỐT NGHI...
BỘ ĐỀ PHÁT TRIỂN THEO CẤU TRÚC ĐỀ MINH HỌA BGD NGÀY 22-3-2024 KỲ THI TỐT NGHI...
 
Đề cương môn giải phẫu......................
Đề cương môn giải phẫu......................Đề cương môn giải phẫu......................
Đề cương môn giải phẫu......................
 
30 ĐỀ PHÁT TRIỂN THEO CẤU TRÚC ĐỀ MINH HỌA BGD NGÀY 22-3-2024 KỲ THI TỐT NGHI...
30 ĐỀ PHÁT TRIỂN THEO CẤU TRÚC ĐỀ MINH HỌA BGD NGÀY 22-3-2024 KỲ THI TỐT NGHI...30 ĐỀ PHÁT TRIỂN THEO CẤU TRÚC ĐỀ MINH HỌA BGD NGÀY 22-3-2024 KỲ THI TỐT NGHI...
30 ĐỀ PHÁT TRIỂN THEO CẤU TRÚC ĐỀ MINH HỌA BGD NGÀY 22-3-2024 KỲ THI TỐT NGHI...
 
sách sinh học đại cương - Textbook.pdf
sách sinh học đại cương   -   Textbook.pdfsách sinh học đại cương   -   Textbook.pdf
sách sinh học đại cương - Textbook.pdf
 
Kiểm tra chạy trạm lí thuyết giữa kì giải phẫu sinh lí
Kiểm tra chạy trạm lí thuyết giữa kì giải phẫu sinh líKiểm tra chạy trạm lí thuyết giữa kì giải phẫu sinh lí
Kiểm tra chạy trạm lí thuyết giữa kì giải phẫu sinh lí
 
chuong-7-van-de-gia-dinh-trong-thoi-ky-qua-do-len-cnxh.pdf
chuong-7-van-de-gia-dinh-trong-thoi-ky-qua-do-len-cnxh.pdfchuong-7-van-de-gia-dinh-trong-thoi-ky-qua-do-len-cnxh.pdf
chuong-7-van-de-gia-dinh-trong-thoi-ky-qua-do-len-cnxh.pdf
 
NQA Lợi ích Từ ISO và ESG Tăng Trưởng và Bền Vững ver01.pdf
NQA Lợi ích Từ ISO và ESG Tăng Trưởng và Bền Vững ver01.pdfNQA Lợi ích Từ ISO và ESG Tăng Trưởng và Bền Vững ver01.pdf
NQA Lợi ích Từ ISO và ESG Tăng Trưởng và Bền Vững ver01.pdf
 
QUẢN LÝ HOẠT ĐỘNG GIÁO DỤC KỸ NĂNG SỐNG CHO HỌC SINH CÁC TRƯỜNG TRUNG HỌC CƠ ...
QUẢN LÝ HOẠT ĐỘNG GIÁO DỤC KỸ NĂNG SỐNG CHO HỌC SINH CÁC TRƯỜNG TRUNG HỌC CƠ ...QUẢN LÝ HOẠT ĐỘNG GIÁO DỤC KỸ NĂNG SỐNG CHO HỌC SINH CÁC TRƯỜNG TRUNG HỌC CƠ ...
QUẢN LÝ HOẠT ĐỘNG GIÁO DỤC KỸ NĂNG SỐNG CHO HỌC SINH CÁC TRƯỜNG TRUNG HỌC CƠ ...
 
Sơ đồ tư duy môn sinh học bậc THPT.pdf
Sơ đồ tư duy môn sinh học bậc THPT.pdfSơ đồ tư duy môn sinh học bậc THPT.pdf
Sơ đồ tư duy môn sinh học bậc THPT.pdf
 
Sáng kiến Dạy học theo định hướng STEM một số chủ đề phần “vật sống”, Khoa họ...
Sáng kiến Dạy học theo định hướng STEM một số chủ đề phần “vật sống”, Khoa họ...Sáng kiến Dạy học theo định hướng STEM một số chủ đề phần “vật sống”, Khoa họ...
Sáng kiến Dạy học theo định hướng STEM một số chủ đề phần “vật sống”, Khoa họ...
 
ôn tập lịch sử hhhhhhhhhhhhhhhhhhhhhhhhhh
ôn tập lịch sử hhhhhhhhhhhhhhhhhhhhhhhhhhôn tập lịch sử hhhhhhhhhhhhhhhhhhhhhhhhhh
ôn tập lịch sử hhhhhhhhhhhhhhhhhhhhhhhhhh
 
Trích dẫn trắc nghiệm tư tưởng HCM5.docx
Trích dẫn trắc nghiệm tư tưởng HCM5.docxTrích dẫn trắc nghiệm tư tưởng HCM5.docx
Trích dẫn trắc nghiệm tư tưởng HCM5.docx
 
30 ĐỀ PHÁT TRIỂN THEO CẤU TRÚC ĐỀ MINH HỌA BGD NGÀY 22-3-2024 KỲ THI TỐT NGHI...
30 ĐỀ PHÁT TRIỂN THEO CẤU TRÚC ĐỀ MINH HỌA BGD NGÀY 22-3-2024 KỲ THI TỐT NGHI...30 ĐỀ PHÁT TRIỂN THEO CẤU TRÚC ĐỀ MINH HỌA BGD NGÀY 22-3-2024 KỲ THI TỐT NGHI...
30 ĐỀ PHÁT TRIỂN THEO CẤU TRÚC ĐỀ MINH HỌA BGD NGÀY 22-3-2024 KỲ THI TỐT NGHI...
 
GIÁO TRÌNH KHỐI NGUỒN CÁC LOẠI - ĐIỆN LẠNH BÁCH KHOA HÀ NỘI
GIÁO TRÌNH  KHỐI NGUỒN CÁC LOẠI - ĐIỆN LẠNH BÁCH KHOA HÀ NỘIGIÁO TRÌNH  KHỐI NGUỒN CÁC LOẠI - ĐIỆN LẠNH BÁCH KHOA HÀ NỘI
GIÁO TRÌNH KHỐI NGUỒN CÁC LOẠI - ĐIỆN LẠNH BÁCH KHOA HÀ NỘI
 
Sáng kiến “Sử dụng ứng dụng Quizizz nhằm nâng cao chất lượng ôn thi tốt nghiệ...
Sáng kiến “Sử dụng ứng dụng Quizizz nhằm nâng cao chất lượng ôn thi tốt nghiệ...Sáng kiến “Sử dụng ứng dụng Quizizz nhằm nâng cao chất lượng ôn thi tốt nghiệ...
Sáng kiến “Sử dụng ứng dụng Quizizz nhằm nâng cao chất lượng ôn thi tốt nghiệ...
 

M T So Dạng Toán Ve Dãy So Sinh B I Các Hàm So Sơ Cap.docx

  • 1. ĐẠI HOC THÁI NGUYÊN TRƯ NG ĐẠI HOC KHOA HOC Tải tài liệu tại sividoc.com Viết đề tài giá sinh viên – ZALO:0973.287.149-TEAMLUANVAN.COM PHÙNG TH± THU HÀ M T SO DẠNG TOÁN VE DÃY SO SINH B I CÁC HÀM SO SƠ CAP LU N VĂN THẠC SĨ TOÁN HOC Thái Nguyên - 2016
  • 2. ĐẠI HOC THÁI NGUYÊN TRƯ NG ĐẠI HOC KHOA HOC Tải tài liệu tại sividoc.com Viết đề tài giá sinh viên – ZALO:0973.287.149-TEAMLUANVAN.COM PHÙNG TH± THU HÀ M T SO DẠNG TOÁN VE DÃY SO SINH B I CÁC HÀM SO SƠ CAP LU N VĂN THẠC SĨ TOÁN HOC Chuyên ngành: Phương pháp Toán sơ cap Mã so: 60 46 01 13 NGƯ I HƯ NG DAN KHOA HOC GS.TSKH. NGUYEN VĂN M U Thái Nguyên - 2016
  • 3. i Viết đề tài giá sinh viên – ZALO:0973.287.149-TEAMLUANVAN.COM Mnc lnc M đau 1 1 M t so kien thfíc bo tr ve dãy so 3 1.1 Dãy so, định nghĩa và tính chat . . . . . . . . . . . . . . . . . . . . . 3 1.2 Giới hạn của dãy so . . . . . . . . . . . . . . . . . . . . . . . . . . . . 5 1.3 M®t vài dãy so đ c bi t . . . . . . . . . . . . . . . . . . . . . . . . . . 6 2 M t so phương pháp giải bài toán ve xác định dãy so 10 2.1 Dãy so sinh bởi hàm đa thác . . . . . . . . . . . . . . . . . . . . . . 10 2.2 Dãy so sinh bởi hàm phân thác hǎu t . . . . . . . . . . . . . . . . . 16 2.3 Dãy so sinh bởi hàm cháa căn thác . . . . . . . . . . . . . . . . . . . 22 2.4 Dãy so sinh bởi các hàm lượng giác và siêu vi t . . . . . . . . . . . . 24 3 M t so phương pháp xác định gi i hạn của dãy so 28 3.1 Sả dụng tính đơn đi u và bị ch n đe tính giới hạn của dãy so . . . . 28 3.2 Sả dụng nguyên lý kep đe tính giới hạn của dãy so . . . . . . . . . . 35 3.3 Sả dụng định lý Lagrange đe tính giới hạn của dãy so . . . . . . . . 37 3.4 Xác định giới hạn của dãy tőng . . . . . . . . . . . . . . . . . . . . . 42 4 Các dạng toán khác liên quan đen dãy so 46 4.1 M®t so dạng toán liên quan đen tính chat của dãy so . . . . . . . . 46 4.2 M®t so dạng toán khác . . . . . . . . . . . . . . . . . . . . . . . . . . 57 Ket lu n 62 Tài li u tham khảo 63
  • 4. 1 Viết đề tài giá sinh viên – ZALO:0973.287.149-TEAMLUANVAN.COM M đau Dãy so là m®t phan quan trong của chương trình Toán phő thông và trong các ngành đại so và giải tích toán hoc. Dãy so có m®t vị trí đ c bi t quan trong trong toán hoc, không chỉ như là m®t đoi tượng đe nghiên cáu mà còn đóng m®t vai trò như m®t công cụ đac lực của các mô hình rời rạc của giải tích trong lý thuyet phương trình, lý thuyet xap xỉ, lý thuyet bieu dien. . . Trong chương trình, sách giáo khoa trung hoc phő thông, n®i dung đe c p đen dãy so rat ít. Vì v y hoc sinh g p rat nhieu khó khăn trong vi c giải các bài toán liên quan đen dãy so khi tham gia thi hoc sinh giỏi các cap. Trong các kỳ thi hoc sinh giỏi Toán cap tỉnh, cap quoc gia, thi Olympic Toán quoc te, thi Olympic sinh viên giǎa các trường đại hoc và cao đȁng, các bài toán ve dãy so được đe c p nhieu và thường thu®c loại khó. Các bài toán ve ước lượng; xác định dãy so và tính giá trị các tőng, tích; các bài toán ve cực trị, xác định giới hạn dãy hay các tính chat của dãy so thường liên quan đen đ c trưng của dãy tương áng. Lu n văn M®t so dạng toán ve dãy so sinh bới các hàm so sơ cap nham nêu m®t so phương pháp xác định dãy so, giới hạn của dãy so và các bài toán liên quan. Lu n văn gom có mở đau, bon chương n®i dung, ket lu n và tài li u tham khảo. Chương 1. M t so kien thfíc bo tr ve dãy so Chương này trình bày các kien thác liên quan đen dãy so. Chương 2. M t so phương pháp giải bài toán ve xác định dãy so Chương này trình bày các bài toán liên quan đen xác định so hạng tőng quát của dãy so sinh bởi các hàm sơ cap cơ bản đó là hàm đa thác, hàm phân thác hǎu t , hàm lượng giác, hàm so mũ và hàm so logarit. Chương 3. M t so phương pháp xác định gi i hạn của dãy so Chương này trình bày m®t so phương pháp xác định giới hạn của dãy so như
  • 5. 2 Viết đề tài giá sinh viên – ZALO:0973.287.149-TEAMLUANVAN.COM phương pháp sả dụng tính đơn đi u và bị ch n, phương pháp sả dụng nguyên lí kep, phương pháp sả dụng định lí Lagrange và xác định giới hạn của dãy tőng. Chương 4. Các dạng toán khác liên quan đen dãy so Chương này trình bày m®t so bài toán liên quan đen tính chat của dãy so nguyên, các dãy so cháa hàm phan nguyên, hàm phan lẻ. Lu n văn được hoàn thành tại trường Đại hoc Khoa hoc - Đại hoc Thái Nguyên với sự hướng dan của GS.TSKH. Nguyen Văn M u. Tác giả xin được bày tỏ lòng biet ơn sâu sac đoi với sự quan tâm hướng dan của thay, tới các thay cô trong Ban giám hi u, Phòng đào tạo và Khoa Toán - Tin của trường Đại hoc Khoa hoc. Đong thời tác giả xin cảm ơn tới Sở Giáo dục và đào tạo Yên Bái, Ban giám hi u và các thay cô trường THPT Chuyên Nguyen Tat Thành đã tạo đieu ki n cho tác giả hoc t p và hoàn thành ke hoạch hoc t p. Thái Nguyên, ngày 25 tháng 5 năm 2016. Hoc viên Phùng Thị Thu Hà
  • 6. 3 Viết đề tài giá sinh viên – ZALO:0973.287.149-TEAMLUANVAN.COM Chương 1 M t so kien thfíc bo tr ve dãy so Trong chương này, tôi trình bày khái ni m cơ bản ve dãy so gom m®t so định nghĩa và các định lý cơ bản, m®t vài dãy so đ c bi t và m®t so bài toán áp dụng. 1.1 Dãy so, định nghĩa và tính chat Định nghĩa 1.1. Dãy so (thực) là m®t hàm so xác định trên t p con của t p so tự nhiên. Với M ⊂ N, thay cho ký hi u u : M → R n ›→ u(n) ta thường dùng ký hi u (un) hay {un} với n ∈ M. Dãy so được goi là vô hạn neu chúng có vô hạn phan tả. Dãy so được goi là hũu hạn neu so phan tả của dãy là hǎu hạn. Phan tả ui được goi là phan tả thá i của dãy. 1.1.1. Dãy so đơn đi u Dãy (un) được goi là đơn đi u tăng neu un ≤ un+1, với moi n = 1, 2, . . . Dãy (un) được goi là đơn đi u giảm neu un ≥ un+1, với moi n = 1, 2, . . . Dãy (un) được goi là tăng thực sự neu un < un+1, với moi n = 1, 2, . . . Dãy (un) được goi là giảm thực sự neu un > un+1, với moi n = 1, 2, . . . Dãy đơn đi u tăng và dãy đơn đi u giảm được goi chung là dãy đơn đi u.
  • 7. 4 Viết đề tài giá sinh viên – ZALO:0973.287.149-TEAMLUANVAN.COM Nh n xét 1.1. • Neu dãy (xn) tăng, dãy (yn) tăng thì dãy (xn + yn) tăng. • Neu dãy (xn) giảm, dãy (yn) giảm thì dãy (xn + yn) giảm. • Neu dãy (xn) tăng thì dãy (−xn) giảm, và neu dãy (xn) giảm thì dãy (−xn) tăng. • Neu hai dãy so dương (xn), (yn) cùng tăng (giảm) thì dãy (xnyn) tăng (giảm). • M®t dãy so có the không tăng, cũng không giảm. Ví dụ dãy so (xn) với xn = (−1)n, ∀n ∈ N. 1.1.2. Dãy so bị ch n Dãy (un) được goi là bị ch n trên neu ton tại m®t so M sao cho un ≤ M, ∀n ∈ N∗ . Dãy (un) được goi là bị ch n dưới neu ton tại m®t so m sao cho un ≥ m, ∀n ∈ N∗ . Dãy (un) được goi là bị ch n neu nó vàa bị ch n trên và vàa bị ch n dưới nghĩa là ton tại m®t so M và m®t so m sao cho m ≤ un ≤ M, ∀n ∈ N∗ . 1.1.3. Dãy so Cauchy Định nghĩa 1.2 (xem [5]). Dãy so (un) được goi là dãy Cauchy neu ∀ε > 0, ∃N0 ∈ N : ∀m, n > N0, |un − um| < ε. Định lj 1.1 (Tiêu chuȁn Cauchy, xem [5]). Dãy so (un) có giới hạn hǎu hạn khi và chỉ khi nó là dãy Cauchy. 1.1.4. Dãy so tuan hoàn Định nghĩa 1.3 (xem [3]). Dãy so (un) được goi là m®t dãy so tuan hoàn (c®ng tính) neu ton tại so nguyên dương l sao cho un+l = un, ∀n ∈ N. (1.1)
  • 8. 5 Viết đề tài giá sinh viên – ZALO:0973.287.149-TEAMLUANVAN.COM So nguyên dương l nhỏ nhat đe dãy (un) thỏa mãn (1.1) được goi là chu kỳ cơ sở của dãy. Dãy so (un) được goi là m®t dãy phản tuan hoàn (c®ng tính) neu ton tại so nguyên dương l sao cho un+l = −un, ∀n ∈ N. (1.2) So nguyên dương l nhỏ nhat đe dãy (un) thỏa mãn (1.2) được goi là chu kỳ cơ sở của dãy. Nh n xét 1.2. a) Dãy tuan hoàn chu kỳ 1 khi và chỉ khi dãy đó là m®t dãy hang. b) Dãy phản tuan hoàn chu kỳ l là dãy tuan hoàn chu kỳ 2l. Tương tự, ta cũng có định nghĩa ve dãy tuan hoàn nhân tính. Định nghĩa 1.4 (xem [3]). Dãy so (un) được goi là m®t dãy tuan hoàn nhân tính neu ton tại so nguyên dương s (s > 1) sao cho usn = un, ∀n ∈ N. (1.3) So nguyên dương s nhỏ nhat đe dãy so (un) thỏa mãn (1.3) được goi là chu kỳ cơ sở của dãy. Dãy so (un) được goi là dãy phản tuan hoàn nhân tính neu ton tại so nguyên dương s (s > 1) sao cho usn = −un, ∀n ∈ N. (1.4) So nguyên dương s (s > 1) nhỏ nhat đe dãy so (un) thỏa mãn (1.4) được goi là chu kỳ cơ sở của dãy. Nh n xét 1.3. Dãy phản tuan hoàn nhân tính chu kỳ s là m®t dãy tuan hoàn nhân tính chu kỳ s2. 1.2 Gi i hạn của dãy so Định nghĩa 1.5 (xem [5]). Ta nói dãy so (un) có giới hạn hǎu hạn a khi n dan tới vô cùng neu với moi ε > 0, ton tại so tự nhiên N0 (phụ thu®c vào dãy so un và ε ) sao cho với moi n > N0 ta có |un − a| < ε. lim n→+∞ un = a ⇔ ∀ε > 0, ∃N0 ∈ N : ∀n > N0, |un − a| < ε.
  • 9. 6 Viết đề tài giá sinh viên – ZALO:0973.287.149-TEAMLUANVAN.COM — n — n →+∞ n Ta nói dãy so (un) dan đen vô cùng khi n dan đen vô cùng neu với moi so thực dương M lớn tùy ý, ton tại so tự nhiên N0 (phụ thu®c vào dãy so un và M) sao cho với moi n > N0 ta có |un| > M. lim n→+∞ un = ∞ ⇔ ∀M > 0, ∃N0 ∈ N : ∀n > N0, |un| > M. Dãy so (un) có giới hạn hǎu hạn được goi là dãy h®i tụ. Dãy so không có giới hạn ho c dan đen vô cùng khi n dan đen vô cùng goi là dãy phân kỳ. Định lj 1.2 (xem [5]). Giả sả ton tại lim n→+∞ un = a; lim n→+∞ vn = b thì a) lim n→+∞ b) lim n→+∞ (un + vn) = lim n→+∞ (un vn) = lim →+∞ un + lim n→+∞ un lim →+∞ vn = a + b. vn = a − b. c) lim n→+∞ (un.vn) = lim n→+∞ un. lim n→+∞ vn = ab. lim u u n a d) neu b /= 0 thì lim n = →+∞ = . n→+∞ vn lim vn b n→+∞ Định lj 1.3. Neu un ≤ vn, ∀n ≥ N0, N0 ∈ N và ton tại n lim un = a; lim n→+∞ vn = b thì a ≤ b. Định lj 1.4 (Định lý Weierstrass, xem [5]). a) Neu dãy (un) đơn đi u tăng và bị ch n trên bởi M thì ton tại giới hạn hǎu hạn lim n→+∞ un = a và a ≤ M. b) Neu dãy (un) đơn đi u giảm và bị ch n dưới bởi m thì ton tại giới hạn hǎu hạn lim n→+∞ un = a và a ≥ m. Nói ngan gon hơn, m®t dãy so đơn đi u và bị ch n thì h®i tụ. Định lj 1.5 (Nguyên lý kep, xem [5]). Neu vn ≤ un ≤ wn, ∀n ≥ N0, N0 ∈ N và lim n→+∞ vn = lim n→+∞ wn = a thì lim n→+∞ un = a. 1.3 M t vài dãy so đ c bi t 1.3.1. Cap so c ng Định nghĩa 1.6 (xem [5]). Dãy so (un) được goi là m®t cap so c®ng khi và chỉ khi ton tại d ∈ R sao cho ∀n ∈ N, un+1 = un + d. u1 được goi là so hạng đau, d được goi là công sai của cap so c®ng.
  • 10. 7 Viết đề tài giá sinh viên – ZALO:0973.287.149-TEAMLUANVAN.COM k · · · ∀ ∈ Tính chat 1.1. Dãy so (un) là cap so c®ng với công sai d thì i) un = u1 + (n − 1)d với moi n = 1, 2, . . . ; ii) uk = uk−1 + uk+1 2 với moi k = 2, 3, . . . ; iii) Cho cap so c®ng hǎu hạn u1, u2, . . . , un−1, un. Ta có u1 + un = u2 + un−1 = u3 + un−2 = . . . M®t cách tőng quát: u1 + un = uk + un+1−k với moi k = 2, 3, . . . , n − 1. iv) Đ t Sn = u1 + u2 + · · · + un−1 + un. Ta có S = (u1 + un)n = [2u1 + (n − 1)d]n . n 2 2 1.3.2. Cap so nhân Định nghĩa 1.7 (xem [5]). Dãy so (un) được goi là m®t cap so nhân khi và chỉ khi ton tại q ∈ R sao cho ∀n ∈ N, un+1 = unq. u1 được goi là so hạng đau, q được goi là công b®i của cap so nhân. Tính chat 1.2. Dãy so (un) là cap so nhân với công b®i q thì i) un = u1qn−1 với moi n = 1, 2, . . . ; ii) u2 = uk−1.uk+1 với moi k = 2, 3, . . . ; iii) Đ t Sn = u1 + u2 + · · · + un−1 + un. Khi q = / u1(qn − 1) 1 ta có Sn = . q − 1 Nh n xét 1.4. Neu |q| < 1 thì (un) được goi là cap so nhân lùi vô hạn. Tőng của cap so nhân lùi vô hạn được tính theo công thác S = u1 1.3.3. Cap so đieu hòa + u2 + u3 + = u1 . 1 − q Định nghĩa 1.8 (xem [3]). Dãy so (un) (un 0 với moi n ∈ N) thỏa mãn đieu ki n un được goi là cap so đieu hòa. = 2un−1un+1 , n N∗ un−1 + un+1
  • 11. 8 Viết đề tài giá sinh viên – ZALO:0973.287.149-TEAMLUANVAN.COM n+1 2 1 − Bài toán 1.1. Cháng minh rang dãy (un) (un 0 với moi n ∈ N) l p thành m®t cap so đieu hòa khi và chỉ khi dãy đã cho thỏa mãn đieu ki n u = 1 , ∀n ∈ N∗ . Bài giai. Ta có un − un−1 1 u = ⇔ u = unun−1 n+1 2 1 n+1 2un−1 − un un − un−1 ⇔ un(un−1 + un+1) = 2un−1un+1 ⇔ un = 2un−1un+1 . un−1 + un+1 V y dãy so (un) l p thành m®t cap so đieu hòa. 1.3.4. Dãy so Fibonacci Định nghĩa 1.9 (xem [5]). Dãy so (un) xác định như sau u1 = 1, u2 = 1 un = un−1 + un−2, ∀n = 3, 4, . . . được goi là dãy so Fibonacci. Dãy Fibonacci có rat nhieu tính chat thú vị và xuat hi n m®t cách tự nhiên trong nhieu lĩnh vực khác nhau. Chúng ta có công thác sau đây đe tìm so hạng tőng quát của dãy so Fibonacci: Công thfíc Binet 1 + √ 5 n 1 − √ 5 n un = √ 5 . 1.3.5. Dãy so sinh b i hàm so Dãy so dạng xn+1 = f(xn). Đây là dạng dãy so thường g p nhat trong các bài toán ve giới hạn dãy so. Dãy so này sě hoàn toàn xác định khi biet giá trị ban đau x0. Do v y sự h®i tụ của dãy 2 2
  • 12. 9 Viết đề tài giá sinh viên – ZALO:0973.287.149-TEAMLUANVAN.COM m so sě phụ thu®c vào tính chat của hàm so f (x) và x0. M®t đ c điem quan trong của dãy so này là neu a là giới hạn của dãy so thì a phải là nghi m của phương trình x = f(x). Chúng ta có m®t so ket quả cơ bản như sau: Định nghĩa 1.10 (xem [5]). Hàm so f : D → D được goi là m®t hàm so co trên D neu ton tại so thực q, 0 < q < 1 sao cho |f(x) − f(y)| ≤ q. |x − y| với moi x, y thu®c D. Định lj 1.6 (xem [5]). Neu f (x) là m®t hàm so co trên D thì dãy so (xn) xác định bởi x0 = a ∈ D, xn+1 = f(xn) h®i tụ. Giới hạn của dãy so là nghi m duy nhat trên D của phương trình x = f(x). ChGng minh. Với moi n > m thì áp dụng định nghĩa hàm so co, ta có |xn − xm| = |f(xn−1) − f(xm−1)| ≤ q |xn−1 − xm−1| ≤ · · · ≤ q |xn−m − x0| (1.5) Tà đây |xn − x0| ≤ |xn − xn−1| + · · · + |x1 − x0| ≤ qn−1 + · · · + 1 |x1 − x0|, suy ra dãy (xn) bị ch n. Xét ε > 0. Tà (1.5), do q < 1 và |xn−m − x0| bị ch n nên ta suy ra ton tại N sao cho qN |xn−m − x0| < ε. Suy ra (xn) là dãy Cauchy và do đó nó h®i tụ. 1.3.6. Định lj trung bình Cesaro Định lj 1.7 (Định lý trung bình Cesaro, xem [5]). Neu lim n→+∞ xn = a thì lim n→+∞ x1 + x2 + · · · + xn n = a. Định lý này có the phát bieu dưới dạng tương đương như sau: "Neu lim n→+∞ (xn+1 — xn ) = a thì lim xn n→+∞ n = a."
  • 13. 10 Viết đề tài giá sinh viên – ZALO:0973.287.149-TEAMLUANVAN.COM n Σ − Chương 2 M t so phương pháp giải bài toán ve xác định dãy so Trong chương này ta xét các bài toán liên quan đen xác định so hạng tőng quát của dãy so sinh bởi các hàm sơ cap cơ bản đó là hàm đa thác, hàm phân thác hǎu t , hàm vô t , hàm lượng giác và các hàm siêu vi t. Các phương pháp chủ yeu sả dụng trong chương này là phương pháp ước lượng, phương pháp lượng giác và phương pháp sả dụng hàm l p. 2.1 Dãy so sinh b i hàm đa thfíc Bài toán 2.1. Cho a, c > 0. Xét dãy (an) xác định bởi: a1 = a an+1 = ca2 + an, ∀n ∈ N∗ . (2.1) Cháng minh rang Bài giai. Tà giả thiet ta có the viet an ≥ √ cn−1nnan+1, ∀n ∈ N∗ . 1 − 1 = can−1 . V y nên an−1 an an n−1 1 1 = a1 an k=1 cak . ak+1
  • 14. 11 Viết đề tài giá sinh viên – ZALO:0973.287.149-TEAMLUANVAN.COM Σ √ ‚ . , n n Suy ra 1 1 n−1 = + cak ≥ n r n cn−1 a1 . a1 an k=1 ak+1 2 Do đó an ≥ √ cn−1nnan+1, ∀n ∈ N∗ . Bài toán 2.2. Tìm so hạng tőng quát của dãy (un) xác định bởi: √ 3 u1 = 2 (2.2) Bài giai. Ta có u √ 3 π = = cos . un = 4u3 n−1 — 3un−1, ∀n ≥ 2. 1 2 6 Tà công thác lượng giác cos 3α = 4 cos3 α − 3 cos α, ta có u = 4 cos3 π − 3 cos π = cos 3π 2 6 3 3π 6 6 3π 32π u3 = 4 cos 3k−1π 6 − 3 cos ∗ = cos 6 6 Giả sả uk = cos 6 với k ∈ N . Khi đó uk+1 = 4 cos3 3k−1π 6 − 3 cos 3k−1π = cos 6 3kπ . 6 3n−1π ∗ V y bang phương pháp quy nạp ta cháng minh được un = cos Bài toán 2.3 (xem [1]). Cho dãy (xn) xác định bởi: 6 , ∀n ∈ N . x1 = 2 + √ 3 xn+1 = x4 − 4x2 + 2, ∀n ∈ N∗ . (2.3) n n Tìm so hạng tőng quát của dãy so (xn). Bài giai. Đ t xn = 2yn. Khi đó 2yn+1 = 16y4 − 16y2 + 2, ∀n = 1, 2, . . . 4 2 ⇔ yn+1 = 8yn − 8yn + 1, ∀n = 1, 2, . . . Ta có y1 = 2 + √ 3 = 2 1 + √ 3 2 = 2 1 + cos π 6 2 π = cos . 12 n a √ s
  • 15. 12 Viết đề tài giá sinh viên – ZALO:0973.287.149-TEAMLUANVAN.COM 3 12 12 12 k+1 12 12 12 1 12 Do công thác lượng giác cos 4α = 8 cos4 α − 8 cos2 α + 1 nên y = 8 cos4 π 2 12 2 π — 8 cos 12 + 1 = cos 4. π . 12 y = 8 cos4 4. π − 8 cos2 4. π + 1 = cos 42. π . Bang quy nạp ta sě cháng minh yn = cos 4n−1. π . (2.4) 12 Th t v y, (2.4) đã đúng với n = 2, n = 3 như trên. Giả sả (2.4) đúng với n = k, k ∈ N∗ tác là yk = cos 4k−1. π . 12 Khi đó y = 8 cos4 4k−1. π − 8 cos2 4k−1. π + 1 = cos 4k. π . Suy ra (2.4) đúng với n = k + 1. V y theo nguyên lý quy nạp ta có yn = cos 4n−1. π , ∀n = 1, 2, . . . Do đó xn = 2 cos 4n−1. , ∀n = 1, 2, . . . π Bài toán 2.4 (xem [1]). Cho dãy (xn) xác định bởi: x1 = 2 (2.5) xn+1 = 16x5 − 20x3 + 5xn, ∀n ∈ N∗ . Tìm so hạng tőng quát của dãy so (xn). Bài giai. Ta có x 1 π = = cos . 1 2 3 Tà công thác cos 5α = 16 cos5 α − 20 cos3 α + 5 cos α, ta có x = 16 cos5 π − 20 cos3 π + 5 cos π = cos 5π . 2 3 5 5π 3 3 5π 3 3 5π 52π Giả sả xn = cos x3 = 16 cos 5n−1π . Khi đó 3 3 − 20 cos + 5 cos 3 = cos . 3 3 xn+1 = 16 cos5 5n−1π 3 − 20 cos 5n−1π + 5 cos 3 5n−1π 5n−1π = cos 3 5nπ . 3 Theo nguyên lý quy nạp, suy ra xn = cos 3 , ∀n = 1, 2, . . . n n 12 3
  • 16. 13 Viết đề tài giá sinh viên – ZALO:0973.287.149-TEAMLUANVAN.COM r − r n Bài toán 2.5 (Đe thi hoc sinh giỏi Quoc gia 2014). Cho hai dãy so dương (xn), (yn) xác định bởi: x1 = 1, y1 = √ 3 và xn+1yn+1 − xn = 0 (2.6) Cháng minh rang hai dãy so trên h®i tụ và tìm giới hạn của chúng. Bài giai. Ta nh n thay π x1 = 1 = 2 sin 6 ; y = √ 3 = 2 cos π . 1 6 Ta sě cháng minh bang quy nạp rang với moi n nguyên dương thì π π xn = 2 sin 3.2n , yn = 2 cos 3.2n . (2.7) Th t v y, với n = 1 m nh đe (2.7) đúng. Giả sả đã có xn π = 2 sin 3.2n , yn π = 2 cos 3.2n . Theo công thác truy hoi ta có xn+1 = √ 2 − yn xn = 2 2 cos π 3.2n π 2 sin 3.2n = 4 sin2 π 3.2n+1 π π = 2 sin 3.2n+1 ; yn+1 = xn+1 = π 2 sin 3.2n+1 = 2 cos 3.2n+1 . Đieu này cháng tỏ (2.7) đúng với n + 1. Theo nguyên lý quy nạp ta có π π xn = 2 sin 3.2n , yn = 2 cos 3.2n , ∀n = 1, 2, . . . Do đó lim xn n→+∞ = lim n→+∞ 2 sin π 3.2n = 0 và lim yn →+∞ = lim n→+∞ 2 cos π 3.2n = 2. V y các dãy (xn), (yn) có giới hạn hǎu hạn và lim n→+∞ xn = 0 và lim n→+∞ yn = 2. Nh n xét 2.1. Đây là bài toán de nhat của kỳ thi. Tuy nhiên, neu không nh n xét được tính chat đ c bi t của x1, y1 thì rat khó khăn đe giải bài toán này bang phương pháp lượng giác. 2 n+1 x + yn = 2, ∀n = 1, 2, . . .
  • 17. 14 Viết đề tài giá sinh viên – ZALO:0973.287.149-TEAMLUANVAN.COM 6 6 6 6 6 6 n Tiep sau đây ta sě xét m®t so bài toán xác định dãy so bang phương pháp sả dụng hàm l p. Đe tìm so hạng tőng quát của dãy so (un) bang phương pháp hàm l p ta thường tìm các hàm so f(x) và h(x) sao cho f(un+1) = h(f(un)) (2.8) Sả dụng (2.8) liên tiep ta thu được f(un+1) = h(f(un)) = h(h(f(un−1))) := h2(f(un−1)) = · · · = hn(f(u1)). (2.9) Tà (2.9) ta tìm được un. Hàm so f được goi là hàm phụ, còn hàm so h được goi là hàm l p. Bài toán 2.6. Tìm so hạng tőng quát của dãy (xn) cho như sau x1 = 3, xn+1 = 7xn − 1, ∀n = 1, 2, . . . Bài giai. Ta có V y xn+1 1 — 6 = 7xn 1 — 1 − 6 = 7xn — 7 = 7 x — 1 , ∀n = 1, 2, . . . x − 1 = 7 xn−1 — 1 = 72 xn−2 — 1 Do đó x = · · · = 7n−1 x1 = 17 .7n−1. 6 = 17 .7n−1 + 1 , ∀n = 1, 2, . . . — 1 Nh n xét 2.2. Trong lời giải trên, đieu quan trong là phải biet xét hi u x Ta có n+1 1 — 6 . xn+1 − k = 7xn − 1 − k = 7 xn − k + 6k − 1. Ta can chon k sao cho 6k −1 = 0 ⇔ k = 1 . V y ta sě xét x n+1 1 — 6 . Có the thay rang dãy so đã cho có dạng x phương trình f(x) = x. n+1 = f(xn 1 ), trong đó f(x) = 7x − 1, và so 6 là nghi m của 6 6 6 n n
  • 18. 15 Viết đề tài giá sinh viên – ZALO:0973.287.149-TEAMLUANVAN.COM n 2 2 3 n n Bài toán 2.7 (xem [1]). Cho dãy (un) như sau u1 = α ∈ R, un+1 = −5u2 − 4un 6 — 5 , ∀n = 1, 2, . . . Tìm so hạng tőng quát của dãy so đã cho. Bài giai. Đ t un 1 = − 5 xn . Khi đó 1 1 2 4 6 — 5 xn+1 = − 5 xn + 5 xn − 5 ⇔xn+1 = xn − 4xn + 6 ⇔xn+1 − 2 = (xn − 2) . V y xn − 2 = 2 xn−1 − 2 = xn−2 − 2 22 = · · · = x1 − 2 2n−1 . So hạng tőng quát của dãy đã cho là un = − h 2 + (5α + 2)2 i , ∀n ∈ N∗ . 1 n−1 Bài toán 2.8 (xem [1]). Cho dãy (un) như sau u1 = α ∈ R, un+1 = 25u3 − 15u2 + 3un, ∀n ∈ N∗ . Tìm so hạng tőng quát của dãy so đã cho. Bài giai. Đ t un = xn . Khi đó ta được dãy (x 5 ) thỏa mãn x1 = 5α và 1 x = 1 x3 − 3 x2 + 3 x 5 n+1 5 n 5 n 5 n 3 2 ⇔xn+1 = xn − 3xn + 3xn ⇔xn+1 − 1 = (xn − 1) . V y với moi so nguyên dương n, ta có xn − 1 = 3 xn−1 − 1 = xn−2 − 1 32 = · · · = x1 − 1 3n−1 = 5α − 1 3n−1 . So hạng tőng quát của dãy đã cho là 3n−1 un = 5α − 1 + 1 5 , ∀n ∈ N∗ . 5 n
  • 19. 16 Viết đề tài giá sinh viên – ZALO:0973.287.149-TEAMLUANVAN.COM 2 2n n Bài toán 2.9. Xét dãy (an) thỏa mãn các đieu ki n sau 0 < an < 1 (2.10) Cháng minh rang an+1 (1 − an ) ≥ 1 , ∀n ∈ N∗ . 1 1 — < a ≤ 1 , ∀n ∈ N∗ . Bài giai. 2 2n n 2 Bang phương pháp quy nạp ta cháng minh 1 − 1 < a . Th t v y, ta có a = 1 − 1 (đúng). Giả sả 1 − 1 < a . Suy ra 1 2 2 2 2k k 1 1 k + 1 Tà giả thiet 1 − ak < 2 + 2k = 2k . 1 suy ra ak+1(1 − ak) ≥ 4 , 1 2k 1 1 ak+1 > 4(1 − a (đieu phải cháng minh). ) > 4(k + 1) = 2 − 2(k + 1) Đe cháng minh bat đȁng thác an 1 ≤ 2 , ta sả dụng h thác 1 an(1 − an) ≤ 4 ≤ an+1(1 − an). Suy ra an ≥ an+1 và dãy (an 1 ) bị ch n trên bởi a với a(1 − a) = 4 . Do đó a = 1 2 và an 1 ≤ 2 . 2.2 Dãy so sinh b i hàm phân thfíc hfiu t Bài toán 2.10 (Đe thi hoc sinh giỏi Quoc gia 2001, bảng B). Cho dãy (xn) xác định bởi: 2 x1 = 3 xn (2.11) xn+1 = 2(2n + 1)x + 1 , ∀n = 1, 2, . . . Hãy tính tőng của 2001 so hạng đau tiên của dãy so (xn). n 4 k
  • 20. 17 Viết đề tài giá sinh viên – ZALO:0973.287.149-TEAMLUANVAN.COM 1 2 2001 1 3 1 3 4002 5 4001 4003 xn+1 = 1 + (1 − √ 2)x , ∀n ∈ N 1 = tan = tan 4 + 8 8 2 π 1 − tan 8 2 √ Bài giai. De thay xn > 0, ∀n = 1, 2, . . . Tà (2.11), ta có xn+1 = 1 1 2(2n + 1) + xn 1 ⇔ xn+1 1 = 2(2n + 1) + . xn Đ t 2 = u xn n. Khi đó u1 = 3 và un+1 = 4(2n + 1) + un , ∀n = 1, 2, . . . Suy ra un+1 − 4(n + 1)2 − 1 = un − 4n2 − 1 = · · · = u1 − (4.12 − 1) = 0 Do đó un = (2n − 1)(2n + 1), ∀n = 1, 2, . . . V y x 2 2 1 1 = = = − , ∀n = 1, 2, . . . n un nên (2n − 1)(2n + 1) 2n − 1 2n + 1 x + x + · · · + x = 1 − 1 + 1 − 1 + · · · + 1 − 1 ⇔x1 + x2 + · · · + x2001 = 1 − 4003 = 4003 . Bài toán 2.11 (xem [1]). Cho dãy (xn) xác định bởi: x1 = √ 3 xn + √ 2 − 1 ∗ (2.12) Tìm so hạng tőng quát của dãy (xn). Bài giai. Tà công thác tan (x + y) = tan x + tan y 1 − tan x tan y , ta có π π π π 2 tan 8 π π tan π = −1 + √ 2 ⇔ tan + 2 tan 8 8 — 1 = 0 ⇔ π 8 tan 8 = −1 − 2 Vì tan π > 0 nên tan π = √ 2 − 1. 8 8 π Ta có xn+1 = xn + tan 8 π , ∀n = 1, 2, . . . 1 − xn. tan 8 n . =
  • 21. 18 Viết đề tài giá sinh viên – ZALO:0973.287.149-TEAMLUANVAN.COM − ∀ − − − π π + 3 8 1 − tan 3 + (n − 1) 8 . tan 8 3 8 8 3 8 n 3 8 n n Do đó x = √ 3 = tan π . 1 3 π π tan + tan 3 8 π π 1 − tan 3 tan 8 tan π + π + tan π 3 8 8 π π x3 = π π π = tan + 2. . 3 8 Bang phương pháp quy nạp, ta sě cháng minh x = tan π +(n−1) π , ∀n = 1, 2, . . . Trường hợp n = 1 đã kiem tra ở trên. Giả sả x = tan π + (n − 1) π . Khi đó n 3 8 n 3 8 tan π + (n 1) π + tan π 3 8 8 xn+1 = π π π = tan hπ + (n − 1) π + π i = tan hπ + n. π i . Theo nguyên lý quy nạp, ta được x = tan hπ + (n − 1) π i , ∀n = 1, 2, . . . Bài toán 2.12 (xem [1]). Cho dãy (xn) như sau x1 = 5, xn+1 = 5xn + 4 , n = 1, 2, . . . xn + 2 Cháng minh rang với moi n ∈ N∗ thì xn /= 4. Tìm so hạng tőng quát của dãy (xn). Bài giai. Ta có x1 = 5 /= 4. Giả sả xn /= 4, ta cháng minh xn+1 /= 4. Neu xn+1 = 4 thì 5xn + 4 xn + 2 = 4 ⇔ 5xn + 4 = 4xn + 8 ⇔ xn = 4, mâu thuan với giả thiet quy nạp. V y xn = / 4, ∀n ∈ N∗ . Ta có x − 4 = 5xn−1 + 4 − 4 = xn−1 − 4 ; xn−1 + 2 xn−1 + 2 x + 1 = 5xn−1 + 4 + 1 = 6(xn−1 + 1) . xn−1 + 2 xn−1 + 2 Suy ra xn + 1 = 6 xn−1 + 1 = 62. xn−2 + 1 = · · · = 6n−1. x1 + 1 = 6n. xn − 4 xn−1 − 4 n n xn 2 4 4.6n + 1 x1 − 4 ∗ Do đó xn + 1 = 6 xn − 4.6 ⇔ xn = 6n 1 , ∀n ∈ N . 4.6n + 1 ∗ V y so hạng tőng quát của dãy (xn) là xn = 6n − 1 , ∀n ∈ N . x2 = = tan . 1 − tan + 3 8 tan 8
  • 22. 19 Viết đề tài giá sinh viên – ZALO:0973.287.149-TEAMLUANVAN.COM n ∀ ∈ N n−1 i (5 − k) x − 2k − 4 = 2 − Nh n xét 2.3. Đe sả dụng phương pháp hàm l p ta xét x − k = 5xn + 4 − k = 5xn − 4 − kxn − 2k n+1 xn + 2 xn + 2 Ta chon k sao cho = (5 − k)xn + 4 − 2k = xn + 2 n xn + 2 5 − k . 2k − 4 = k ⇔ 2k − 4 = 5k − k2 ⇔ k2 − 3k − 4 = 0 ⇔ " k = −1 5 − k V y nên trong lời giải trên ta đã xét xn − 4 và xn + 1. k = 4 Bài toán 2.13 (xem [1]). Tìm so hạng tőng quát của dãy so (xn) cho như sau Bài giai. x1 = α ∈ R, xn+1 = 8xn , n ∗. 4 + x2 Neu α = −2 thì xn = −2, ∀n = 1, 2, . . . Xét α /= −2. Ta có 2 − xn = 2 − 8xn−1 = 4 + x2 n−1 2x2 − 8xn−1 + 8 4 + x2 n−1 2(2 − xn−1)2 n−1 (1); 2 + xn = 2 + 8xn−1 = 2x2 n−1 + 8xn−1 + 8 2(2 + xn−1)2 (2) 4 + x2 n−1 4 + x2 n−1 4 + x2 n−1 Xét hàm so f(x) = 2 − x . Tà (1) và (2) ta có 2 + x f(x ) = 2 − xn = n 2 + xn 2 − xn−1 2 2 + xn−1 = h f(xn−1) i h i2n−1 = h f(xn−2) i22 = · · · = h f(x1) i2n−1 = h f(α) 2n−1 . 2 − xn 2 + xn = β ⇔ 2 − xn = 2β + βxn ⇔ xn = 2 − 2β . 1 + β V y, neu α = −2 thì xn = −2, ∀n = 1, 2, . . . 2 h 1 − 2 − α 2n−1 2 + α neu α −2 thì xn = 2 α 2n−1 , ∀n = 1, 2, . . . 1 + 2 + α . Ta có f(α) Đ t β = 4 + x = 2 i
  • 23. 20 Viết đề tài giá sinh viên – ZALO:0973.287.149-TEAMLUANVAN.COM n − 2 f(xn) = x = + 2 = f(xn−1) n n n n n Bài toán 2.14 (xem [1]). Tìm so hạng tőng quát của dãy so (xn) cho như sau x3 + 12xn Bài giai. Ta có x1 = α > 0, xn+1 = n 3x2 + 4 , ∀n ∈ N∗ . x3 −1 + 12xn−1 x3 −1 − 6x2 −1 + 12xn−1 − 8 (xn−1 − 2)3 xn − 2 = n 3x2 + 4 n−1 — 2 = n 3x2 + 4 n−1 = 3x2 + 4 n−1 . (2.13) xn + 2 = x3 −1 + 12xn−1 + 2 = x3 n−1 + 6x2 n−1 + 12xn−1 + 8 (xn−1 + 2)3 . (2.14) 3x2 + 4 n−1 3x2 + 4 n−1 3x2 + 4 n−1 Xét hàm so f(x) = x − 2 . Tà (2.13) và (2.14) ta có x + 2 xn − 2 xn−1 − 2 3 h i3 Tà (2.15) ta có = h f(xn−2) i32 = · · · = h f(x1) i3n−1 = h f(α) i3n−1 . (2.15) xn − 2 xn + 2 . = h f(α) i3n−1 ⇔ xn − 2 = h f(α) i3n−1 .xn + 2. h f(α) i3n−1 2 + 2 h f(α) i3n−1 2 + 2 α − 2 3n−1 α + 2 ⇔ xn = 1 − h f(α) 3n−1 ⇔ xn = 1 − α − 2 3n−1 , ∀n = 1, 2, . . . α + 2 α − 2 3n−1 V y so hạng tőng quát của dãy đã cho là xn = 2 + 2 α + 2 , ∀n = 1, 2, . . . α − 2 3n−1 α + 2 Bài toán 2.15 (xem [1]). Tìm so hạng tőng quát của dãy so (un) cho như sau: 4un(4u2 + 1) u1 = α ∈ R, un+1 = n 16u4 + 24u2 + 1 , ∀n ∈ N∗ . n n Bài giai. De thay, với moi so nguyên dương n thì luôn ton tại un. Neu α = 1 2 thì un 1 = − 2 , ∀n = 1, 2, . . . Xét α /= − 1 . Ta có 32u3 + 8un 2un+1 + 1 = 16u4 + 32u3 + 24u2 + 8un + 1 + 1 = 16u4 + 24u2 + 1 16u4 + 24u2 + 1 n n n n + 2 n−1 x n n = i 1 −
  • 24. 21 Viết đề tài giá sinh viên – ZALO:0973.287.149-TEAMLUANVAN.COM − 2 n n − 1 4 2α− 1 n n ∀ ∈ N∗ 1 n+1 n n n n n n n n − n 16u4 + 24u2 + 1 16u4 + 24u2 + 1 f(un) = 2u + 1 = − = f(un−1) n 2un + 1 n n n n √ √ √ √ 4n−1 . (2un + 1)4 = 16u4 + 24u2 + 1 . (2.16) M t khác n n 32u3 + 8un −16u4 + 32u3 − 24u2 + 8un − 1 2un+1 − 1 = − 1 = n n n n −(2un + 1)4 = 16u4 + 24u2 + 1 . (2.17) n n Xét hàm so f(x) = 2x − 1 . Tà (2.16)và (2.17) ta có 2x + 1 2un − 1 2un−1 − 1 4 h i4 Tà (2.18) ta có = − hf(un−2) i42 = · · · = − h f(u1) i4n−1 = − hf(α) i4n−1 . (2.18) 2u − 1 h i4n−1 1 − h f(α) i4n−1 V y, neu α = 1 2 thì un 1 = − 2 , ∀n = 1, 2, . . . 2α − 1 4n−1 neu α /= − 1 thì u = 1 − 2 + 2 2α + 1 , ∀n = 1, 2, . . . 2α + 1 Bài toán 2.16 (xem [1]). Tìm so hạng tőng quát của dãy so (xn) cho như sau x4 + 12x2 + 4 x = α > 0, x = , n . 4x3 + 8xn Bài giai. De thay xn > 0, ∀n ∈ N∗ . Ta có xn+1 + x4 + 12x2 + 4 2 = + 2 4x3 + 8xn x4 + 4 √ 2x3 + 12x2 + 8 √ 2xn + 4 (xn + √ 2)4 = n n n = . (2.19) M t khác 4x3 + 8xn 4x3 + 8xn xn+1 − x4 + 12x2 + 4 2 = 2 4x3 + 8xn 2 + 2 f(α) = − + 1 n−1 2u n ⇔ un = h f(α) i
  • 25. 22 Viết đề tài giá sinh viên – ZALO:0973.287.149-TEAMLUANVAN.COM n = n n n = n − − − r q 1 − u 6 r q − − r − x4 − 4 √ 2x3 + 12x2 − 8 √ 2xn + 4 4x3 + 8xn (xn − √ 2)4 4x3 + 8xn . (2.20) x √ 2 Xét hàm so f(x) = x + √ 2 , ∀x > 0. Tà (2.19) và (2.20) ta suy ra xn − √ 2 xn−1 − √ 2 4 h i4 f(xn) = xn h + √ 2 = i42 xn−1 h + √ 2 i4n−1 = f(xn−1) h i4n−1 = f(xn−2) = · · · = f(x1) h i4n−1 = f(α) . (2.21) Đ t βn = f(α) . Tà (2.21) ta có xn − √ 2 √ √ √ 2 + √ 2βn x + √ 2 = βn ⇔ xn − 2 = βnxn + 2βn ⇔ xn = . 1 − βn So hạng tőng quát của dãy đã cho là √ 2 + √ 2 α √ 2 4n−1 α + √ 2 xn = α √ 2 4n−1 , ∀n ∈ N∗ . 1 − α + √ 2 2.3 Dãy so sinh b i hàm chfía căn thfíc Bài toán 2.17. Tìm so hạng tőng quát của dãy (un) xác định bởi: 1 u1 = Bài giai. un = 2 2 − 2 2 n−1 2 , ∀n ≥ 2. (2.22) Ta có u 1 π = = sin . 1 2 6 Tà công thác lượng giác cos2 α = 1 − sin2 α ta có r 2 − 2 q 1 − sin2 π r 2 1 − cos π Giả sả u u2 = 2 = π = sin . = sin . n 6 π
  • 26. 23 Viết đề tài giá sinh viên – ZALO:0973.287.149-TEAMLUANVAN.COM 2 2 .6 n 2n−1.6 2 2 1 sin2 π 2n−1.6 2 1 cos π 2n−1 .6 π Khi đó un+1 = 2 = 2 = sin 2n.6 . V y, theo nguyên lý quy nạp ta cháng minh được un π = sin 2n−1.6 .
  • 27. 24 Viết đề tài giá sinh viên – ZALO:0973.287.149-TEAMLUANVAN.COM n n+1 3 23 24 k+1 k 2k+1 2k+2 Bài toán 2.18. Tìm so hạng tőng quát của dãy (un) xác định bởi: Bài giai. Ta có u = √ 2 = 2 cos π ; 1 22 u1 = √ 2 un+1 = √ 2 + un, ∀n ≥ 1. (2.23) u = √ 2 + √ 2 = r 2 1 + √ 2 = r 2 1 + cos π = 2 cos π ; 2 u = √ 2 + u 2 4 23 = r 2 1 + cos π = 2 cos π . Giả sả uk π = 2 cos 2k+1 với k ≥ 1. Khi đó u = √ 2 + u = r 2 1 + cos π = 2 cos π . V y, theo nguyên lý quy nạp ta cháng minh được un Bài toán 2.19. Cho dãy (xn) xác định bởi: π = 2 cos 2n+1 , ∀n ≥ 1. 1 x1 = 4 xn ∗ (2.24) xn+1 = 1 + 2xn Σ n + 2 √ x2 + 2xn , ∀n ∈ N . Bài giai. Tà giả thiet ta có xn > 0, ∀n ∈ N∗ . Đ t 1 xn = un . Tà công thác (2.24), ta có u1 = 4 và un+1 = un + 2 + 2 √ 1 + 2un, ∀n ∈ N∗ . (2.25) Nhân hai ve của (2.25) với 8, roi c®ng hai ve với 4 ta được 4 + 8un+1 = 4 + 8un + 16 + 16 √ 1 + 2un, ∀n ∈ N∗ . (2.26) Đ t vn = 2 √ 1 + 2un, vn > 0. Tà (2.26) suy ra v2 = (vn + 4)2, ∀n ∈ N∗ . Do vn > 0, ∀n ∈ N∗ nên vn+1 = vn + 4, ∀n ∈ N∗ . V y (vn) là cap so c®ng với công sai d = 4, so hạng đau v1 = 6. k=1 2 Đ t yn = xk. Tìm so hạng tőng quát của dãy (yn).
  • 28. 25 Viết đề tài giá sinh viên – ZALO:0973.287.149-TEAMLUANVAN.COM n k 2 n + 1 n n 2 n + 1 k=1 Do đó vn = 6 + (n − 1)4 = 2 + 4n ⇒ un = 2(n2 + n), ∀n ∈ N∗ . Suy ra x = 1 = 1 1 − 1 , ∀n ∈ N∗ . Do đó n 2n(n + 1) 2 n n + 1 n y = Σ x = 1 1 − 1 , ∀n ∈ N∗ . V y so hạng tőng quát của dãy (y ) là y = 1 1 − 1 , ∀n ∈ N∗ . 2.4 Dãy so sinh b i các hàm lư ng giác và siêu vi t 2.4.1. Dãy so sinh b i các hàm lư ng giác Bài toán 2.20. Cho dãy so (xn) xác định như sau: x0 = a xn+1 = xn + sin xn, ∀n ∈ N. (2.27) Cháng minh rang với moi so thực a dãy (xn) có giới hạn hǎu hạn khi n → +∞. Bài giai. Trường hợp 1. Với a = kπ, k ∈ Z. Tà công thác xác định dãy ta có xn = a, ∀n ∈ N. Do đó dãy đã cho có giới hạn hǎu hạn khi n → +∞ và lim xn = a Trường hợp 2. Với a /= kπ, k ∈ Z. Xét hàm so f(x) = x + sin x, x ∈ R. Khi đó dãy (xn ) được viet lại dưới dạng x0 = a xn+1 = f(xn), ∀n ∈ N. Ta có fJ(x) = 1 + cos x ≥ 0, ∀x ∈ R, suy ra f(x) đong bien trên R và do đó dãy (xn) đơn đi u. Ta xét các khả năng sau: i) Neu a ∈ (2kπ; (2k + 1)π), k ∈ Z thì sin a > 0 nên dãy (xn) đơn đi u tăng. Bang quy nạp ta cháng minh xn ∈ (2kπ; (2k + 1)π), n ∈ N. Th t v y, với n = 0 hien nhiên x0 ∈ (2kπ; (2k + 1)π). Giả sả đã có xn ∈ (2kπ; (2k + 1)π), n ∈ N. Do f(x) đong bien trên R nên 2kπ = f(2kπ) < f(xn) = xn+1 < f((2k + 1)π) = (2k + 1)π
  • 29. 26 Viết đề tài giá sinh viên – ZALO:0973.287.149-TEAMLUANVAN.COM nghĩa là có xn+1 ∈ (2kπ; (2k + 1)π). Theo nguyên lý quy nạp ta có xn ∈ (2kπ; (2k + 1)π), n ∈ N. Do dãy (xn) đơn đi u tăng và bị ch n trong khoảng (2kπ; (2k + 1)π) nên dãy (xn) có giới hạn hǎu hạn khi n → +∞. ii) Neu a ∈ ((2k − 1)π; 2kπ), k ∈ Z thì sin a < 0 nên dãy (xn) đơn đi u giảm. Tương tự trường hợp i), bang quy nạp ta cháng minh được xn ∈ ((2k − 1)π; 2kπ), n ∈ N. Do dãy (xn) đơn đi u giảm và bị ch n trong khoảng ((2k − 1)π; 2kπ) nên dãy (xn) có giới hạn hǎu hạn khi n → +∞. 2.4.2. Dãy so sinh b i hàm siêu vi t Bài toán 2.21. Tìm so hạng tőng quát của dãy (xn) xác định bởi: Bài giai. Đ t yn = xn − 3n, ta được x0 = 8 xn+1 = 2xn + 3n, ∀n ∈ N. (2.28) y0 = 7 yn+1 + 3.3n = 2(yn + 3n) + 3n ⇔ y0 = 7 yn+1 = 2yn. Tà đó ta có (yn) là cap so nhân với so hạng đau y0 = 7, công b®i q = 2. Theo công thác tőng quát của cap so nhân ta được yn = 7.2n, suy ra công thác so hạng tőng quát của dãy (xn) là xn = 7.2n + 3n. Bài toán 2.22. Tìm so hạng tőng quát của dãy (xn) xác định bởi: Bài giai. Đ t xn = yn.7n, ta được x0 = 101 xn+1 = 7xn + 7n+1, ∀n ∈ N. (2.29) x0 = y0.70 yn+1.7n+1 = 7.yn.7n + 7n+1 ⇔ y0 = 101 yn+1 = yn + 1.
  • 30. 27 Viết đề tài giá sinh viên – ZALO:0973.287.149-TEAMLUANVAN.COM n−1 n n n+1 n 1 Tà đó ta có (yn) là cap so c®ng với so hạng đau y0 = 101, công sai d = 1. Theo công thác tőng quát của cap so c®ng ta được yn = 101 + n, suy ra công thác so hạng tőng quát của dãy (xn) là xn = (101 + n)7n. Bài toán 2.23 (Đe thi hoc sinh giỏi Quoc gia 2010). Cho dãy (an) xác định bởi: a1 = 5 (2.30) an = q n an−1 + 2n−1 + 2.3n−1, ∀n ≥ 2. a) Tìm so hạng tőng quát của dãy (an). b) Cháng minh rang (an) là dãy so giảm. Bài giai. a) Tà công thác (2.30) ta có an = an−1 + 2n−1 + 2.3n−1 n n−1 Thay lan lượt n bởi n − 1, n − 2, . . . , 2 ta được an−1 = an−2 + 2n−2 + 2.3n−2 n−1 n−2 . . . a2 = a1 + 21 + 2.31. 2 1 C®ng các đȁng thác trên theo ve và giản ước các so hạng bang nhau ở hai ve ta được an = a1 + Σ (2k−1 + 2.3k−1) = 5 + 2n − 2 + 3n − 3 = 2n + 3n. Suy ra an = √ n 2n + 3n, ∀n ≥ 2. b) Ta có an+1 = √ n 2n + 3n(2n + 3n) > √ n 3n(2n + 3n) = 3.2n + 3n+1 > 2n+1 + 3n+1 = an+1, ∀n = 2, 3, . . . V y an > an+1, ∀n ≥ 2 nên (an) là dãy so giảm. Bài toán 2.24. Cháng minh rang neu dãy so (un) là m®t cap so nhân với các so hạng dương thì dãy so (vn) với vn = loga un, ∀n ∈ N, 0 < a /= 1 sě l p thành m®t cap so c®ng. k=2
  • 31. 28 Viết đề tài giá sinh viên – ZALO:0973.287.149-TEAMLUANVAN.COM Bài giai. Giả sả (un) là m®t cap so nhân với công b®i q. Xét dãy so (vn) với vn = loga un, ∀n ∈ N, 0 < a = / 1. Ta có Khi đó v0 = loga u0, v1 = loga u1, v2 = loga u2, . . . , vn = loga un. v1 − v0 = v2 − v1 = v3 − v2 = · · · = vn − vn−1 = loga q. V y, dãy so (vn) l p thành m®t cap so c®ng với công sai bang loga q.
  • 32. 29 Viết đề tài giá sinh viên – ZALO:0973.287.149-TEAMLUANVAN.COM ∀ n+1 Chương 3 M t so phương pháp xác định gi i hạn của dãy so 3.1 Sfi dnng tính đơn đi u và bị ch n đe tính gi i hạn của dãy so Đe cháng minh dãy so xn+1 = f(xn) h®i tụ bang phương pháp sả dụng tính đơn đi u và bị ch n, ta có the thực hi n như sau: Bước 1: Dự đoán xem dãy so đã cho tăng hay giảm (tính giá trị m®t vài so hạng đau). Giải phương trình L = f(L) đe dự đoán giới hạn L của dãy (xn). Neu dự đoán dãy tăng thì cháng minh xn ≤ L, ∀n = 1, 2, . . . đe suy ra dãy bị ch n trên. Neu dự đoán dãy giảm thì cháng minh xn ≥ L, ∀n = 1, 2, . . . đe suy ra dãy bị ch n dưới. Bước 2: Sả dụng ket quả dãy bị ch n ở bước 1 đe cháng minh dãy đơn đi u. Bước 3: Sả dụng định lý Weierstrass đe cháng minh dãy đã cho h®i tụ. Bài toán 3.1. Cho dãy (xn) xác định bởi: x1 = a (a > 1) x2 + 7 (3.1) x = n , n = 1, 2, . . . 2(xn + 3) Cháng minh rang dãy (xn) có giới hạn. Tìm giới hạn đó.
  • 33. 30 Viết đề tài giá sinh viên – ZALO:0973.287.149-TEAMLUANVAN.COM — − n+1 — n − n+1 n n Bài giai. Theo giả thiet x1 = a > 1. Giả sả xn > 1 với n ∈ N∗ . Ta có x2 + 7 (xn − 1)2 x 1 = n 1 = 2(xn + 3) 2(xn + 3) > 0. Suy ra xn+1 > 1. Theo nguyên lý quy nạp ta được xn > 1, ∀n ∈ N∗ . Suy ra dãy (xn) bị ch n dưới bởi 1. Xét hi u x2 + 7 x x = x = 2(xn + 3) (1 − xn)(7 + xn) 2(xn + 3) < 0. Suy ra xn+1 < xn, ∀n ∈ N∗ . Do đó dãy so (xn) là dãy giảm. Vì dãy so (xn) là dãy giảm và bị ch n dưới bởi 1 nên có giới hạn hǎu hạn là b ≥ 1. Tà công thác (3.1), chuyen qua giới hạn ta được b2 + 7 2 b = 2(b + 3) ⇔ b + 6b − 7 = 0 ⇔ b = 1 b = −7. Vì b ≥ 1 nên được b = 1 hay lim xn = 1. Bài toán 3.2 (Đe thi hoc sinh giỏi Quoc gia 2012). Cho dãy (xn) xác định bởi: x1 = 3 n + 2 (3.2) xn = 3n xn−1 + 2 , ∀n ≥ 2. Cháng minh rang dãy (xn) có giới hạn. Tìm giới hạn đó. Bài giai. Bang quy nạp, ta sě cháng minh với moi n = 3, 4, . . . , ta có 10 Do x2 = 3 n + 2 xn−1 ≥ n − 1 . (3.3) nên (3.3) đúng khi n = 3. Giả sả xk−1 k + 2 ≥ k − 1 , ∀k ≥ 3. "
  • 34. 31 Viết đề tài giá sinh viên – ZALO:0973.287.149-TEAMLUANVAN.COM 2 2 k − 1 k k 3k k−1 k 3k k − 1 k 2ak 2 Với k ≥ 3, xét x ≥ k + 3 ⇔ k + 2 x + 2 ≥ k + 3 ⇔ k + 2 k + 2 + 2 ≥ k + 3 (luôn đúng). ⇔ (k + 2) k + 2 + 2 ≥ 3(k + 3) ⇔ (k + 2)3k ≥ 3(k + 3)(k − 1) ⇔ 3k + 6k ≥ 3k + 6k − 9. V y xk k + 3 ≥ k . Theo nguyên lý quy nạp, suy ra xn−1 n + 2 ≥ n − 1 , ∀n = 3, 4, . . . De thay rang xn ≥ 0 với moi n nên dãy (xn) bị ch n dưới bởi 0. Xét xn — xn−1 = n + 2 x 3n n−1 + 2 − x n−1 = 2[n + 2 − (n − 1)xn−1] 3n ≤ 0, ∀n ≥ 3 (do (3.3). V y dãy (xn) giảm và bị ch n dưới bởi 0 nên có giới hạn hǎu hạn là a. Tà công thác (3.2) chuyen qua giới hạn ta được V y lim n→+∞ xn = 1. 1 a = 3 (a + 2) ⇔ a = 1 Bài toán 3.3 (Đe thi hoc sinh giỏi Quoc gia 2013). Cho dãy (an) xác định bởi: a1 = 3 an + 2 (3.4) an+1 = 3 − 2an , ∀n ≥ 1. Cháng minh rang dãy (an) có giới hạn. Tìm giới hạn đó. Bài giai. Bang quy nạp, ta sě cháng minh 1 < an < 2, ∀n ≥ 1. (3.5) 3 Do a2 = 2 nên(3.5) đúng khi n = 1, n = 2. Giả sả (3.5) đúng với n = k > 1 tác là 1 < ak < 2, ta can cháng minh 1 < 3 − ak + 2 < 2 ⇔ ak + 2 < 2ak < a + 2. k
  • 35. 32 Viết đề tài giá sinh viên – ZALO:0973.287.149-TEAMLUANVAN.COM 2 2 2 − − ∀ ∈ 2x − — − Xét các hàm so f(x) = 2x − (x + 2) và g(x) = 2x − x + 2 . Ta có fJ(x) = 2x ln 2 − 1 > 0, ∀x ∈ (1; 2) và gJ(x) = 2x ln 2 − 1 > 0, ∀x ∈ (1; 2). Suy ra f và g đeu đong bien trên khoảng (1; 2). Do đó tà 1 < ak < 2 ta có f(ak) < f(2) ⇒ 2ak − (ak + 2) < 0 g(ak) > g(1) ⇒ 2ak − ak + 2 > 0. V y (3.5) đúng với moi n ∈ N∗ . Tiep tục ta sě cháng minh dãy (an) là dãy so tăng. Xét hàm so h(x) = 3 x + 2 2x trên khoảng (1; 2). Ta có hJ(x) = (x + 2)2x ln 2 2x 22x = ln 4 + x ln 2 − 1 > 0, x (1; 2) nên hàm h(x) 2x đong bien trên khoảng (1; 2). Ket hợp với a 3 = > a , bang phương pháp quy nạp suy ra dãy (a ) là dãy tăng. 2 2 1 n Dãy (an) đã cho tăng và bị ch n trên bởi 2 nên có giới hạn hǎu hạn. Giả sả giới hạn đó là L ∈ (1; 2], ta có L = 3 − L + 2 2L . Ta sě cháng minh phương trình này có nghi m duy nhat trên (1; 2]. Th t v y, xét hàm so m(x) = 3 − x + 2 − x, x ∈ (1; 2]. mJ(x) = ln 4 + x ln 2 − 1 1 = 2x ln 4 + x ln 2 1 2x 2x . Ta xét tiep hàm so n(x) = ln 4 + x ln 2 − 1 − 2x, x ∈ (1; 2]. nJ(x) = ln 2(1 − 2x) < 0, ∀x ∈ (1; 2]. nên n(x) là hàm so nghịch bien trên (1; 2]. Suy ra n(x) < n(1) = ln 4 + ln 2 − 3 = ln 8 − 3 < 0 hay ln 4 + x ln 2 − 1 − 2x < 0 với x ∈ (1; 2]. Do đó, hàm so m(x) nghịch bien trên nảa khoảng (1; 2] và phương trình m(x) = 0 có không quá m®t nghi m trên (1; 2]. M t khác m(2) = 0 nên x = 2 là nghi m duy nhat của phương trình m(x) = 0. Tà đó suy ra L = 2. V y lim n→+∞ an = 2.
  • 36. 33 Viết đề tài giá sinh viên – ZALO:0973.287.149-TEAMLUANVAN.COM n n 1 n+1 2 n 4 n Bài toán 3.4. Cho dãy so (un) xác định bởi: u1 = u2 = 1 un+1 = √ un + √ un−1, ∀n ≥ 2. (3.6) Tính lim n→+∞ Bài giai. un. Ta thay u1 = u2, u3 = 1 + 1 = 2 > u2, u4 = √ u3 + √ u2 = √ 2 + 1 > u3. Dự đoán dãy so (un) là dãy so dương và tăng. Ta sě cháng minh dự đoán bang phương pháp quy nạp, tác là un+1 > un, ∀n ≥ 2. Rõ ràng un > 0, ∀n ≥ 1. Khi n = 2 ta có u3 = 2 > u1 = 2. Giả sả uk+1 > uk, ∀k ≥ 2. Ta có uk+2 = √ uk+1 + √ uk > √ uk + √ uk−1 = uk+1, ∀k ≥ 2 nên dãy (un) là dãy so dương tăng, suy ra un ≥ u1 = 1, ∀n ≥ 1. Hơn nǎa, ta thay ∀n ≥ 3, un = √ un−1 + √ un−2 < √ un + √ un = 2 √ un hay u2 < 4un ⇒ un < 4 (do un > 0), suy ra dãy (un) bị ch n trên bởi 4. Dãy so (un) tăng và bị ch n trên nên có giới hạn hǎu hạn. Giả sả lim n→+∞ un = α. Khi đó α ≥ 1. Tà h thác truy hoi suy ra lim n→+∞ un+1 = lim n→+∞ √ un + lim n→+∞ √ un−1 hay α = √ α + √ α suy ra α2 = 4α. Do α ≥ 1 nên α = 4. V y lim n→+∞ un = 4. Bài toán 3.5 (Đe thi hoc sinh giỏi Quoc gia 2015). Cho a là m®t so thực không âm và dãy (un) xác định bởi: u1 = 3 (3.7) un+1 = 1 2 un + n2 4n2 + a √ u2 + 3, ∀n ≥ 1. a) Với a = 0, cháng minh rang dãy (un) có giới hạn. Tìm giới hạn đó. b) Với moi a ∈ [0; 1], cháng minh rang dãy (un) có giới hạn. Bài giai. a) Với a = 0, ta có dãy (u ) xác định bởi u = 3, u = 1 u + 1√ u2 + 3, ∀n ≥ 1. Xét hàm so f(x) = 1 x + 1√ x2 + 3 với x > 0. Ta có 2 4 fJ(x) = 1 + 1 √ x > 0, ∀x > 0. 2 4 x2 + 3 n
  • 37. 34 Viết đề tài giá sinh viên – ZALO:0973.287.149-TEAMLUANVAN.COM √ ⇔ n n n 2 4 un + u2 + 3 ≤ un+1 ≤ un + xn+1 = 2 xn + 4 xn + 3 yn+1 = 2 yn + 4n2 + 1 1 1 n n Suy ra hàm so f(x) đong bien trên khoảng (0; +∞). Tà công thác thác truy hoi ta suy ra với moi n thì un > 0, un+1 = f(un) và 3 3 u2 = 2 + 2 < u1, bang phương pháp quy nạp suy ra dãy (un) là dãy giảm và bị ch n dưới bởi 0 nên nó có giới hạn hǎu hạn. Đ t lim n→+∞ un = x, x ≥ 0. Tà công thác (3.7) chuyen qua giới hạn ta được x = 1 x + 1√ x2 + 3 ⇔ √ x2 + 3 = 2x V y lim n→+∞ un = 1. x ≥ 0 x2 + 3 = 4x2 2 2 ⇔ x = 1. b) Với a ∈ [0; 1], ta có n ≤ n ≤ 1 . Do đó 4n2 + 1 1 n2 4n2 + a 4 √ 1 1√ Xét hai dãy so (xn) và (yn) xác định bởi x1 = 3 và y1 = 3 1√ 2 2 √ Theo ket quả câu a) ta có lim n→+∞ xn = 1. Ta sě cháng minh lim n→+∞ yn = 1 và tà đó suy ra được ket quả lim n→+∞ un = 1 và đó là đieu can cháng minh. Ta có 1 n2 (yn − 1)(yn + 1) 2n2 1 yn+1 − 1 = 2 (yn − 1) + 4n2 + 1 √ y2 + 3 + 2 + 4n2 + 1 − 2 1 n2 (yn − 1)(yn + 1) 1 = 2 (yn − 1) + 4n2 + 1 √ y2 + 3 + 2 − 2(4n2 + 1) 1 n2 (yn − 1)(yn + 1) < 2 (yn − 1) + 4n2 + 1 √ y2 + 3 + 2 1 n2 (yn + 1) De thay = (yn − 1) 2 + 4n2 + 1 √ y2 + 3 + 2 . 1 n2 (yn + 1) 1 n2 1 1 3 0 < 2 + 4n2 + 1 √ y2 + 3 + 2 < 2 + 4n2 + 1 < 2 + 4 = 4 , ∀n = 1, 2, . . . Neu yn > 1, ∀n thì ta có |yn+1 — 1| 3 < 4 . |yn — 1| < . . . < 3 n−1 |y1 — 1|. Tà đó suy ra lim n→+∞ yn = 1. 4 n 2 4n 2 n n n + 1 2 4 u2 + 3. y2 + 3. .
  • 38. 35 Viết đề tài giá sinh viên – ZALO:0973.287.149-TEAMLUANVAN.COM →+∞ ≥ uk n n k uk Neu ton tại n0 ∈ N∗ đe yn0 ≤ 1 thì ta có n lim yn < 1, ∀n > n0. 2 Neu ton tại N > n0 sao cho yN+1 > yN , khi đó do f(x) = x 4x2 + 1 là m®t hàm so đong bien trên khoảng (0; +∞) nên ta có (N + 1)2 N2 Suy ra 4(N + 1)2 + 1 > 4N2 + 1 . 1 (N + 1)2 q 2 1 N2 q 2 2 yN+1 + 4(N + 1)2 + 1 hay yN+2 > yN+1. yN+1 + 3 > 2 yN + 4N2 + 1 yN + 3 Bang phương pháp quy nạp ta cháng minh được dãy (yn) tăng tà so hạng thá N trở đi. Do đó dãy (yn) h®i tụ. Đ t lim n→+∞ yn = y ta tìm được y = 1. Neu dãy (yn) là dãy giảm ke tà so hạng thá n0 trở đi, ket hợp với dãy (yn) bị ch n dưới bởi 0 ta suy ra dãy (yn) h®i tụ. Đ t lim n→+∞ yn = y ta được y = 1 nên trường hợp này loại. V y lim n→+∞ yn = 1 và bài toán được cháng minh. Bài toán 3.6. Cho a > 0, dãy so (un) xác định bởi: u1 > 0 1 a (3.8) Tính lim n→+∞ un. un+1 = 2 un + n , ∀n ≥ 1. Nh n xét 3.1. Với ví dụ này, vi c tìm được công thác của so hạng tőng quát của dãy so là khá khó khăn nên ta sě cháng minh rang dãy (un) là dãy so giảm và bị ch n dưới bởi √ a. Bài giai. Th t v y, theo bat đȁng thác AM - GM ta có u2 = 1 u 2 1 + a √ a. u1 Giả sả uk ≥ √ a, ∀k ≥ 2, ta cháng minh uk+1 ≥ √ a. Theo bat đȁng thác AM - GM và giả thiet quy nạp ta có uk+1 = 1 u 2 k + a ≥ r u . a = √ a. Do đó un ≥ √ a, ∀n ≥ 2, nên dãy (un) bị ch n dưới bởi √ a. M t khác, ta có un+1 = 1 + a mà u ≥ √ a, ∀n ≥ 2, suy ra 1 ≤ 1 . un 2 2u2 n 2u2 2a u
  • 39. 36 Viết đề tài giá sinh viên – ZALO:0973.287.149-TEAMLUANVAN.COM n 4 4 0 < un = u . u · · · u . · · · 4 .u1 = 4 , ∀n ∈ N n Do đó un+1 1 a = + 1 a ≤ + = 1 ⇒ u ≤ u , ∀n ≥ 1 nên dãy (u ) là dãy giảm. un 2 2u2 2 2a n+1 n n V y dãy so (un) có giới hạn hǎu hạn. Giả sả lim n→+∞ un = α, khi đó α > 0. Tà h thác truy hoi suy ra lim un+1 = lim 1 u + a ⇒ 1 α + a ⇒ α = √ a (do α > 0). n→+∞ n→+∞ 2 un 2 α V y lim n→+∞ un = √ a. 3.2 Sfi dnng nguyên lj kep đe tính gi i hạn của dãy so Bài toán 3.7. Cho dãy so (un) xác định bởi: 1 u1 = 4 Tìm lim n→+∞ Bài giai. un. un+1 = u2 + un 2 , ∀n ≥ 1. Bang quy nạp ta de dàng cháng minh được un > 0, ∀n ∈ N∗ . 1 Ta cũng cháng minh un ≤ , ∀n ∈ N∗ . Với n = 1 thì u1 1 = , m nh đe đúng. 4 Giả sả uk 1 ≤ 4 , ∀k ≥ 1, ta cháng minh uk+1 1 ≤ 4 . Th y v y, u 1 2 1 3 ≤ ⇒ u ≤ u và u 3 1 3 ≤ . = . Do đó k 4 k 4 k 4 k 4 4 16 1 1 3 3 1 uk+1 ≤ 4 uk + 2 uk = 4 uk ≤ 16 < 4 . 1 Theo nguyên lý quy nạp, ta có 0 < un ≤ , ∀n ∈ N∗ . Suy ra un+1 = u + 1 ≤ 1 + 1 = 3 , ∀n ∈ N∗ . un Do đó, ta có n 2 un 4 2 4 un−1 u2 3 3 3 1 3 n−1 ∗ Mà lim 1 3 n−1 = 0, nên theo nguyên lý kep thì lim un = 0. n→+∞ 4 4 n→+∞ 4 4 4 1 n−2 n−1 n .u1 ≤ .
  • 40. 37 Viết đề tài giá sinh viên – ZALO:0973.287.149-TEAMLUANVAN.COM 2 n n √ u + 1 n 0 < un = u . u · · · u .u1 ≤ . . = n n u2 + 1 a2 + 1 Nh n xét 3.2. Bài toán trên khó tìm được so hạng tőng quát. Có the giải bài toán trên bang cách chỉ ra dãy so trên là dãy so giảm và bị ch n dưới bởi 0. Bài toán 3.8. Cho dãy so (un) xác định bởi: 1 u1 = 2 un Tìm lim n→+∞ un. un+1 = n + 1 , ∀n ≥ 1. Nh n xét 3.3. Vi c xác định công thác tőng quát của dãy so là khá khó khăn. Do đó đe tìm giới hạn của dãy so ta đánh giá t so un+1 un tà đó dùng định lý giới hạn kep đe tìm giới hạn. Bài giai. Bang quy nạp ta cháng minh được un > 0, ∀n ≥ 1. Tà h thác truy hoi, ta có un+1 1 1 = ≤ , ∀n ≥ 1. Ta có un un un−1 n + 1 2 u2 1 1 1 1 1 n Mà lim n→+∞ 1 n = 0 nên theo nguyên lý giới hạn kep thì lim n→+∞ un = 0. Bài toán 3.9. Cho dãy so (un) xác định bởi: u1 = a, (−1 < a < 0) un + 1 (3.9) un+1 = √ u2 + 1 − 1, ∀n ≥ 1. 1 a) Cháng minh rang 0 < un+1 + 1 ≤ √ a2 + 1 (un + 1), ∀n ≥ 1. b) Tính lim n→+∞ un. Bài giai. Bang cháng minh quy nạp ta có −1 < un < 0, ∀n ∈ N∗ . Tà đó suy ra 0 < un + 1 < 1 và √ u2 + 1 > 1. un + 1 Suy ra un+1 = 2 − 1 < (un + 1) − 1 = un, ∀n ≥ 1, nên dãy (un) là dãy giảm. n Do đó −1 < un ≤ un−1 ≤ · · · ≤ u1 = a < 0, ∀n ≥ 1. ⇒ u2 ≥ a2 ⇒ √ u2 + 1 ≥ √ a2 + 1 ⇒ √ 1 1 ≤ √ . 2 2 2 2 2 1 n−2 n−1 · · · , ∀n ≥ 1. .
  • 41. 38 Viết đề tài giá sinh viên – ZALO:0973.287.149-TEAMLUANVAN.COM n ∈ | | n xn+1 = f(xn), ∀n ∈ N∗ . 0 < < 1 a2 + 1 lim n→+∞ (a + 1) a2 + 1 1 = 1 Suy ra un + 1 1 0 < un+1 + 1 = √ u2 + 1 ≤ √ a2 + 1 (un + 1), ∀n ≥ 1. ⇒ 0 < un + 1 1 ≤ √ a2 + 1 (un−1 + 1) ≤ √ 1 a2 + 1 2 (un−2 + 1) ≤ · · · ≤ √ 1 n−1 ∀ ≥ hay −1 < un ≤ √ 1 a2 + 1 n−1 .(a + 1) − 1, ∀n ≥ 1. a2 + 1 (u1 + 1), n 1. 1 Vì √ nên h √ 1 n−1 − i − . Do đó, theo nguyên lý giới hạn kep ta có lim n→+∞ un = −1. 3.3 Sfi dnng định lj Lagrange đe tính gi i hạn của dãy so Định nghĩa 3.1 (xem [5]). Hàm so y = f(x) xác định trên khoảng (a; b) được goi là liên tục tại x0 (a; b) neu lim x→x0 f(x) = f(x0). Nh n xét 3.4. Hàm so sơ cap liên tục trên t p xác định của nó. Định lj 3.1. Neu hàm so y = f (x) liên tục trên đoạn [a; b] và f (a).f (b) < 0 thì phương trình f(x) = 0 có nghi m trên khoảng (a; b). Neu thêm giả thiet f(x) đơn đi u trên đoạn [a; b] thì phương trình f (x) = 0 có đúng m®t nghi m trên khoảng (a; b). Định lj 3.2 (Định lý Lagrange, xem [5]). Neu hàm so y = f(x) liên tục trên đoạn [a; b] và có đạo hàm trên khoảng (a; b) thì ton tại m®t so c ∈ (a; b) sao cho f(b) − f(a) = fJ(c)(b − a). Định lj 3.3. Neu q là so mà q < 1 thì lim →+∞ qn = 0. Khi g p dãy so (xn) xác định bởi: x1 = a với f(x) là hàm so xác định trên D và |fJ(x)| < 1, ∀x ∈ D, phương trình f(x) = x có nghi m duy nhat trên t p con C nào đó của D, ta có the sả dụng định lý Lagrange.
  • 42. 39 Viết đề tài giá sinh viên – ZALO:0973.287.149-TEAMLUANVAN.COM n n ≥ x 3 3 = 1 x 3 + 125 3 x2 125 . 2 = 5. n 3 x2 3 3x3 3 5 x2 Khi đó xn+1 n + xn xn.xn Bài toán 3.10. Cho dãy (xn) xác định bởi: x1 = a (a > 0) (3.10) xn+1 = 1 2x 3 n + 125 , ∀n ∈ N∗ . Cháng minh rang dãy (xn) có giới hạn. Tìm giới hạn đó. Bài giai. Tà giả thiet suy ra xn > 0, ∀n ∈ N∗ .r Xét hàm so f(x) = 1 2x + 125 trên khoảng (0; +∞). Khi đó xn+1 = f(xn), ∀n = 1, 2, . . . và hàm so f(x) xác định, liên tục trên khoảng (0; +∞). Ta có fJ(x) = 2 − 250 , suy ra |fJ(x)| < 2 < 1, ∀x ≥ 5. Theo định lý Lagrange ta có |xn — 5| = |f(xn−1) − f(5)| = |fJ(xn )|.|xn−1 2 — 5| ≤ 3 .|xn−1 — 5|, ∀n > 1. Do đó |xn — | ≤ 2 n | — 5|, ∀n > 1. Do lim n→+∞ 2 n = 0 nên lim n→+∞ xn = 5. Bài toán 3.11. Cho dãy (xn) xác định bởi: x1 = a (a > 0) xn+1 = ln(1 + 2e−xn ) (n = 1, 2, . . .) Cháng minh rang dãy (xn) có giới hạn. Tìm giới hạn đó. Bài giai. Tà giả thiet suy ra xn > 0, ∀n ∈ N∗ . Xét hàm so f(x) = ln(1 + 2e−x) trên khoảng (0; +∞). (3.11) Khi đó xn+1 = f(xn), ∀n = 1, 2, . . . và hàm so f(x) xác định, liên tục trên khoảng (0; +∞). −x Ta có fJ(x) = −2e 1 + 2e−x = −2 ex + 2 suy ra |fJ(xn 2 )| = ex + 2 2 < 3 < 1, ∀x > 0. Đ t g(x) = f(x) − x. Khi đó gJ(x) = fJ(x) − 1 < 0, ∀x > 0 suy ra g(x) nghịch bien trên khoảng (0; +∞) mà g(ln 2) = ln(1 + 2e− ln 2) − ln 2 = 0 nên x = ln 2 là nghi m duy nhat của phương trình f(x) = x trên khoảng (0; +∞). x1
  • 43. 40 Viết đề tài giá sinh viên – ZALO:0973.287.149-TEAMLUANVAN.COM 4 ∗ 2 2 = .fJ(c). . |xn − 2| < 3 ln 3 . |xn − 2| Ta cháng minh L = ln 2 là giới hạn của dãy (xn). Theo định lý Lagrange ton tại c nam giǎa xn và ln 2 sao cho |xn+1 − ln 2| = |f(xn) − f(ln 2)| = |fJ(c)|.|xn − ln 2| 2 < 3 .|xn − ln 2| < 3 .|xn−1 − ln 2| 2 n Do lim 2 n = 0 nên lim < · · · < 3 |x1 − ln 2|. xn = 0. n→+∞ 3 n→+∞ Bài toán 3.12. Cho dãy (xn) xác định bởi: x1 = a (a > 0) (3.12) 4 x = log (x3 1 , ∀n ∈ N∗ . Cháng minh rang dãy (xn) có giới hạn. Tìm giới hạn đó. Bài giai. Tà giả thiet suy ra xn > 3 , ∀n ∈ N . 3 1 4 Xét hàm so f(x) = log3(x + 1)3 + 3 trên khoảng (0; +∞). Khi đó xn+1 = f(xn), ∀n = 1, 2, . . . và hàm so f(x) xác định, liên tục trên khoảng (0; +∞). 2 Ta có fJ(x) = x . (1 + x3) ln 3 3 x3 x3 r 3 x6 3 2 Với moi x > 0 ta có x √ 3 4 + 1 = 2 + 2 + 1 ≥ 3 4 = √ 3 4 x . Do đó |fJ(x)| ≤ 3 ln 2 < 1, ∀x > 0. Đ t g(x) = f(x) − x. Khi đó gJ(x) = fJ(x) − 1 < 0, ∀x > 0 suy ra g(x) nghịch bien trên khoảng (0; +∞) mà g(2) = 0 nên x = 2 là nghi m duy nhat của phương trình f(x) = x trên khoảng (0; +∞). Ta cháng minh L = 2 là giới hạn của dãy (xn). Theo định lý Lagrange ton tại c nam giǎa xn và 2 sao cho |xn+1 − 2| = |f(xn) − f(2)| . . √ 3 4 √ 3 4 < 3 ln3 2 . |xn−1 − 2| < · · · < √ 3 4 3 ln 3 n . |x1 − 2| . n + 1)3 + 3 3 n+1
  • 44. 41 Viết đề tài giá sinh viên – ZALO:0973.287.149-TEAMLUANVAN.COM 2 2 2 2 2 4 . . . . − fJ(x) = 1 + cos x − sin x = 1 − √ 2 sin x − π < 0 với x ∈ hπ ; π i π 0 4 3π 2 Do lim n→+∞ √ 3 4 n 3 ln 3 = 0 nên lim n→+∞ xn = 2. Bài toán 3.13. Cho dãy (xn) xác định bởi: x1 = 2 (3.13) xn+1 = xn + sin xn + cos xn, ∀n ∈ N∗ . Cháng minh rang dãy (xn) có giới hạn. Tìm giới hạn đó. Bài giai. Ta cháng minh xn ∈ h ; π i , ∀n ∈ N∗ . π Xét hàm so f(x) = x + sin x + cos x trên hπ ; π i . Ta có f(x) xác định, liên tục trên khoảng π ; π . 4 2 nên hàm so f(x) nghịch bien trên khoảng π ; π . 2 Với x ≥ ta có x = f(x ) ≤ f = + 1 < π. π π π Với xn ≤ π ta có xn+1 = f(xn π ) ≥ f(π) = π − 1 ≥ 2 . V y với moi xn ∈ hπ ; π i thì xn+1 ∈ hπ ; π i . Trên hπ ; π i , phương trình f(x) = x + sin x + cos x = 0 có nghi m x = 3π , |fJ(x)| = |1 + cos x − sin x| = |1 − √ 2 sin x − π | ≤ √ 2 − 1. Theo định lý Lagrange ton tại c nam giǎa xn và x0 sao cho |xn+1 − x0| = |f(xn) − f(x0)| = fJ(c) .|f(xn) − f(x0)| = fJ(c) .|xn − x0| ≤ ( √ 2 − 1).|xn − x0|. Suy ra |xn − x0| ≤ ( √ 2 − 1)n−1|x1 − x0|, ∀n > 1. Do lim n→+∞ ( √ 2 1)n−1 = 0 nên lim xn = . n→+∞ 4 Bài toán 3.14. Xét phương trình (trong đó n là so nguyên dương) 1 1 1 1 1 x − 1 + 4x − 1 + · · · + k2x − 1 + · · · + n2x − 1 = 2 . (3.14) a) Cháng minh rang với moi so nguyên dương n, phương trình nêu trên có nghi m duy nhat lớn hơn 1, kí hi u nghi m đó là xn. b) Cháng minh rang dãy so (xn) có giới hạn bang 4 khi n → +∞. 2 2 n n+1 2 n
  • 45. 42 Viết đề tài giá sinh viên – ZALO:0973.287.149-TEAMLUANVAN.COM ∞ x − ∀ ∈ n − 2 2 2 − 1 2 + 1 2n − 1 2n + 1 2 2 2n + 1 2(2n + 1) n Bài giai. Viet lại phương trình của bài toán dưới dạng: 1 1 1 1 1 − 2 + x − 1 + 4x − 1 + · · · + k2x − 1 + · · · + n2x − 1 = 0. Với moi n = 1, 2, 3, . . . ta xét hàm so: 1 1 1 1 1 fn(x) = − 2 + x − 1 + 4x − 1 + · · · + k2x − 1 + · · · + n2x − 1 . a) De thay, với moi n ∈ N∗ , hàm so fn(x) liên tục và nghịch bien trên khoảng (1; +∞). Hơn nǎa ta có lim x→1+ fn(x) = + , lim →+∞ 1 fn(x) = − 2 < 0. Tà đó suy ra với moi n = 1, 2, . . . phương trình fn(x) = 0 có nghi m duy nhat xn ∈ (1; +∞). b) Với moi n ∈ N∗ , ta có: 1 1 1 1 1 fn(4) = − 2 + 22 − 1 + 42 − 1 + · · · + (2k)2 − 1 + · · · + (2n)2 − 1 1 1 1 1 = − 2 + (2 − 1)(2 + 1) + (4 − 1)(4 + 1) + · · · + (2n − 1)(2n + 1) = − 1 + 1 h 1 − 1 + · · · + 1 − 1 i = − 1 + 1 1 − 1 = −1 < 0 = f (x). Tà đó, do fn(x) = 0 nghịch bien trên (1; +∞) nên suy ra xn < 4, ∀n ∈ N∗ . M t khác, do với moi n = 1, 2, . . . , hàm fn(x) có đạo hàm trên đoạn [xn; 4] nên theo định lý Lagrange, suy ra với moi n ∈ N∗ ton tại tn ∈ (xn; 4) sao cho 2 fn(4) − fn(xn) = f ′ (t) = − 1 − 4 − · · · − n < − 1 (3.15) 4 − xn (t − 1)2 (4t − 1)2 (n2t − 1)2 9 (do 1 < t < 4). Mà fn(xn) = 0 nên tà (3.15) ta có −1 < − 1 ⇔ 4 − x 9 < ⇔ x 9 > 4 − (3.16) 2(2n + 1)(4 − xn) 9 n 2(2n + 1) n 2(2n + 1) Tà (3.16) có 4 9 2(2n + 1) < xn < 4, n N∗ mà lim →+∞ 4 9 = 4 = 2(2n + 1) lim n→+∞ 4, nên sả dụng nguyên lý kep ta được lim n→+∞ xn = 4. n
  • 46. 43 Viết đề tài giá sinh viên – ZALO:0973.287.149-TEAMLUANVAN.COM Σ n n x = n + 1 n un+1 = u2 − 5un + 9, ∀n ∈ N∗ . = — 2 — 3 − u = u — 3 − u — 3 , ∀n ∈ N∗ . n→+∞ n u1 − 3 un+1 − 3 u1 − 3 7 un. 3.4 Xác định gi i hạn của dãy tong Bài toán 3.15. Cho dãy so (un) xác định bởi: u1 = 10 (3.17) Đ t vn = n k=1 1 uk − 2 , ∀n ∈ N∗ . Tìm lim n→+∞ vn. Bài giai. Tà (3.17), ta có un+1 − 3 = (un − 2)(un − 3) 1 ⇔ un+1 = — 3 un 1 1 — 3 − un − 2 ⇔ 1 = 1 − 1 , ∀n ∈ N∗ . Do đó un − 2 un − 3 un+1 − 3 Σ 1 Σ 1 1 1 1 M t khác un+1 − un = (un − 3)2 > 0, ∀n ∈ N∗ nên dãy (un) là dãy tăng. Neu dãy (un) bị ch n trên thì dãy (un) có giới hạn là a. Tà (3.17), chuyen qua giới hạn ta có a = a2 − 5a + 9 ⇔ a2 − 6a + 9 = 0 ⇔ a = 3. Đieu này vô lý vì u1 = 10 và dãy (un) là dãy tăng. Do đó lim n→+∞ un = +∞. V y, lim v = lim 1 − 1 = 1 = 1 . Bài toán 3.16. Cho dãy so (xn) xác định bởi: x1 = 3 Σ n 1 √ xn(xn + 3)(x2 + 3xn + 2) + 1, ∀n ∈ N∗ . (3.18) Bài giai. Tà giả thiet suy ra xn > 0, ∀n ∈ N∗ . i=1 xi + 2 n+1 1 — 3 k+1 k u k=1 k u k=1 n vn = Đ t un = . Tìm lim n→+∞
  • 47. 44 Viết đề tài giá sinh viên – ZALO:0973.287.149-TEAMLUANVAN.COM √ x = x (x + 3)(x + 3 x +2) + 1 n+1 n n n n n − n n n ∀ ∈ N ∗ n + 1 n n n n n n n n n→+∞ n n→+∞ x1 + 1 xn+1 + 1 4 u1 = 2017 Σ 1 Ta có 2 n = q (x2 + 3xn + 1)2 = x2 + 3xn + 1. Xét xn+1 −xn = x2 +3xn +1−xn = (xn + 1)2 > 0, ∀n ∈ N∗ nên dãy (xn) là dãy tăng. Neu dãy (xn) bị ch n trên thì dãy (xn) có giới hạn hǎu hạn là a. Tà xn+1 = x2 +3xn+1 suy ra a = a2 + 3a + 1 suy ra a = 1 (vô lí). Do đó lim n→+∞ xn = +∞. Tà xn+1 = x2 + 3xn + 1 suy ra xn+1 + 1 = x2 + 3xn + 2 = (xn + 1)(xn + 2). n 1 ⇒ xn+1 = + 1 (xn 1 + 1)(xn n = + 2) xn 1 1 + 1 − xn + 2 ⇒ xn 1 = + 2 xn 1 + 1 − x 1 n+1 + 1 Σ 1 Σ 1 1 1 1 Suy ra ⇒ un = i=1 = xi + 2 i=1 xi + 1 − xi + 1 = x + 1 − x . n+1 + 1 lim u = lim 1 − 1 = 1 . Bài toán 3.17. Cho dãy so (un) xác định bởi: u4 + 20162 u = , n . u3 − un + 4032 (3.19) n 1 Đ t vn = 3 , ∀n ∈ N∗ . Tìm lim n vn. k=1 uk + 2016 Bài giai. Tà (3.19), ta có u4 + 20162 →+∞ (un − 2016)(u3 + 2016) un+1 − 2016 = n − 2016 = n . u3 − un + 4032 un(u2 − 1) + 2016 Tà đó bang quy nạp ta cháng minh được un > 2016, ∀n ∈ N∗ . Ta có Suy ra un+1 − 2016 = (un − 2016)(u3 + 2016) (u3 + 2016) − (un − 2016) 1 1 1 un+1 = − — 2016 un − 2016 u3 + 2016 .
  • 48. 45 Viết đề tài giá sinh viên – ZALO:0973.287.149-TEAMLUANVAN.COM n n n n n n − 2016 1 hay Do đó 1 u3 + 2016 = u 1 — 2016 − u n+1 1 . — 2016 Σ 1 Σ 1 1 vn = k=1 u3 + 2016 = 1 k=1 uk − 2016 − u 1 k+1 — 2016 1 M t khác = u − 2016 − u n+1 — 2016 = 1 − u n+1 . — 2016 un+1 − un = u4 − 4032un + 20162 u3 − un + 4032 (un − 2016)2 u3 − un + 4032 > 0, ∀n ∈ N∗ nên dãy (un) là dãy tăng. Neu dãy (un) bị ch n trên thì dãy (un) có giới hạn là a. a4 + 20162 Tà (3.19), chuyen qua giới hạn ta có a = a3 − a + 4032 ⇔ a = 2016. Đieu này vô lý vì u1 = 2017 và dãy (un) là dãy tăng. Do đó lim n→+∞ un = +∞. V y, lim vn n→+∞ = lim n→+∞ 1 1 = 1. un+1 − 2016 Bài toán 3.18. Cho dãy so (un) xác định bởi: u1 = 1 u2016 (3.20) un+1 = n 2016 + un, ∀n ∈ N∗ . Đ t vn = 2015 1 + 2015 2 + 2015 3 2015 + · · · + n . Tìm lim vn. u2 u3 u4 un+1 n→+∞ Bài giai. Tà giả thiet suy ra un > 0, ∀n ∈ N∗ . Ta có u2016 un+1 − un = n > 0, ∀n ∈ N∗ nên dãy (un) là dãy tăng. Neu dãy (un) bị ch n trên thì dãy (un) có giới hạn là a. a2016 Tà (3.20), chuyen qua giới hạn ta có a = + a ⇔ a = 0. Đieu này vô lý vì 2016 u1 = 1 và dãy (un) là dãy tăng. Do đó lim n un = +∞. Tà un+1 — un u2016 = n 2016 > 0, ta có 1 →+∞ 1 u2016 n un − u n+1 = 2016un . un+1 u u u u k n =
  • 49. 46 Viết đề tài giá sinh viên – ZALO:0973.287.149-TEAMLUANVAN.COM n − u1 un+1 un+1 3 Suy ra Khi đó 2016 1 un 1 un+1 2015 = n . un+1 vn = Σ k=1 2015 k uk+1 = 2016 1 u1 1 1 + u2 u2 1 u + · · · + 1 un+1 = 2016 1 − 1 = 2016 1 − 1 . V y, lim vn n→+∞ = lim n→+∞ 2016 1 1 = 2016. un+1 u u − − −
  • 50. 47 Viết đề tài giá sinh viên – ZALO:0973.287.149-TEAMLUANVAN.COM n Chương 4 Các dạng toán khác liên quan đen dãy so 4.1 M t so dạng toán liên quan đen tính chat của dãy so Trong phan này, ta xét m®t so dạng toán cháng minh tính chat của dãy so như dãy so nguyên, dãy so tuan hoàn. Bài toán 4.1. Cho dãy so (un) xác định bởi: Cháng minh rang: u0 = 1, u1 = 2 un+2 = 4un+1 − un, ∀n ∈ N. (4.1) a) un − 1 là m®t so chính phương với moi n lẻ. b) un − 1 6 là m®t so chính phương với moi n chȁn. Nh n xét 4.1. Đây là dạng toán ve phương trình sai phân tuyen tính cap hai, song ở đây ta sě xét phương pháp cháng minh không sả dụng đen tính chat của sai phân. Đe cháng minh dãy so (bn) thỏa mãn bn là so chính phương với moi so nguyên dương n ta thường sả dụng m®t so hướng sau: • Hướng 1: Ta sě chỉ ra sự ton tại dãy so nguyên (cn) thỏa mãn bn = c2 , ∀n ∈ N∗ . Dãy so (cn) thường dự đoán bang cách tính m®t so giá trị đau c1, c2, . . . và tìm ra quy lu t của dãy (cn).
  • 51. 48 Viết đề tài giá sinh viên – ZALO:0973.287.149-TEAMLUANVAN.COM n n ⇔ n • Hướng 2: Ta cháng minh bnbn+2 là m®t so chính phương với moi so nguyên dương n, sau đó cháng minh bang quy nạp. • Hướng 3: Dựa vào công thác truy hoi ta tính được bn = c2 . Bài giai. a) Cách 1. Ta dự đoán dãy so (cn) sao cho u2n+1 − 1 = c2 , ta có u1 = 2, u3 = 26, u5 = 362, u7 = 5042 suy ra c0 = 1, c1 = 5, c2 = 19, c3 = 71. Khi đó ta thả thiet l p quan h truy hoi của dãy (cn) theo dãy tuyen tính cap hai. Giả sả cn+2 = acn+1 + bcn và tà c0 = 1, c1 = 5, c2 = 19, c3 = 71, ta được 5a + b = 19 19a + 5b = 71 a = 4 b = −1. Do đó ta dự đoán dãy so (cn ) là c0 = 1, c1 = 5 cn+2 = 4cn+1 − cn, n = 0, 1, . . . Ta sě cháng minh bang quy nạp u2n+1 − 1 = c2 , n = 0, 1, . . . (4.2) Th t v y (4.2) đúng với n = 0. Giả sả (4.2) đúng đen n, ta sě cháng minh (4.2) đúng đen n + 1. Ta có u2n+3 − 1 = 4u2n+2 − u2n+1 − 1 = 4(4u2n+1 − u2n) − u2n+1 − 1 = 16u2n+1 − 4u2n − u2n+1 − 1 = 15u2n+1 − (u2n+1 + u2n−1) − 1 = 14u2n+1 − u2n−1 − 1 = 14(c2 + 1) − c2 − 1 − 1 hay n n−1 u2n+3 − 1 = 12c2 − c2 − 12. (4.3) n n−1
  • 52. 49 Viết đề tài giá sinh viên – ZALO:0973.287.149-TEAMLUANVAN.COM n 2 n+1 n+1 Theo h thác cơ bản của dãy tuyen tính cap hai ta được cn+1cn−1 − c2 = −6 ⇒ (4cn − cn−1)cn−1 − cn = −6 2 2 Ta có ⇒ cn + cn−1 − 4cncn−1 − 6 = 0. c2 = (4cn − cn−1)2 = 16c2 − 8cncn−1 + c2 = 16c2 − 2 c2 + c2 — 6 + c2 hay n+1 n n−1 n n n−1 n−1 c2 = 14c2 − c2 − 12. (4.4) n+1 n n−1 Tà (4.3) và (4.4) suy ra u2n+3 − 1 = c2 . n+1 Do đó ta cháng minh được (4.2) đúng đen n + 1 suy ra (4.2) đúng. V y un − 1 là m®t so chính phương với moi n lẻ. Cách 2. Ta có un+2un − u2 = 3, ∀n ≥ 0. Tà h thác này ta được (un+2 −1)(un −1) = un+2un −un+2 −un +1 = u2 +3−4un+1 +1 = (un+1 − 2)2 . (4.5) Tà h thác (4.5), bang phương pháp quy nạp suy ra un − 1 là so chính phương với moi so nguyên dương n lẻ. b) Ta cháng minh theo hướng 2 như sau: un+2 − 1 . 6 un − 1 6 = un+2un − un+2 − un + 1 = 36 2 n+1 — 4un+1 + 4 36 un+1 − 2 2 Tà đȁng thác này, bang phương pháp quy nạp suy ra un − 1 6 là so chính phương. Bài toán 4.2 (TST Vi t Nam 2012). Cho dãy so (un) được xác định như sau: u1 = 1, u2 = 2011 un+2 = 4022un+1 − un, n = 1, 2, . . . (4.6) Cháng minh rang u2012 + 1 2012 là m®t so chính phương. Bài giai. Ta sě giải bài toán tőng quát sau: Cho p là m®t so nguyên dương lẻ và dãy so (un) được xác định như sau: u1 = 1, u2 = p un+2 = 2pun+1 − un, n = 1, 2, . . . 6 u = .
  • 53. 50 Viết đề tài giá sinh viên – ZALO:0973.287.149-TEAMLUANVAN.COM n+1 2 2 n+1 2 n+1 n+1 2 p + 1 p + 1 p + 1 n+1 n+1 n n p + 1 p + 1 2 2 2 Cháng minh rang u2n + 1 p + 1 là so chính phương với moi so nguyên dương n. Cách 1. Ta sě cháng minh theo hướng 1. Ta tính m®t vài giá trị đau tiên u2 + 1 = 1, u4 + 1 = (2p − 1)2, u6 + 1 = (4p2 − 2p + 1)2, . . . Ta dự đoán được u2n + 1 = x2 , trong đó (x ) là dãy so được xác định như sau: p + 1 n n x1 = 1, x2 = 2p − 1, · · · , xn+2 = 2pxn+1 − xn, n = 1, 2, . . . Ta sě cháng minh ket quả trên bang phương pháp quy nạp. Ta có xn+2xn − x2 = (−1)n−1(x3x1 − x2)2 = 2p − 2 2 ⇒ xn+2xn = xn+1 + 2p − 2 ⇒ (2pxn+1 − xn)xn = xn+1 + 2p − 2 2 2 Suy ra ⇒ xn+1 + xn + 2p − 2 = 2pxnxn+1. 2 n+2 = (2pxn+1 − xn)2 =4p2x2 — 4pxn+1xn + xn =4p2x2 − 2 x2 + x2 + 2p − 2 + x2 = 4p2 − 2 x2 − xn − 4p + 4. Do đó 4p2 − 2 u2n+2 + 1 − u2n + 1 − 4p + 4 4p2 − 2 u2n+2 − u2n + 1 p + 1 = u2n+4 + 1 . p + 1 Suy ra u2n+4 + 1 = x2 . Cách 2. p + 1 n+2 Ta sě cháng minh theo hướng 2. Trước het ta có h thác cơ bản sau un+2un − u2 = (−1)n−1 u3u1 − u2 = 2p2 − 1 − p2 = p2 − 1 ⇒ un+2un = un+1 + p − 1. x =
  • 54. 51 Viết đề tài giá sinh viên – ZALO:0973.287.149-TEAMLUANVAN.COM u u = 7u − u , n = 2,3, . ..n+1 n n−1 n n+1 n+1 p + 1 Ta có un+2 + 1 un + 1 = un+2un + un+2 + un + 1 (p + 1)2 2 = n+1 u2 + p2 − 1 + 2pun+1 + 1 (p + 1)2 + p 2 Tà đȁng thác này, bang phương pháp quy nạp ta được u2n + 1 p + 1 là so chính phương với moi so nguyên dương n. Bài toán 4.3 (China South East Mathematical 2011). Cho dãy so (un) được xác định như sau u1 = u2 = 1 Cháng minh rang với moi so nguyên dương n ta có un + un+1 + 2 là m®t so chính phương. Bài giai. Tính m®t vài giá trị đau tiên ta được: u1 + u2 + 2 = 22, u2 + u3 + 2 = 32, u3 + u4 + 2 = 72, u4 + u5 + 2 = 182. Tà đó ta dự đoán un + un+1 + 2 = x2 , trong đó dãy so (xn) được xác định như sau: x1 = 2, x2 = 3, xn+1 = 3xn − xn−1, n = 2, 3, . . . Ta sě cháng minh dự đoán này bang phương pháp quy nạp. Ta có xn+1xn−1 − x2 = 5 ⇒ (3xn − xn−1)xn−1 − x2 = 5 n n 2 2 ⇒ 3xnxn−1 = xn−1 + xn + 5 = xn−1 + xn + xn + xn+1 + 9 = xn+1 + 2xn + xn−1 + 9. Theo công thác truy hoi của dãy (xn) ta được: x2 = (3xn − xn−1)2 = 9x2 + x2 — 6xnxn−1 n+1 n n−1 Do đó x2 = 9(un + un+1 + 2) + un−1 + un + 2 − 2(un+1 + 2un + un−1 + 9) = 7un+1 − un + 7un − un−1 + 2 = un+2 + un+1 + 2. = un+1 + un+2 + 2 hay bài toán được cháng minh. p + 1 p + 1 = .
  • 55. 52 Viết đề tài giá sinh viên – ZALO:0973.287.149-TEAMLUANVAN.COM u = 3u− u , n = 1, 2, .. .n+2 n+1 n 2 √ ∗ 8 u + 1, n ∈ N . n ⇔ Bài toán 4.4 (Balkan MO 2002). Cho dãy so (un) xác định như sau: u1 = 20, u2 = 30 Tìm tat cả các so nguyên dương n sao cho 1 + 5unun+1 là m®t so chính phương. Bài giai. De thay dãy (un) là dãy so tăng, suy ra với n ≥ 4 ta có un + un+1 ≥ u4 + u5 > u3 + u4 = 250. (4.7) +) n ∈ {1, 2} không thỏa mãn. + n = 3 thì 1 + 5u3u4 = 2512 suy ra n = 3 thỏa mãn. +) n ≥ 4, theo tính chat cơ bản của dãy tuyen tính cap hai ta có: un+2un = u2 + (−1)n−1(u3u1 − u2) = u2 + 500 n+1 2 2 n+1 ⇒ (3un+1 − un)un = un+1 + 500 2 2 ⇒ 3un+1un = un+1 + un + 500 ⇒ 5un+1un + 1 = (un+1 + un) + 501. Giả sả 1 + 5unun+1 là so chính phương, 1 + 5unun+1 = a2, a ∈ N∗ . Khi đó ta có (un+1 + un)2 + 501 = a2 ⇔ (a − un+1 − un)(a + un+1 + un) = 501 = 1.501 = 3.167. Ta xét các trường hợp sau: Trường hợp 1: a + un + un+1 = 501 a − un − un+1 = 1 Trường hợp 2: a = 251 un + un+1 = 250 (mâu thuan với (4.7)). ⇔ a + un + un+1 = 167 a = 85 (mâu thuan với (4.7)). a − un − un+1 = 3 un + un+1 = 82 Do đó với n ≥ 4 thì 1 + 5unun+1 không phải là so chính phương. V y n = 3 là so nguyên dương thỏa mãn yêu cau bài toán. Bài toán 4.5. Cho dãy so (un) xác định bởi: u1 = 1 un+1 = 3un + 2
  • 56. 53 Viết đề tài giá sinh viên – ZALO:0973.287.149-TEAMLUANVAN.COM 2 u n 2 n n+1 n Cháng minh moi so hạng của dãy so đeu là so nguyên. Bài giai. Tà giả thiet ta có un+1 − 3un = √ 8u2 + 1 ⇔ u2 — 6un+1un + u = 1. Thay n bang n + 1 ta được h : 2 n+1 — 6un+1un + u2 = 1 u2 − 6unun−1 + u2 = 1. n Trà tàng ve của hai phương trình ta có n−1 2 n+1 — un−1 — 6un+1un + 6unun−1 = 0 ⇔ (un+1 − un−1)(un+1 + un−1 − 6un) = 0. Suy ra un+1 = un−1 ho c un+1 = 6un − un−1. Trường hợp un+1 = un−1 không the xảy ra. Trường hợp un+1 = 6un − un−1, xác định dãy (un) như sau: u1 = 1, u2 = 6, un+1 = 6un − un−1, ∀n ≥ 2. Do u1, u2 ∈ Z nên tà công thác xác định dãy ta có un ∈ Z, ∀n ∈ N∗ . V y moi so hạng của dãy so đeu là so nguyên. Bài toán 4.6. Cho dãy so (un) thỏa mãn un+2 = unun+1 , n = 1, 2, . . . 2un − un+1 Tìm đieu ki n can và đủ đoi với u1, u2 đe dãy so có vô so so hạng nguyên. Bài giai. Trước het ta cháng minh uk 0 với ∀k = 1, 2, . . . Giả sả ∃k ∈ N∗ đe uk = 0 thì uk+2 = ukuk+1 = 0. 2uk − uk+1 Tương tự, suy ra uk+3 = 0, suy ra 2uk+2 − uk+3 = 0, do đó uk+4 không ton tại. V y uk /= 0, ∀k ∈ N∗ . Đ t vn 1 = , theo cách thiet l p dãy so, ta có v un n+2 1 = un+2 , suy ra vn+2 = 2un − un+1 unun+1 = 2vn+1 — vn, n = 1, 2, . . . ⇔ vn+2 + vn = 2vn+1, ∀n ∈ N∗ . u
  • 57. 54 Viết đề tài giá sinh viên – ZALO:0973.287.149-TEAMLUANVAN.COM | | n n n n n n+1 n+1 Khi đó dãy (vn) l p thành cap so c®ng với công sai d. Vì vn 1 = , nên neu un un ∈ Z thì vn ∈ [−1; 1]. Do đó neu ton tại dãy con vô hạn của (un) là các so nguyên khác 0 thì trong dãy (vn) có m®t dãy con mà moi phan tả của nó đeu thu®c [−1; 1]. Đieu này chỉ xảy ra khi d = 0, vì neu d /= 0 thì ta thay lim n→+∞ vn = lim →+∞ |v1 + (n − 1) d| = +∞ nên neu ∃k0 đe ∀k ≥ k0 thì |vk| > 1, đieu này không xảy ra. V y d = 0, suy ra v1 = v2 = · · · = vn = . . . hay u1 = u2 = · · · = un = . . . Do đó đieu ki n can và đủ đe dãy (un) có vô so so hạng nguyên là u1 = u2 = a ∈ Z. Bài toán 4.7. Cho dãy so (un) được xác định như sau: u0 = 1, u1 = 13 un+2 = 14un+1 − un, n ≥ 0. (4.8) Cháng minh rang với moi so tự nhiên n, ton tại các so tự nhiên k, l sao cho un = k2 + (k + 1)2, u2 = (l + 1)3 − l3. Bài giai. Ta có un = k2 + (k + 1)2 = 2k2 + 2k + 1 ⇔ 2un − 1 = (2k + 1)2 và u2 = (l + 1)3 − l3 = 3l2 + 3l + 1 ⇔ 12u2 − 3 = (6l + 3)2. Như v y bài toán quy ve cháng minh 2un − 1, 12u2 − 3 là các so chính phương. Trước het ta có h thác cơ bản sau un+2un − u2 = 12, ∀n ≥ 0. Th t v y, ta có un+2 + un un+1 = un+1 + un−1 un ⇒un(un+2 + un) = un+1(un+1 + un−1) 2 2 2 Xét ⇒un+2un − un+1 = un+1un−1 − un = · · · = u0u2 − u1 = 12. (2un+2 − 1)(2un − 1) = 4un+2un − 2(un+2 + un) + 1 = 4(u2 + 12) − 28un+1 + 1 = (2un+1 + 7)2.
  • 58. 55 Viết đề tài giá sinh viên – ZALO:0973.287.149-TEAMLUANVAN.COM 2 2 2 n n n+2 n n n n Lại có (12u2 — 3)(12u − 3) = 144(un+2un) − 36(u + u2 ) + 9 = 144(un+2un)2 − 36(un+2 + un)2 + 72un+2un + 9 = 144(un+2un)2 − 36(14un+1)2 + 72un+2un + 9 = 144(un+2un)2 − 36.142(un+2un − 12)2 + 72un+2un + 9 = 144(un+2un)2 − 36.194un+2un + 2912 = (12un+2un − 291)2. V y ta cháng minh được 2un − 1, 12u2 − 3 là các so chính phương. Sau đây ta xét m®t so bài toán ve dãy so tuan hoàn c®ng tính và tuan hoàn nhân tính. Bài toán 4.8. Xác định dãy (xn) sao cho xn+3 = xn + 1, n = 0, 1, 2, . . . Bài giai. Đ t xn n = + yn 3 . Khi đó ta có yn+3 + n + 3 = 3 n 3 + yn + 1, hay yn+3 = yn, n = 0, 1, 2, . . . V y nên Do đó y0 = y3 = y6 = . . . y1 = y4 = y7 = . . . ⇔ yn = y2 = y5 = y8 = . . . a tùy ý với n = 3k, k ∈ N b tùy ý với n = 3k + 1, k ∈ N c tùy ý với n = 3k + 2, k ∈ N. xn = a + 3 tùy ý với n = 3k, k ∈ N b + 3 tùy ý với n = 3k + 1, k ∈ N n c + 3 tùy ý với n = 3k + 2, k ∈ N. Bài toán 4.9. Xác định dãy (xn) sao cho xn+3 = 2xn, n = 0, 1, 2, . . . Bài giai. Đ t xn = 23 yn. Khi đó ta có n+3 n 2 3 yn+3 = 2(23 yn), n+2
  • 59. 56 Viết đề tài giá sinh viên – ZALO:0973.287.149-TEAMLUANVAN.COM hay yn+3 = yn, n = 0, 1, 2, . . . V y nên y0 = y3 = y6 = . . . y1 = y4 = y7 = . . . ⇔ yn = y2 = y5 = y8 = . . . n a tùy ý với n = 3k, k ∈ N b tùy ý với n = 3k + 1, k ∈ N c tùy ý với n = 3k + 2, k ∈ N. Do đó xn = 2 3 yn, trong đó yn = a tùy ý với n = 3k, k ∈ N b tùy ý với n = 3k + 1, k ∈ N c tùy ý với n = 3k + 2, k ∈ N. Bài toán 4.10. Xác định dãy (un) thỏa mãn đieu ki n u2n+1 = 3un, ∀n ∈ N. (4.9) Bài giai. Đ t n + 1 = m, m = 1, 2, . . . Khi đó ta có the viet (4.9) dưới dạng u2m−1 = 3um−1, ∀m ∈ N∗ hay với v2m = 3vm, ∀m ∈ N∗ (4.10) vm = um−1, ∀m ∈ N∗ . Tà (4.10) ta có v0 = 0. Đ t vm = mlog2 3 ym, m ∈ N∗ . Khi đó (4.10) có dạng y2m = ym, m ∈ N∗ . V y (ym) là dãy tuan hoàn nhân tính chu kỳ 2. Khi đó, ta có yn Tà đó suy ra với yn = = tùy ý với n lẻ y2k+1 với n có dạng 2m(2k + 1), m ∈ N∗ , k ∈ N. um = vm+1 = mlog2 3 ym+1, tùy ý với n lẻ y2k+1 với n có dạng 2m(2k + 1), m ∈ N∗ , k ∈ N.
  • 60. 57 Viết đề tài giá sinh viên – ZALO:0973.287.149-TEAMLUANVAN.COM Bài toán 4.11. Xác định dãy (un) thỏa mãn đieu ki n u2n+1 = −3un + 4, ∀n ∈ N. (4.11) Bài giai. Đ t n + 1 = m, m = 1, 2, . . . Khi đó ta có the viet (4.11) dưới dạng u2m−1 = −3um−1 + 4, ∀m ∈ N∗ hay với v2m = −3vm + 4, ∀m ∈ N∗ (4.12) vm = um−1, ∀m ∈ N∗ . Đ t vm = 1 + xm, m ∈ N∗ . Khi đó (4.12) có dạng x2m = −3xm, m ∈ N∗ . (4.13) Đ t xm = mlog2 3 ym, m ∈ N∗ . Khi đó (4.13) có dạng y2m = −ym, m ∈ N∗ . V y (ym) là dãy phản tuan hoàn nhân tính chu kỳ 2. Khi đó, ta có yn = Tà đó suy ra tùy ý với n lẻ , −y2k+1 với n có dạng 22m+1(2k + 1), m, k ∈ N, y2k+1 với n có dạng 22m(2k + 1), m ∈ N∗ , k ∈ N. um = vm+1 = 1 + (m + 1)log2 3 ym+1, với yn = tùy ý với n lẻ , −y2k+1 với n có dạng 22m+1(2k + 1), m, k ∈ N, y2k+1 với n có dạng 22m(2k + 1), m ∈ N∗ , k ∈ N.
  • 61. 58 Viết đề tài giá sinh viên – ZALO:0973.287.149-TEAMLUANVAN.COM 1 2 2 3 — − un+1 = un(u2 n−1 — 2) − 2 4.2 M t so dạng toán khác Bài toán 4.12. Cho dãy (un) xác định bởi: u0 = 2 5 u1 = 2 5 (4.14) Cháng minh rang [un] = 2 22 − (−1)n 3 , ∀n ∈ N∗ . ( đây, ta kí hi u [un] là so nguyên lớn nhat không vượt quá un). Bài giai. Ta có 1 u2 = 2 + 2 . u3 = u2(u2 − 1) − 2 + 1 2 = 2 + 1 h 2 + 1 — 2 i = 23 1 + 23 . u4 = 23 + h 2 + − 2 i = 25 1 1 1 + 25 . Ta cháng minh bang quy nạp un = 2an + 2 Th t v y, tà (4.14), ta có −an với an = 2n − (−1n) 3 , ∀n ≥ 1. un+1 = 2an + 2−an h 2an−1 + 2−an−1 − 2 i − 2 + 1 = 2an+2an−1 + 2−an−2an−1 + 22an−1−an + 2an−2an−1 − 2 − 2−1. De thay do đó un+1 = 2an+1 + 2−an+1 . an + 2an−1 = an+1 2an−1 − an = (−1)n Vì v y un = 2 2n−(−1)n 3 1 2 2n−(−1)n , ∀n ≥ 1. Mà 2 2n−(−1)n 3 ∈ + và 0 < 1 2n−(−1)n 3 < 1 nên [un] = 2 22 ( 1)n 3 , ∀n ≥ 1. Bài toán 4.13 (Đe thi hoc sinh giỏi Quoc gia 2016). a) Cho dãy (an) xác định bởi an = ln(2n2 + 1) − ln(n2 + n + 1), với n = 1, 2, . . . Cháng minh rang chỉ có hǎu hạn so n sao cho {an 1 } < 2 . 2 23 2 + Z 2 , ∀n ∈ N∗ . 2 2
  • 62. 59 Viết đề tài giá sinh viên – ZALO:0973.287.149-TEAMLUANVAN.COM { } n — − n b) Cho dãy (bn) xác định bởi bn = ln(2n2 + 1) + ln(n2 + n + 1), với n = 1, 2, . . . Cháng minh rang ton tại vô hạn so n sao cho {bn 1 } < 2016 . Trong đó {x} là ký hi u phan lẻ của so thực x : {x} = x − [x]. Bài giai. 2n2 + 1 a) De thay 1 ≤ n2 + n + 1 < 2 với moi n = 1, 2, . . . Tà đó suy ra 0 ≤ an < ln 2 < 1 và [an] = 0. Với ket quả này, ta có {an} = an và lim n→+∞ an = lim →+∞ an = lim n→+∞ 2n2 + 1 ln n2 + n + 1 = ln 2. Do đó, ton tại n0 ∈ N∗ đe {an 1 } = ln 2 − 1992 với moi n ≥ n0. Bây giờ, neu có vô hạn so n đe {an 1 } < 2 , ta chon n1 > n0 là m®t trong các so đó. Khi đó, theo các lý lu n ở trên, ta có 1 1 hay 2 > {an1 } > ln 2 − 1992 , 1 1 1992 > ln 2 − 2 . Mâu thuan này cháng tỏ chỉ có hǎu hạn so n sao cho {an 1 } < 2 . b) De thay dãy (bn) tăng và lim n→+∞ bn = +∞. Ngoài ra, ta cũng có (2n2 + 1)(n2 + n + 1) lim n→+∞ (bn bn 1) = lim →+∞ ln (2n2 − 4n + 3)(n2 — n + 1) = 0. Trở lại bài toán, giả sả ton tại hǎu hạn n đe {bn 1 } < 2016 . Khi đó, ta thay ton tại n0 ∈ N∗ đe {bn 1 } ≥ 2016 với moi n ≥ n0 . Do lim n→+∞ (bn — bn−1) = 0 nên ton tại n1 ∈ N∗ đủ lớn đe 1 bn − bn−1 < 2016 với moi n ≥ n1. Vì dãy (bn) tăng và dan tới vô hạn nên ton tại vô so các so n > max{n0, n1} đe [bn] − [bn−1] = 1. Xét các so n như the, tà bat đȁng thác ở trên, ta suy ra hay 1 [bn] − [bn−1] + {bn} − {bn−1} < 2016 , 2015 {bn−1} > {bn} + 2016 .
  • 63. 60 Viết đề tài giá sinh viên – ZALO:0973.287.149-TEAMLUANVAN.COM hu hu k+1 k — u = 2u − − u k u0 = 1, u1 = 3 n+1 Do {bn 2015 } ≥ 2016 nên {bn−1} > 1. Mâu thuan này cháng tỏ ton tại vô hạn so n sao cho {bn 1 } < 2016 . Bài toán 4.14 (TST Vi t Nam 2011). Cho dãy so (un) được xác định như sau: un+2 = 1 + 2 n+1 un i , n ≥ 0. (4.15) Cháng minh rang un+2un − u2 = 2n với moi so tự nhiên n. ( đây, ta kí hi u [un] là so nguyên lớn nhat không vượt quá un). Bài giai. Ta thả dự đoán (un) là dãy tuyen tính dạng un+2 = pun+1 + qun + r với moi n ≥ 0. Theo công thác truy hoi ta tính được u2 = 10, u3 = 34, u4 = 116. Tà un+2 = pun+1 + qun + r ta có h Do đó 3p + q + r = 10 10p + 3q + r = 34 34p + 10q + r = 116 p = 4 ⇔ q = −2 r = 0. un+2 = 4un+1 − 2un, ∀n ≥ 0. (1) Ta sě cháng minh dãy (un) thỏa mãn công thác truy hoi (1) bang hai cách. Cách 1. Ta cháng minh bang quy nạp công thác truy hoi (1). Tà đȁng thác un+2 = 1 + 2 n+1 un i , bang quy nạp ta suy ra un+1 > 2un, ∀n ≥ 0 nên un > 2un−1 > · · · > 2nu0 = 2n, ∀n ≥ 1. (2) De thay (1) đúng với n = 0. Giả sả (1) đúng đen n = k ≥ 0 tác là uk+2 = 4uk+1 − 2un uk+2 + 2uk ⇒ uk+1 2 = uk+1 + 2uk−1 uk 2 k ⇒ uk+2uk − uk+1 = 2(uk+1uk−1 − uk) = · · · = 2 . uk+2uk 2u2 Ta có uk+2uk − u2 = 2(uk+1uk−1 − u2) ⇒ uk+22uk uk+1 k k+1 k 1 k+1 4u2 ⇒ uk+1 — 2uk+1 = 4uk−1 − u k+1
  • 64. 61 Viết đề tài giá sinh viên – ZALO:0973.287.149-TEAMLUANVAN.COM k k+2 ⇒ − u + 4u − k k − 1 k — k+2 k+2 k+1 hu hu 2 2 n n+1 x x hx hx n+1 i uk+2(4uk+1 − uk+2) uk+1 4u2 — 2uk+1 = 4uk−1 − u u2 4uk+2 2uk+1 = k+1 4u2 uk+1 u2 = k+2 uk+1 4(uk+1uk−1 − u2 ) uk+1 Ket hợp với (2) ta được: u2 = k+2 + uk+1 4.2k−1 uk+1 u2 = k+2 + uk+1 2k+1 . uk+1 2 k+2 uk+1 u2 < 4u 2u < + 1 uk+1 ⇒ 4uk+2 − 2uk+1 = 2 k+2 uk+1 + 1 i = 2 k+2 uk+1 + 1 = uk+3 Do đó (1) đúng với n = k + 1. V y đȁng thác (1) đúng với moi n ≥ 0. Tà (1) suy ra được un+2 = 4un+1 − 2un un+2 + 2un ⇒ un+1 = un+1 + 2un−1 un ⇒ un+2un — un+1 = 2(un+1un−1 — un) = · · · = 2 hay un+2un − u2 = 2n với moi so tự nhiên n. Cách 2. Ta xây dựng dãy (xn) thỏa mãn x0 = 1, x1 = 3 và xn+2 = 4xn+1 − 2xn, ∀n ≥ 0. Tà cách xây dựng dãy (xn) ta được: xn+2 + 2xn = xn+1 + 2xn−1 ⇒ x x + 2x2 = x2 + 2x x xn+1 xn 2 n+2 n 2 n n+1 n n+1 2 n−1 n ⇒ xn+2xn − xn+1 = 2(xn+1xn−1 − xn) = · · · = 2 (x2x0 − x1) = 2 x2 2n n+1 ⇒ xn+2 = + . xn xn Bang quy nạp de thay dãy (xn) là m®t dãy tăng và do đó xn = 4xn−1 − 2xn−2 > 2xn−1 > 22xn−2 > · · · > 2nx1 = 2n. Suy ra 2 n+1 xn 2 2n < xn+2 = n+1 + xn xn 2 < n+1 + 1 xn ⇒ xn+2 = 2 n+1 xn + 1 i = 1 + 2 n+1 xn i , ∀n ≥ 0. hx2 i V y dãy (xn) là m®t dãy thỏa mãn x0 = 1, x1 = 3 và xn+2 = 1 + Do đó ta được un = xn, ∀n ≥ 0. n+1 xn , ∀n ≥ 0. V y un+2un − u2 = 2n với moi so tự nhiên n. k+1 u x ⇒ +
  • 65. 62 Viết đề tài giá sinh viên – ZALO:0973.287.149-TEAMLUANVAN.COM n k n−2 2 n 2 Bài toán 4.15. Cho dãy so (un) được xác định như sau: u0 = 0, u1 = 1 (4.16) un+2 − 3un+1 + un 2 = (−1)n+1, ∀n ≥ 0. Cháng minh rang với moi so tự nhiên n, un là so chính phương. Bài giai. Ta có u2 = 1; u3 = 4; u4 = 9; u5 = 25. Do đó u0 = F 2; u1 = F2; u2 = F2; u3 = F2; u4 = F2; u5 = F2, ở đó (Fn) là dãy 0 1 2 Fibonacci. 3 4 5 Tà đó ta có định hướng cháng minh un = F2 bang quy nạp theo n. Giả sả uk = F 2 với moi k ≤ n. Như v y un = F2; un−1 = F2 ; un−2 = F2 . n n−1 n−2 Tà giả thiet ta có un+1 − 3un + un−1 = 2.(−1)n và un − 3un−1 + un−2 = 2.(−1)n−1. C®ng hai đȁng thác trên ta được un+1 − 2un − 2un−1 + un−2 = 0, n ≥ 2. Tà đó suy ra un+1 = 2F2 + 2F2 − F n n−1 n−2 = (Fn + Fn−1)2 + (Fn − Fn−1)2 − F2 2 n+1 + F2 n−2 — Fn−2 n+1. V y un = F2, ∀n ≥ 0 (đieu phải cháng minh). 2 = F = F
  • 66. 63 Viết đề tài giá sinh viên – ZALO:0973.287.149-TEAMLUANVAN.COM Ket lu n Lu n văn ”M®t so dạng toán ve dãy so sinh bởi các hàm so sơ cap” trình bày nhǎng van đe sau: 1. Lu n văn đã trình bày chi tiet m®t so tính chat cơ bản của dãy so và các dạng toán liên quan. 2. Trình bày các dạng toán ve xác định dãy so sinh bởi các hàm hǎu t (đa thác, phân thác hǎu t ), hàm vô t , hàm lượng giác và các hàm siêu vi t. 3. Trình bày m®t so phương pháp tính giới hạn của dãy so. 4. Trình bày các đe toán thi hoc sinh giỏi trong nước, Olympic khu vực và quoc te liên quan đen dãy so.
  • 67. Viết đề tài giá sinh viên – ZALO:0973.287.149-TEAMLUANVAN.COM Tài li u tham khảo [A] Tieng Vi t [1] Nguyen Tài Chung (2013), Boi dưỡng hoc sinh giói chuyên khảo dãy so, NXB ĐHQG Hà N®i. [2] Phan Huy Khải (2009), Chuyên đe so hoc và dãy so, NXB Giáo dục. [3] Nguyen Văn M u (2006), M®t so bài toán chon loc ve dãy so, NXB Giáo dục. [4] Nguyen Văn M u, Lê Ngoc Lăng, Phạm The Long, Nguyen Minh Tuan (2006), Các đe thi Olympic Sinh viên toàn quoc, NXB Giáo dục. [5] Nguyen Văn M u (Chủ biên), Tran Nam Dũng, Nguyen Minh Tuan (2007), Chuyên đe chon loc dãy so và áp dựng, NXB Giáo dục. [6] Nguyen Văn M u, Nguyen Thủy Thanh (2003), Giới hạn dãy so và hàm so, NXB Giáo dục. [7] Lê Đình Thịnh (Chủ biên), Đ ng Đình Châu, Lê Đình Định, Phan Văn Hạp (2001), Phương trình sai phân và m®t so úng dựng, NXB Giáo dục. [B] Tieng Anh [8] Radulescu.T-L.T, Radulescu.V.D, Andreescu.T (2009), Problems in Real Analysis: Advanced Calculus on the Real Axis, Springer Sci- ences+Business Media. [9] Paulo Ney de Sausa, Jorge- Nume Silva (1998), Berkeley Problems in Mathematics, Springer.